SlideShare a Scribd company logo
1 of 99
Download to read offline
Unit - II: Z - TRANSFORMS ...
Transformation OR Transform: A transform is a mathematical devices which
transforms one function to the another function. Most of the science and engineering
problems are ending with the mathematical equations like equations, ordinary
differential equations, partial differential equations and difference equations.
In the present topic, we discuss the basic definition of Z - transform, properties of Z -
transforms, Z - transforms of the standard functions, problems on Z - transforms,
initial value and final value theorem of the Z - transforms and problems, inverse Z -
transforms, definition of the inverse Z - transforms, formulas of the inverse Z
-transforms of standard functions and problems. Application of Z -transforms to
solve difference equations.
Dr. Vijaya kumar, MSRIT, Bengaluru - 560 054
Engineering Mathematics - IV - IM41 May 5, 2022 1 / 99
Continued Unit - II on Z - Transforms ...
Definition of the Z - transforms: The Z - transforms of the discrete function un,
defined for all n ≥ 0 and un = 0, for n < 0, is denoted by ZT [un] OR Z[un] OR U(z)
OR Ū(z) and is defined as,
ZT [un] = Z[un] =
∞
X
n=0
unz−n
= U(z) = Ū(z)
Where Z OR ZT is the Z - transforms operator and z is the Z - transforms parameter.
Definition of the Inverse Z - Transforms: The function U(z) is said to be
inverse Z - transforms of the function un. The inverse Z - transforms of the function
U(z) is denoted by Z−1
[U(z)] and is given by
un = Z−1
[U(z)]
where, Z−1
is the inverse Z - transforms operator.
Properties of the Z - transforms:
1. Linearity Property: If c1, c2, ..., cp all are constants then
Z[c1u1(n) + c2u2(n) + ... + cpup(n)] = c1Z[u1(n)] + c2Z[f2(x)] + ... + cpZ[up(x)]
Dr. Vijaya kumar, MSRIT, Bengaluru - 560 054
Engineering Mathematics - IV - IM41 May 5, 2022 2 / 99
Continued Unit - II on Properties of the Z - Transforms ...
Poof: We have, by definition of Z - transforms
Z[c1u1(n) + c2u2(n) + ... + cpup(n)] =
∞
X
n=0
(c1u1(n) + c2u2(n) + ... + cpup(n))z−n
Z[c1u1(n)+c2u2(n)+...+cpup(n)] = c1
∞
X
n=0
u1(n)z−n
+c2
∞
X
n=0
u2(n)z−n
+...+cp
∞
X
n=0
up(n)z−
Z[c1u1(n) + c2u2(n) + ... + cpup(n)] = c1Z[u1(n)] + c2Z[u2(n)] + ... + cpZ[up(n)]
2. Damping Property OR Damping rule: If Z[un = U(z)], then prove that (i)
Z[an
un] = U(z
a
), (ii) Z[a−n
un] = U(az).
Poof: (i) We have, by definition of Z - transforms
Z[un] =
∞
X
n=0
unz−n
= U(z)
Therefore,
Z[an
un] =
∞
X
n=0
(an
un)z−n
Dr. Vijaya kumar, MSRIT, Bengaluru - 560 054
Engineering Mathematics - IV - IM41 May 5, 2022 3 / 99
Continued Unit - II on Properties of the Z - Transforms ...
Z[an
un] =
∞
X
n=0
un
z−n
a−n
Z[an
un] =
∞
X
n=0
un(
z
a
)−n
Comparing this with the definition here instead of z is z
a
Z[an
un] = U(
z
a
)
(ii) We have, by definition of Z - transforms
Z[un] =
∞
X
n=0
unz−n
= U(z)
Therefore,
Z[a−n
un] =
∞
X
n=0
(a−n
un)z−n
Z[a−n
un] =
∞
X
n=0
un(az)−n
Dr. Vijaya kumar, MSRIT, Bengaluru - 560 054
Engineering Mathematics - IV - IM41 May 5, 2022 4 / 99
Continued Unit - II on Properties of the Z - Transforms ...
Comparing this with the definition here instead of z is az
Z[a−n
un] = U(az)
3. Left Shifting Property: If Z[un = U(z)], then prove that (i)
Z[un+1] = z[U(z) − u0],
(ii) Z[un+2] = z2
[U(z) − u0 − u1
z
],
(iii)Z[un+3] = z3
[U(z) − u0 − u1
z
− u2
z2 ] and
(iv) Z[un+k] = zk
[U(z) −
Pk−1
r=0 urzr
].
Poof: (i) We have, by definition of Z - transforms
Z[un] = U(z) =
∞
X
n=0
unz−n
∴
Z[un+1] =
∞
X
n=0
un+1z−n
Multiply and dividing by z in the RHS
Z[un+1] = (z)(
1
z
)
∞
X
n=0
un+1z−n
Dr. Vijaya kumar, MSRIT, Bengaluru - 560 054
Engineering Mathematics - IV - IM41 May 5, 2022 5 / 99
Continued Unit - II on Properties of the Z - Transforms ...
Z[un+1] = z
∞
X
n=0
un+1
1
z
z−n
Z[un+1] = z
∞
X
n=0
un+1z−1
z−n
Z[un+1] = z
∞
X
n=0
un+1z−n−1
Z[un+1] = z
∞
X
n=0
un+1z−(n+1)
Expanding the summation in the RHS
Z[un+1] = z[u1z−1
+ u2z−2
+ u3z−3
+ ...]
Now, adding and subtracting the missing term u0 in the RHS
Z[un+1] = z[u0 + u1z−1
+ u2z−2
+ u3z−3
+ ... − u0]
Z[un+1] = z[(u0 + u1z−1
+ u2z−2
+ u3z−3
+ ...) − u0]
Dr. Vijaya kumar, MSRIT, Bengaluru - 560 054
Engineering Mathematics - IV - IM41 May 5, 2022 6 / 99
Continued Unit - II on Properties of the Z - Transforms ...
Z[un+1] = z[
∞
X
n=0
unz−n
− u0]
Z[un+1] = z[U(z) − u0]
(ii) We have, by definition of Z - transforms
Z[un] = U(z) =
∞
X
n=0
unz−n
∴
Z[un+2] =
∞
X
n=0
un+2z−n
Multiply and dividing by z2
in the RHS
Z[un+2] = (z2
)(
1
z2
)
∞
X
n=0
un+2z−n
Z[un+2] = z2
∞
X
n=0
un+2
1
z2
z−n
Dr. Vijaya kumar, MSRIT, Bengaluru - 560 054
Engineering Mathematics - IV - IM41 May 5, 2022 7 / 99
Continued Unit - II on Properties of the Z - Transforms ...
Z[un+2] = z2
∞
X
n=0
un+2z−2
z−n
Z[un+2] = z2
∞
X
n=0
un+2z−n−2
Z[un+2] = z2
∞
X
n=0
un+2z−(n+2)
Expanding the summation in the RHS
Z[un+2] = z2
[u2z−2
+ u3z−3
+ u4z−4
+ ...]
Now, adding and subtracting the missing terms u0 and u1z−1
in the RHS
Z[un+2] = z2
[u0 + u1z−1
+ u2z−2
+ u3z−3
+ ... − u0 − u1z−1
]
Z[un+2] = z2
[(u0 + u1z−1
+ u2z−2
+ u3z−3
+ ...) − u0 − u1z−1
]
Z[un+2] = z2
[
∞
X
n=0
unz−n
− u0 −
u1
z
]
Z[un+2] = z2
[U(z) − u0 −
u1
z
]
Dr. Vijaya kumar, MSRIT, Bengaluru - 560 054
Engineering Mathematics - IV - IM41 May 5, 2022 8 / 99
Continued Unit - II on Properties of the Z - Transforms ...
(iii) We have, by definition of Z - transforms
Z[un] = U(z) =
∞
X
n=0
unz−n
∴
Z[un+3] =
∞
X
n=0
un+3z−n
Multiply and dividing by z3
in the RHS
Z[un+3] = (z3
)(
1
z3
)
∞
X
n=0
un+3z−n
Z[un+3] = z3
∞
X
n=0
un+3
1
z3
z−n
Z[un+3] = z3
∞
X
n=0
un+3z−3
z−n
Z[un+3] = z3
∞
X
n=0
un+3z−n−3
Dr. Vijaya kumar, MSRIT, Bengaluru - 560 054
Engineering Mathematics - IV - IM41 May 5, 2022 9 / 99
Continued Unit - II on Properties of the Z - Transforms ...
Z[un+3] = z3
∞
X
n=0
un+3z−(n+3)
Expanding the summation in the RHS
Z[un+3] = z3
[u3z−3
+ u4z−4
+ u5z−5
+ ...]
Now, adding and subtracting the missing terms u0, u1z−1
and u2z−2
in the RHS
Z[un+3] = z3
[u0 + u1z−1
+ u2z−2
+ u3z−3
+ ... − u0 − u1z−1
− u2z−2
]
Z[un+3] = z3
[(u0 + u1z−1
+ u2z−2
+ u3z−3
+ ...) − u0 − u1z−1
− u2z−2
]
Z[un+3] = z3
[
∞
X
n=0
unz−n
− u0 −
u1
z
−
u2
z2
]
Z[un+3] = z3
[U(z) − u0 −
u1
z
−
u2
z2
]
Similarly, we can prove (iv) Z[un+k] = zk
[U(z) −
Pk−1
r=0 urzr
].
Dr. Vijaya kumar, MSRIT, Bengaluru - 560 054
Engineering Mathematics - IV - IM41 May 5, 2022 10 / 99
Continued Unit - II on Properties of the Z - Transforms ...
4. Right Shifting Property: If Z[un = U(z)], then prove that
Z[un−k] = z−k
U(z), k > 0.
Property - 05: If Z[un = U(z)], then prove that Z[np
] = −z d
dz
[Z(np−1
)], where p is
any positive integer.
Poof: We have, by definition of Z - transforms
Z[np
] =
∞
X
n=0
np
z−n
Multiply and dividing by n and z in the RHS
Z[np
] = (z)
∞
X
n=0
(n)(
1
n
)np
(
1
z
)z−n
Z[np
] = z
∞
X
n=0
nn−1
np
z−1
z−n
Z[np
] = z
∞
X
n=0
nnp−1
z−n−1
(1)
Dr. Vijaya kumar, MSRIT, Bengaluru - 560 054
Engineering Mathematics - IV - IM41 May 5, 2022 11 / 99
Continued Unit - II on Properties of the Z - Transforms ...
Again, by definition of Z - transforms
Z[np−1
] =
∞
X
n=0
np−1
z−n
Differentiating w.r.t. z on both sides
d
dz
Z[np−1
] =
d
dz
[
∞
X
n=0
np−1
z−n
]
d
dz
[Z(np−1
)] =
∞
X
n=0
d
dz
[np−1
z−n
]
d
dz
[Z(np−1
)] =
∞
X
n=0
np−1
(−n)z−n−1
d
dz
[Z(np−1
)] = −
∞
X
n=0
nnp−1
z−n−1
∞
X
n=0
nnp−1
z−n−1
= −
d
dz
[Z(np−1
)] (2)
Dr. Vijaya kumar, MSRIT, Bengaluru - 560 054
Engineering Mathematics - IV - IM41 May 5, 2022 12 / 99
Continued Unit - II on Properties of the Z - Transforms ...
Substituting equation (2) in equation (1), we get
Z[np
] = z[−
d
dz
[Z(np−1
)]]
Z[np
] = −z
d
dz
[Z(np−1
)]
Hence, the property is proved.
Dr. Vijaya kumar, MSRIT, Bengaluru - 560 054
Engineering Mathematics - IV - IM41 May 5, 2022 13 / 99
Continued Unit - II on Z - Transforms of the Standard Functions ...
1. Find the Z - transforms of 1.
Solution: We know that, by definition of the Z - transforms
Z[un] =
∞
X
n=0
unz−n
= U(z)
but, here un = 1
Z[1] =
∞
X
n=0
(1)z−n
Z[1] =
∞
X
n=0
1
zn
Z[1] =
∞
X
n=0
1
z
!n
We have, by Binomial expansion
(1 − x)−1
= 1 + x + x2
+ x3
+ ... =
∞
X
n=0
xn
Dr. Vijaya kumar, MSRIT, Bengaluru - 560 054
Engineering Mathematics - IV - IM41 May 5, 2022 14 / 99
Continued Unit - II on Z - Transforms of the Standard Functions ...
here, x = 1
z
in the above equation
Z[1] = 1 −
1
z
!−1
Z[1] =
z − 1
z
!−1
Z[1] =
z
z − 1
Replace 1 by −1 , we get
Z[−1] =
z
z + 1
2. Find the Z - transforms of an
.
Solution: We know that, by definition of the Z - transforms
Z[un] =
∞
X
n=0
unz−n
= U(z)
but, here un = an
Z[an
] =
∞
X
n=0
(an
)z−n
n
∞
X an
Dr. Vijaya kumar, MSRIT, Bengaluru - 560 054
Engineering Mathematics - IV - IM41 May 5, 2022 15 / 99
Continued Unit - II on Z - Transforms of the Standard Functions ...
Z[an
] =
∞
X
n=0
(
a
z
)n
We have, by Binomial expansion
(1 − x)−1
= 1 + x + x2
+ x3
+ ... =
∞
X
n=0
xn
here, x = a
z
in the above equation
Z[an
] = 1 −
a
z
!−1
Z[an
] =
z − a
z
!−1
Z[an
] =
z
z − a
Replace a by −a , we get
Z[(−a)n
] =
z
z + a
Dr. Vijaya kumar, MSRIT, Bengaluru - 560 054
Engineering Mathematics - IV - IM41 May 5, 2022 16 / 99
Continued Unit - II on Z - Transforms of the Standard Functions ...
Similarly, the Z - transforms
(a)
Z[enθ
] = Z[(eθ
)n
] =
z
z − eθ
(b)
Z[e−nθ
] = Z[(e−θ
)n
] =
z
z − e−θ
(c)
Z[einθ
] = Z[(eiθ
)n
] =
z
z − eiθ
(d)
Z[e−inθ
] = Z[(e−iθ
)n
] =
z
z − e−iθ
(e) Z - transforms of any constant k
Z[k] =
kz
z − 1
Dr. Vijaya kumar, MSRIT, Bengaluru - 560 054
Engineering Mathematics - IV - IM41 May 5, 2022 17 / 99
Continued Unit - II on Z - Transforms of the Standard Functions ...
We know that,
I.
einθ
= cos(nθ) + i sin(nθ)
II.
e−inθ
= cos(nθ) − i sin(nθ)
III.
cos(nθ) =
einθ
+ e−inθ
2
IV.
einθ
+ e−inθ
= 2 cos(nθ)
V.
sin(nθ) =
einθ
− e−inθ
2i
VI.
einθ
− e−inθ
= 2i sin(nθ)
VII.
sinh(nθ) =
enθ
− e−nθ
2
VIII.
cosh(nθ) =
enθ
+ e−nθ
2
Dr. Vijaya kumar, MSRIT, Bengaluru - 560 054
Engineering Mathematics - IV - IM41 May 5, 2022 18 / 99
Continued Unit - II on Z - Transforms of the Standard Functions ...
3. Obtain the Z - transforms of sinh(nθ) and cosh(nθ).
Solution: (i) To obtain the Z - transforms of sinh(nθ)
We know that,
Z[enθ
] =
z
z − eθ
and
Z[e−nθ
] =
z
z − e−θ
Also, we have
sinh(nθ) =
enθ
− e−nθ
2
Applying the Z - transforms on the both sides
Z[sinh(nθ)] = Z[
enθ
− e−nθ
2
]
Z[sinh(nθ)] =
1
2
"
Z[enθ
] − Z[e−nθ
]
#
Substituting Z[enθ
] = z
z−eθ and Z[e−nθ
] = z
z−e−θ
Dr. Vijaya kumar, MSRIT, Bengaluru - 560 054
Engineering Mathematics - IV - IM41 May 5, 2022 19 / 99
Continued Unit - II on Z - Transforms of the Standard Functions ...
Z[sinh(nθ)] =
1
2
"
z
z − eθ
−
z
z − e−θ
#
Z[sinh(nθ)] =
1
2
"
z(z − e−θ
) − z(z − eθ
)
(z − eθ)(z − e−θ)
#
Z[sinh(nθ)] =
1
2
[
z2
− ze−θ
− z2
+ zeθ
z2 − ze−θ − zeθ + eθe−θ
]
Z[sinh(nθ)] =
1
2
"
z(eθ
− e−θ
)
z2 − z(eθ + e−θ) + e0
#
Z[sinh(nθ)] =
1
2
"
z(2 sinh(θ))
z2 − z(2 cosh(θ)) + 1
#
Z[sinh(nθ)] =
1
1
"
z(sinh(θ))
z2 − 2z cosh(θ) + 1
#
Z[sinh(nθ)] =
z sinh(θ)
z2 − 2z cosh(θ) + 1
Dr. Vijaya kumar, MSRIT, Bengaluru - 560 054
Engineering Mathematics - IV - IM41 May 5, 2022 20 / 99
Continued Unit - II on Z - Transforms of the Standard Functions ...
(ii) To obtain the Z - transforms of cosh(nθ)
We know that,
Z[enθ
] =
z
z − eθ
and
Z[e−nθ
] =
z
z − e−θ
Also, we have
cosh(nθ) =
enθ
+ e−nθ
2
Applying the Z - transforms on the both sides
Z[cosh(nθ)] = Z[
enθ
+ e−nθ
2
]
Z[cosh(nθ)] =
1
2
"
Z[enθ
] + Z[e−nθ
]
#
Substituting Z[enθ
] = z
z−eθ and Z[e−nθ
] = z
z−e−θ
Dr. Vijaya kumar, MSRIT, Bengaluru - 560 054
Engineering Mathematics - IV - IM41 May 5, 2022 21 / 99
Continued Unit - II on Z - Transforms of the Standard Functions ...
Z[cosh(nθ)] =
1
2
"
z
z − eθ
+
z
z − e−θ
#
Z[cosh(nθ)] =
1
2
"
z(z − e−θ
) + z(z − eθ
)
(z − eθ)(z − e−θ)
#
Z[cosh(nθ)] =
1
2
"
z2
− ze−θ
+ z2
− zeθ
z2 − ze−θ − zeθ + eθe−θ
#
Z[cosh θ)] =
1
2
"
2z2
− z(eθ
+ e−θ
)
z2 − z(eθ + e−θ) + e0
#
Z[cosh(nθ)] =
1
2
"
2z2
− z(2 cosh(θ))
z2 − z(2 cosh(θ)) + 1
#
Z[cosh(nθ)] =
1
1
"
z2
− z(cosh(θ))
z2 − 2z cosh(θ) + 1
#
Z[cosh(nθ)] =
z2
− z cosh(θ)
z2 − 2z cosh(θ) + 1
Dr. Vijaya kumar, MSRIT, Bengaluru - 560 054
Engineering Mathematics - IV - IM41 May 5, 2022 22 / 99
Continued Unit - II on Z - Transforms of the Standard Functions ...
4. Obtain the Z - transforms of sin(nθ) and cos(nθ).
Solution: We know that,
einθ
= cos(nθ) + i sin(nθ)
Applying Z - transforms on both sides
Z[einθ
] = Z[cos(nθ) + i sin(nθ)]
Z[cos(nθ)] + iZ[sin(nθ)] = Z[einθ
]
Substituting Z[einθ
] = z
z−eiθ in the RHS
Z[cos(nθ)] + iZ[sin(nθ)] =
z
z − eiθ
Z[cos(nθ)] + iZ[sin(nθ)] =
z
z − (cos(θ) + i sin(θ))
Z[cos(nθ)] + iZ[sin(nθ)] =
z
(z − cos(θ)) − i sin(θ)
Now, rationalize RHS
Dr. Vijaya kumar, MSRIT, Bengaluru - 560 054
Engineering Mathematics - IV - IM41 May 5, 2022 23 / 99
Continued Unit - II on Z - Transforms of the Standard Functions ...
Z[cos(nθ)] + iZ[sin(nθ)] =
z((z − cos(θ)) + i sin(θ))
((z − cos(θ)) − i sin(θ))((z − cos(θ)) + i sin(θ))
Z[cos(nθ)] + iZ[sin(nθ)] =
(z2
− z cos(θ)) + iz sin(θ)
(z − cos(θ))2 − (i sin(θ))2
Z[cos(nθ)] + iZ[sin(nθ)] =
(z2
− z cos(θ)) + iz sin(θ)
z2 − 2z cos(θ) + cos2(θ) − i2 sin2
(θ)
Z[cos(nθ)] + iZ[sin(nθ)] =
(z2
− z cos(θ)) + iz sin(θ)
z2 − 2z cos(θ) + cos2(θ) + sin2
(θ)
Z[cos(nθ)] + iZ[sin(nθ)] =
(z2
− z cos(θ)) + iz sin(θ)
z2 − 2z cos(θ) + 1
Separate the real and imaginary parts in the RHS
Z[cos(nθ)] + iZ[sin(nθ)] =
z2
− z cos(θ)
z2 − 2z cos(θ) + 1
+ i
z sin(θ)
z2 − 2z cos(θ) + 1
Comparing the real and imaginary parts on both sides, we get
Dr. Vijaya kumar, MSRIT, Bengaluru - 560 054
Engineering Mathematics - IV - IM41 May 5, 2022 24 / 99
Continued Unit - II on Z - Transforms of the Standard Functions ...
Z[sin(nθ)] =
z sin(θ)
z2 − 2z cos(θ) + 1
and
Z[cos(nθ)] =
z2
− z cos(θ)
z2 − 2z cos(θ) + 1
5. Find the Z - transforms of n.
Solution: We know that, by property
Z[np
] = −z
d
dz
[Z(np−1
)]
Substitute p = 1
Z[n1
] = −z
d
dz
[Z(n1−1
)]
Z[n] = −z
d
dz
[Z(n0
)]
Z[n] = −z
d
dz
[Z(1)]
substituting Z[1] = z
z−1
in the RHS
Dr. Vijaya kumar, MSRIT, Bengaluru - 560 054
Engineering Mathematics - IV - IM41 May 5, 2022 25 / 99
Continued Unit - II on Z - Transforms of the Standard Functions ...
Z[n] = −z
d
dz
[
z
z − 1
]
Differentiate using the quotient rule
Z[n] = −z[
(z − 1)(1) − (z)(1)
(z − 1)2
]
Z[n] = −z[
z − 1 − z
(z − 1)2
]
Z[n] = −z[
−1
(z − 1)2
]
Z[n] =
z
(z − 1)2
6. Find the Z - transforms of n2
.
Solution: We know that, by property
Z[np
] = −z
d
dz
[Z(np−1
)]
Substitute p = 2
Z[n2
] = −z
d
dz
[Z(n2−1
)]
Dr. Vijaya kumar, MSRIT, Bengaluru - 560 054
Engineering Mathematics - IV - IM41 May 5, 2022 26 / 99
Continued Unit - II on Z - Transforms of the Standard Functions ...
Z[n2
] = −z
d
dz
[Z(n1
)]
Z[n2
] = −z
d
dz
[Z(n)]
substituting Z[n] = z
(z−1)2 in the RHS
Z[n2
] = −z
d
dz
[
z
(z − 1)2
]
Differentiate using the quotient rule
Z[n2
] = −z[
(z − 1)2
(1) − (z)2(z − 1)(1)
(z − 1)4
]
Z[n2
] = −z(z − 1)[
z − 1 − 2z
(z − 1)4
]
Z[n2
] = −z[
−z − 1
(z − 1)3
]
Z[n2
] =
z2
+ z
(z − 1)3
Dr. Vijaya kumar, MSRIT, Bengaluru - 560 054
Engineering Mathematics - IV - IM41 May 5, 2022 27 / 99
Continued Unit - II on Z - Transforms of the Standard Functions ...
7. Find the Z - transforms of n3
.
Solution: We know that, by property
Z[np
] = −z
d
dz
[Z(np−1
)]
Substitute p = 3
Z[n3
] = −z
d
dz
[Z(n3−1
)]
Z[n3
] = −z
d
dz
[Z(n2
)]
substituting Z[n2
] = z2
+z
(z−1)3 in the RHS
Z[n3
] = −z
d
dz
[fracz2
+ z(z − 1)3
]
Differentiate using the quotient rule
Z[n3
] = −z[
(z − 1)3
(2z + 1) − (z2
+ z)3(z − 1)2
(1)
(z − 1)6
]
Z[n3
] = −z(z − 1)2
[
(z − 1)(2z + 1) − 3(z2
+ z)
(z − 1)6
]
Dr. Vijaya kumar, MSRIT, Bengaluru - 560 054
Engineering Mathematics - IV - IM41 May 5, 2022 28 / 99
Continued Unit - II on Z - Transforms of the Standard Functions ...
Z[n3
] = −z[
2z2
+ z − 2z − 1 − 3z2
− 3z
(z − 1)4
]
Z[n3
] = −z[
−z2
− 4z − 1
(z − 1)4
]
Z[n3
] =
z3
+ 4z2
+ z
(z − 1)4
8. Find the Z - transforms of n4
.
Solution: We know that, by property
Z[np
] = −z
d
dz
[Z(np−1
)]
Substitute p = 4
Z[n4
] = −z
d
dz
[Z(n4−1
)]
Z[n4
] = −z
d
dz
[Z(n3
)]
Dr. Vijaya kumar, MSRIT, Bengaluru - 560 054
Engineering Mathematics - IV - IM41 May 5, 2022 29 / 99
Continued Unit - II on Z - Transforms of the Standard Functions ...
substituting Z[n3
] = z3
+4z2
+z
(z−1)4 in the RHS
Z[n4
] = −z
d
dz
[
z3
+ 4z2
+ z
(z − 1)4
]
Differentiate using the quotient rule
Z[n4
] = −z[
(z − 1)4
(3z2
+ 8z + 1) − (z3
+ 4z2
+ z)4(z − 1)3
(1)
(z − 1)8
]
Z[n4
] = −z(z − 1)3
[
(z − 1)(3z2
+ 8z + 1) − 4(z3
+ 4z2
+ z)
(z − 1)8
]
Z[n4
] = −z[
(z − 1)(3z2
+ 8z + 1) − 4z3
− 16z2
− 4z
(z − 1)5
]
Z[n4
] = −z[
3z3
+ 8z2
+ z − 3z2
− 8z − 1 − 4z3
− 16z2
− 4z
(z − 1)5
]
Z[n4
] = −z[
−z3
− 11z2
− 11z − 1
(z − 1)5
]
Z[n4
] =
z4
+ 11z3
+ 11z2
+ z
(z − 1)5
Dr. Vijaya kumar, MSRIT, Bengaluru - 560 054
Engineering Mathematics - IV - IM41 May 5, 2022 30 / 99
Continued Unit - II on Z - Transforms of the Standard Functions ...
9. Find the Z - transforms of 1
n
, for n ≥ 1.
Solution: We know that, by definition of the Z - transforms
Z
"
1
n
#
=
∞
X
n=1
1
n
!
z−n
Expanding the summation in the RHS
Z
"
1
n
#
=
1
1
!
z−1
+
1
2
!
z−2
+
1
3
!
z−3
+ ...
Z
"
1
n
#
=
1
z
+
1
2z2
+
1
3z3
+ ...
Z
"
1
n
#
=
(1
z
)
1
+
(1
z
)2
2
+
(1
z
)3
3
+ ...
Z[
1
n
] = −[−
1
z
!
1
−
1
z
!2
2
−
1
z
!3
3
− ...]
We know that, by logarithmic series
log(1 − x) = −x −
x2
−
x3
− ...
Dr. Vijaya kumar, MSRIT, Bengaluru - 560 054
Engineering Mathematics - IV - IM41 May 5, 2022 31 / 99
Continued Unit - II on Z - Transforms of the Standard Functions ...
Here, x = 1
z
Z
"
1
n
#
= −
"
log 1 −
1
z
!#
Z
"
1
n
#
= − log
z − 1
z
!
Z
"
1
n
#
= log
z − 1
z
!−1
Z
"
1
n
#
= log
z
z − 1
!
10. Find the Z - transforms of 1
n+1
, for n ≥ 0.
Solution: We know that, by definition of the Z - transforms
Z
"
1
n + 1
#
=
∞
X
n=0
1
n + 1
!
z−n
Expanding the summation in the RHS
Z
"
1
n + 1
#
=
1
1
!
z0
+
1
2
!
z−1
+
1
3
!
z−2
+
1
4
!
z−3
+ ...
Dr. Vijaya kumar, MSRIT, Bengaluru - 560 054
Engineering Mathematics - IV - IM41 May 5, 2022 32 / 99
Continued Unit - II on Z - Transforms of the Standard Functions ...
Multiply and dividing by −z in the RHS
Z
"
1
n + 1
#
= (−z)[−
(1
z
)
1
−
(1
z
)2
2
−
(1
z
)3
3
− ...]
We know that, by logarithmic series
log(1 − x) = −x −
x2
2
−
x3
3
− ...
Z
"
1
n + 1
#
= (−z)[log(1 −
1
z
)]
Z
"
1
n + 1
#
= −z log
z − 1
z
!
Z
"
1
n + 1
#
= z log
z − 1
z
!−1
Z
"
1
n + 1
#
= z log
z
z − 1
!
Dr. Vijaya kumar, MSRIT, Bengaluru - 560 054
Engineering Mathematics - IV - IM41 May 5, 2022 33 / 99
Continued Unit - II on Z - Transforms of the Standard Functions ...
11. Define unit step sequence for discrete and hence find its Z - transforms.
Solution:
Definition of the unit step sequence for discrete:
The unit step sequence for discrete is denoted by u(n) or H(n) and is defined as
u(n) = H(n) =
(
1, for n ≥ 0
0, for n < 0
To find the Z - transforms of the unit step sequence for discrete:
We have, by definition of Z - transforms
Z[H(n)] =
∞
X
n=0
(H(n))z−n
Expanding the summation in the RHS
Z[H(n)] = H(0)z0
+ H(1)z−1
+ H(2)z−2
+ H(3)z−3
+ ...
Z[H(n)] = 1 +
1
z
!
+
1
z
!2
+
1
z
!3
+ ...
Dr. Vijaya kumar, MSRIT, Bengaluru - 560 054
Engineering Mathematics - IV - IM41 May 5, 2022 34 / 99
Continued Unit - II on Z - Transforms of the Standard Functions ...
We have, by Binomial expansion
(1 − x)−1
= 1 + x + x2
+ x3
+ ...
Z[H(n)] = 1 −
1
z
!−1
Z[H(n)] =
z − 1
z
!−1
Z[H(n)] =
z
z − 1
12. Define unit impulse sequence for discrete and hence find its Z - transforms.
Solution:
Definition of the unit impulse sequence for discrete:
The unit impulse sequence for discrete is denoted by δ(n) and is defined as
δ(n) =
(
1, for n = 0
0, for n 6= 0
Dr. Vijaya kumar, MSRIT, Bengaluru - 560 054
Engineering Mathematics - IV - IM41 May 5, 2022 35 / 99
Continued Unit - II on Z - Transforms of the Standard Functions ...
To find the Z - transforms of the unit impulse sequence for discrete:
We have, by definition of Z - transforms
Z[δ(n)] =
∞
X
n=0
(δ(n))z−n
Expanding the summation in the RHS
Z[δ(n)] = δ(0)z0
+ δ(1)z−1
+ δ(2)z−2
+ δ(3)z−3
+ ...
Z[δ(n)] = 1 + (0) + (0) + (0) + ...
Z[δ(n)] = 1
Dr. Vijaya kumar, MSRIT, Bengaluru - 560 054
Engineering Mathematics - IV - IM41 May 5, 2022 36 / 99
Continued Unit - II on Formulas of the Z - Transforms of the Standard
Functions ...
1.
Z[1] =
z
z − 1
2.
Z[−1] =
z
z + 1
3.
Z[an
] =
z
z − a
4.
Z[(−a)n
] =
z
z + a
5.
Z[sinh(nθ)] =
z sinh(θ)
z2 − 2z cosh(θ) + 1
6.
Z[cosh(nθ)] =
z2
− z cosh(θ)
z2 − 2z cosh(θ) + 1
Dr. Vijaya kumar, MSRIT, Bengaluru - 560 054
Engineering Mathematics - IV - IM41 May 5, 2022 37 / 99
Continued Unit - II on Formulas of the Z - Transforms of the Standard
Functions ...
7.
Z[sin(nθ)] =
z sin(θ)
z2 − 2z cos(θ) + 1
8.
Z[cos(nθ)] =
z2
− z cos(θ)
z2 − 2z cos(θ) + 1
9.
Z[n] =
z
(z − 1)2
10.
Z[n2
] =
z2
+ z
(z − 1)3
11.
Z[n3
] =
z3
+ 4z2
+ z
(z − 1)4
Dr. Vijaya kumar, MSRIT, Bengaluru - 560 054
Engineering Mathematics - IV - IM41 May 5, 2022 38 / 99
Continued Unit - II on Formulas of the Z - Transforms of the Standard
Functions ...
12.
Z[n4
] =
z4
+ 11z3
+ 11z2
+ z
(z − 1)5
13.
Z[
1
n
] = log(
z
z − 1
)
14.
Z[
1
n + 1
] = z log(
z
z − 1
)
15.
Z[H(n)] =
z
z − 1
16.
Z[δ(n)] = 1
17.
Z[k] =
kz
z − 1
Dr. Vijaya kumar, MSRIT, Bengaluru - 560 054
Engineering Mathematics - IV - IM41 May 5, 2022 39 / 99
Continued Unit - II Problems on Z - Transforms ...
Problem - 01 : Find the Z - transforms of the 3n + sin(5n) − cosh(4n) + 5.
Solution: By data 3n + sin(5n) − cosh(4n) + 5
Applying the Z - transforms for the problem
Z[3n + sin(5n) − cosh(4n) + 5] = 3Z[n] + Z[sin(5n)] − Z[cosh(4n)] + Z[5]
Z[3n+sin(5n)−cosh(4n)+5] = 3(
z
(z − 1)2
)+(
z sin(5)
z2 − 2z cos(5) + 1
)−(
z2
− z cosh(4)
z2 − 2z cosh(4) + 1
)+
Z[3n+sin(5n)−cosh(4n)+5] =
3z
(z − 1)2
+
z sin(5)
z2 − 2z cos(5) + 1
−
z2
− z cosh(4)
z2 − 2z cosh(4) + 1
+
5z
z − 1
Problem - 02 : Find the Z - transforms of the cos(3n + 2).
Solution: By data cos(3n + 2)
cos(3n + 2) = cos(3n) cos(2) − sin(3n) sin(2)
Applying the Z - transforms on both sides
Z[cos(3n + 2)] = Z[cos(3n) cos(2) − sin(3n) sin(2)]
Z[cos(3n + 2)] = Z[cos(3n) cos(2)] − Z[sin(3n) sin(2)]
Z[cos(3n + 2)] = (cos(2))Z[cos(3n)] − (sin(2))Z[sin(3n)]
Dr. Vijaya kumar, MSRIT, Bengaluru - 560 054
Engineering Mathematics - IV - IM41 May 5, 2022 40 / 99
Continued Unit - II Problems on Z - Transforms ...
Z[cos(3n + 2)] = (cos(2))(
z2
− z cos(3)
z2 − 2z cos(3) + 1
) − (sin(2))(
z sin(3)
z2 − 2z cos(3) + 1
)
Z[cos(3n + 2)] =
z2
cos(2) − z cos(3) cos(2) − z sin(3) sin(2)
z2 − 2z cos(3) + 1
Z[cos(3n + 2)] =
z2
cos(2) − z(cos(3) cos(2) + sin(3) sin(2))
z2 − 2z cos(3) + 1
Z[cos(3n + 2)] =
z2
cos(2) − z(cos(3 − 2))
z2 − 2z cos(3) + 1
Z[cos(3n + 2)] =
z2
cos(2) − z cos(1)
z2 − 2z cos(3) + 1
Problem - 03 : Find the Z - transforms of the sin(π
4
− nπ
2
).
Solution: Given sin(π
4
− nπ
2
)
sin(
π
4
−
nπ
2
) = sin(
π
4
) cos(
nπ
2
) − cos(
π
4
) sin(
nπ
2
)
But, sin(π
4
) = 1
√
2
and cos(π
4
) = 1
√
2
Dr. Vijaya kumar, MSRIT, Bengaluru - 560 054
Engineering Mathematics - IV - IM41 May 5, 2022 41 / 99
Continued Unit - II Problems on Z - Transforms ...
sin(
π
4
−
nπ
2
) = (
1
√
2
) cos(
nπ
2
) − (
1
√
2
) sin(
nπ
2
)
Applying the Z - transforms on both sides
Z[sin(
π
4
−
nπ
2
)] = (
1
√
2
)Z[cos(
nπ
2
) − sin(
nπ
2
)]
Z[sin(
π
4
−
nπ
2
)] =
1
√
2
[(
z2
− z cos(π
2
)
z2 − 2z cos(π
2
) + 1
) − (
z sin(π
2
)
z2 − 2z cos(π
2
) + 1
)]
sin(π
2
) = 1 and cos(π
2
) = 0
Z[sin(
π
4
−
nπ
2
)] =
1
√
2
[(
z2
− z(0)
z2 − 2z(0) + 1
) − (
z(1)
z2 − 2z(0) + 1
)]
Z[sin(
π
4
−
nπ
2
)] =
z2
− z
√
2(z2 + 1)
Problem - 04 : Find the Z - transforms of the 4n2
+ cos(nπ
4
) + 10a3
.
Solution: Given 4n2
+ cos(nπ
4
) + 10a3
Applying the z - transforms for the given
problem
Z[4n2
+ cos(
nπ
4
) + 10a3
] = 4Z[n2
] + Z[cos(
nπ
4
)] + Z[10a3
]
Dr. Vijaya kumar, MSRIT, Bengaluru - 560 054
Engineering Mathematics - IV - IM41 May 5, 2022 42 / 99
Continued Unit - II Problems on Z - Transforms ...
Z[4n2
+ cos(
nπ
4
) + 10a3
] = 4(
z2
+ z
(z − 1)3
) + (
z2
− z cos(π
4
)
z2 − 2z cos(π
4
) + 1
) + (
10a3
z
z − 1
)
but, cos(π
4
= 1
√
2
Z[4n2
+ cos(
nπ
4
) + 10a3
] =
4(z2
+ z)
(z − 1)3
+
z2
− z( 1
√
2
)
z2 − 2z( 1
√
2
) + 1
+
10a3
z
z − 1
Z[4n2
+ cos(
nπ
4
) + 10a3
] =
4(z2
+ z)
(z − 1)3
+
√
2z2
− z
√
2z2 − 2z +
√
2
+
10a3
z
z − 1
Problem - 05 : Find the Z - transforms of the (3n + 2)2
.
Solution: By data (3n + 2)2
(3n + 2)2
= (3n)2
+ (2)2
+ 2(3n)(2)
(3n + 2)2
= 9n2
+ 4 + 12n
Applying the z - transforms on both sides
Z[(3n + 2)2
] = Z[9n2
+ 4 + 12n]
Z[(3n + 2)2
] = 9Z[n2
] + 4Z[1] + 12Z[n]
Dr. Vijaya kumar, MSRIT, Bengaluru - 560 054
Engineering Mathematics - IV - IM41 May 5, 2022 43 / 99
Continued Unit - II Problems on Z - Transforms ...
Z[(3n + 2)2
] = 9(
z2
+ z
(z − 1)3
) + 4(
z
z − 1
) + 12(
z
(z − 1)2
)
Z[(3n + 2)2
] =
9(z2
+ z)
(z − 1)3
+
4z
z − 1
+
12z
(z − 1)2
Problem - 06 : Find the Z - transforms of the (2n − 1)3
.
Solution: By data (2n − 1)3
(2n − 1)3
= (2n)3
− 3(2n)2
(1) + 3(2n)(1)2
− (1)3
(2n − 1)3
= 8n3
− 12n2
+ 6n − 1
Applying the z - transforms on both sides
Z[(2n − 1)3
] = Z[8n3
− 12n2
+ 6n − 1]
Z[(2n − 1)3
] = 8Z[n3
] − 12Z[n2
] + 6Z[n] − Z[1]
Z[(2n − 1)3
] = 8(
z3
+ 4z2
+ z
(z − 1)4
) − 12(
z2
+ z
(z − 1)3
) + 6(
z
(z − 1)2
) − (
z
z − 1
)
Z[(2n − 1)3
] =
8(z3
+ 4z2
+ z)
(z − 1)4
−
12(z2
+ z)
(z − 1)3
+
6z
(z − 1)2
−
z
z − 1
Dr. Vijaya kumar, MSRIT, Bengaluru - 560 054
Engineering Mathematics - IV - IM41 May 5, 2022 44 / 99
Continued Unit - II Problems on Z - Transforms ...
Problem - 07 : Find the Z - transforms of the an
n.
Solution: By data an
n
We know that, by damping property
Z[an
un] = U(
z
a
)
Here, un = n and U(z) = Z[n] = z
(z−1)2 , Replace z by z
a
Z[an
n] = [
z
(z − 1)2
]z→ z
a
Z[an
n] = [
(z
a
)
(z
a
− 1)2
]
Z[an
n] =
az
(z − a)2
Problem - 08 : Find the Z - transforms of the a−n
n2
.
Solution: By data a−n
n2
We know that, by damping property
Z[a−n
un] = U(az)
Here, un = n2
and U(z) = Z[n2
] = z2
+z
(z−1)3 , Replace z by az
Dr. Vijaya kumar, MSRIT, Bengaluru - 560 054
Engineering Mathematics - IV - IM41 May 5, 2022 45 / 99
Continued Unit - II Problems on Z - Transforms ...
Z[a−n
n2
] = [
z2
+ z
(z − 1)3
]z→az
Z[a−n
n2
] = [
(az)2
+ az
(az − 1)3
]
Z[a−n
n2
] =
a2
z2
+ az
(az − 1)3
Problem - 09 : Find the Z - transforms of the an
n2
.
Solution: By data an
n2
We know that, by damping property
Z[an
un] = U(
z
a
)
Here, un = n2
and U(z) = Z[n2
] = z2
+z
(z−1)3 , Replace z by z
a
Z[an
n2
] = [
z2
+ z
(z − 1)3
]z→ z
a
Dr. Vijaya kumar, MSRIT, Bengaluru - 560 054
Engineering Mathematics - IV - IM41 May 5, 2022 46 / 99
Continued Unit - II Problems on Z - Transforms ...
Z[an
n2
] = [
(z
a
)2
+ (z
a
)
(z
a
− 1)3
]
Z[an
n2
] =
az2
+ a2
z
(z − a)3
Problem - 10 : Find the Z - transforms of the a−n
ebn
.
Solution: By data a−n
ebn
We know that, by damping property
Z[a−n
un] = U(az)
Here, un = ebn
and U(z) = Z[ebn
] = z
z−eb , Replace z by az
Z[a−n
ebn
] = [
z
z − eb
]z→az
Z[a−n
ebn
] = [
az
az − eb
]
Z[a−n
ebn
] =
az
az − eb
Dr. Vijaya kumar, MSRIT, Bengaluru - 560 054
Engineering Mathematics - IV - IM41 May 5, 2022 47 / 99
Continued Unit - II Problems on Z - Transforms ...
Problem - 11 : Find the Z - transforms of the an
cos(nθ).
Solution: By data an
cos(nθ)
We know that, by damping property
Z[an
un] = U(
z
a
)
Here, un = cos(nθ) and U(z) = Z[cos(nθ)] = z2
−z cos(θ)
z2−2z cos(θ)+1
, Replace z by z
a
Z[an
cos(nθ)] = [
z2
− z cos(θ)
z2 − 2z cos(θ) + 1
]z→ z
a
Z[an
cos(nθ)] =
(z
a
)2
− (z
a
) cos(θ)
(z
a
)2 − 2(z
a
) cos(θ) + 1
Z[an
cos(nθ)] =
z2
− az cos(θ)
z2 − 2az cos(θ) + a2
Problem - 12 : Find the Z - transforms of the a−n
sin(nθ).
Solution: By data a−n
sin(nθ)
We know that, by damping property
Z[a−n
un] = U(az)
Here, un = sin(nθ) and U(z) = Z[sin(nθ)] = z sin(θ)
z2−2z cos(θ)+1
, Replace z by az
Dr. Vijaya kumar, MSRIT, Bengaluru - 560 054
Engineering Mathematics - IV - IM41 May 5, 2022 48 / 99
Continued Unit - II Problems on Z - Transforms ...
Z[a−n
sin(nθ)] = [
z sin(θ)
z2 − 2z cos(θ) + 1
]z→az
Z[a−n
sin(nθ)] =
az sin(θ)
(az)2 − 2az cos(θ) + 1
Z[a−n
sin(nθ)] =
az sin(θ)
a2z2 − 2az cos(θ) + 1
Problem - 13 : Find the Z - transforms of the an
sinh(nθ).
Solution: By data an
sinh(nθ)
We know that, by damping property
Z[an
un] = U(
z
a
)
Here, un = sinh(nθ) and U(z) = Z[sinh(nθ)] = z sinh(θ)
z2−2z cosh(θ)+1
, Replace z by z
a
Z[an
sinh(nθ)] = [
z sinh(θ)
z2 − 2z cosh(θ) + 1
]z→ z
a
Z[an
sinh(nθ)] =
(z
a
) sinh(θ)
(z
a
)2 − 2(z
a
) cosh(θ) + 1
Dr. Vijaya kumar, MSRIT, Bengaluru - 560 054
Engineering Mathematics - IV - IM41 May 5, 2022 49 / 99
Continued Unit - II Problems on Z - Transforms ...
Z[an
sinh(nθ)] =
az sinh(θ)
z2 − 2az cosh(θ) + a2
Problem - 14 : Find the Z - transforms of the a−n
cosh(nθ).
Solution: By data a−n
cosh(nθ)
We know that, by damping property
Z[a−n
un] = U(az)
Here, un = cosh(nθ) and U(z) = Z[cosh(nθ)] = z2
−z cosh(θ)
z2−2z cosh(θ)+1
, Replace z by az
Z[a−n
cosh(nθ)] = [
z2
− z cosh(θ)
z2 − 2z cosh(θ) + 1
]z→az
Z[a−n
cosh(nθ)] =
(az)2
− (az) cosh(θ)
(az)2 − 2(az) cosh(θ) + 1
Z[a−n
cosh(nθ)] =
a2
z2
− az cosh(θ)
a2z2 − 2az cos(θ) + 1
Dr. Vijaya kumar, MSRIT, Bengaluru - 560 054
Engineering Mathematics - IV - IM41 May 5, 2022 50 / 99
Continued Unit - II Problems on Z - Transforms ...
Problem - 15 : Find the Z - transforms of the 1
n(n+1)
.
Solution: By data 1
n(n+1)
, adding and subtracting n in the numerator
1
n(n + 1)
=
n + 1 − n
n(n + 1)
1
n(n + 1)
=
(n + 1) − n
n(n + 1)
1
n(n + 1)
=
(n + 1)
n(n + 1)
−
n
n(n + 1)
1
n(n + 1)
=
1
n
−
1
n + 1
Applying the Z - transforms on both sides
Z[
1
n(n + 1)
] = Z[
1
n
] − Z[
1
n + 1
]
We know that
Z[
1
n
] = log(
z
z − 1
)
and
Z[
1
n + 1
] = z log(
z
z − 1
)
Dr. Vijaya kumar, MSRIT, Bengaluru - 560 054
Engineering Mathematics - IV - IM41 May 5, 2022 51 / 99
Continued Unit - II Problems on Z - Transforms ...
Using these results in the above equation
Z[
1
n(n + 1)
] = (log(
z
z − 1
)) − (z log(
z
z − 1
))
Z[
1
n(n + 1)
] = (1 − z) log(
z
z − 1
)
Problem - 16 : Find the Z - transforms of the n
n+1
.
Solution: By data n
n+1
, adding and subtracting 1 in the numerator
n
n + 1
=
n + 1 − 1
n + 1
n
n + 1
=
(n + 1) − 1
n + 1
n
n + 1
=
(n + 1)
n + 1
−
1
n + 1
n
n + 1
= 1 −
1
n + 1
Applying the Z - transforms on both sides
Z[
n
n + 1
] = Z[1] − Z[
1
n + 1
]
Dr. Vijaya kumar, MSRIT, Bengaluru - 560 054
Engineering Mathematics - IV - IM41 May 5, 2022 52 / 99
Continued Unit - II Problems on Z - Transforms ...
We know that
Z[1] =
z
z − 1
and
Z[
1
n + 1
] = z log(
z
z − 1
)
Using these results in the above equation
Z[
n
n + 1
] =
z
z − 1
− z log(
z
z − 1
)
Problem - 17 : Show that ZT [ 1
n!
] = e
1
z , and hence evaluate (i) ZT [ 1
(n+1)!
],
(ii) ZT [ 1
(n+2)!
].
Solution: We have, by definition of Z - transform
ZT [
1
n!
] =
∞
X
n=0
(
1
n!
)z−n
Expanding the summation in the RHS
ZT [
1
n!
] = [(
1
0!
)z0
+ (
1
1!
)z−1
+ (
1
2!
)z−2
+ (
1
3!
)z−3
+ ...]
Dr. Vijaya kumar, MSRIT, Bengaluru - 560 054
Engineering Mathematics - IV - IM41 May 5, 2022 53 / 99
Continued Unit - II Problems on Z - Transforms ...
ZT [
1
n!
] = 1 +
(1
z
)
1!
+
(1
z
)2
2!
+
(1
z
)3
3!
+ ...
We know that, by exponential series
ex
= 1 +
x
1!
+
x2
2!
+
x3
3!
+ ...
Here x = 1
z
ZT [
1
n!
] = e
1
z
(i) To evaluate ZT [ 1
(n+1)!
]
Taking un = 1
n!
, un+1 = 1
(n+1)!
, u0 = 1
0!
= 1, and U(z) = ZT [un] = ZT [ 1
n!
] = e
1
z
We have, by shifting property
ZT [un+1] = z[U(z) − u0]
Now, substituting all results
ZT [
1
(n + 1)!
] = z[e
1
z − 1]
Dr. Vijaya kumar, MSRIT, Bengaluru - 560 054
Engineering Mathematics - IV - IM41 May 5, 2022 54 / 99
Continued Unit - II Problems on Z - Transforms ...
(ii) To evaluate ZT [ 1
(n+2)!
]
Taking un = 1
n!
, un+2 = 1
(n+2)!
, u0 = 1
0!
= 1, u1 = 1
1!
= 1, and
U(z) = ZT [un] = ZT [ 1
n!
] = e
1
z
We have, by shifting property
ZT [un+2] = z2
[U(z) − u0 −
u1
z
]
Now, substituting all results
ZT [
1
(n + 2)!
] = z2
[e
1
z − 1 −
1
z
]
Dr. Vijaya kumar, MSRIT, Bengaluru - 560 054
Engineering Mathematics - IV - IM41 May 5, 2022 55 / 99
Continued Unit - II on Initial Value of the Z - Transforms ...
Initial Value Theorem of the Z - transforms:
Statement: If Z[un = U(z)], then prove that u0 = lim
z→
U(z). Hence prove that
(i) u1 = lim
z→
z[U(z) − u0],
(ii) u2 = lim
z→
z2
[U(z) − u0 − u1
z
],
(iii) u3 = lim
z→
z3
[U(z) − u0 − u1
z
− u2
z2 ].
Proof: We know that, by definition of the Z - transforms
Z[un] = U(z) =
∞
X
n=0
unz−n
Expanding the summation in the RHS
U(z) = u0z0
+ u1z−1
+ u2z−2
+ ...
U(z) = u0(1) +
u1
z
+
u2
z2
+ ...
Applying the limit z → ∞ on both sides
lim
z→∞
U(z) = lim
z→∞
[u0 +
u1
z
+
u2
z2
+ ...]
lim
z→∞
U(z) = [u0 + (0) + (0) + ...]
Dr. Vijaya kumar, MSRIT, Bengaluru - 560 054
Engineering Mathematics - IV - IM41 May 5, 2022 56 / 99
Continued Unit - II on Initial Value of the Z - Transforms ...
u0 = lim
z→∞
U(z)
Hence initial value theorem is proved (i) To prove this u1 = lim
z→
z[U(z) − u0],
We know that, by definition of the Z - transforms
Z[un] = U(z) =
∞
X
n=0
unz−n
Expanding the summation in the RHS
U(z) = u0z0
+ u1z−1
+ u2z−2
+ u3z−3
+ ...
U(z) − u0 =
u1
z
+
u2
z2
+
u3
z3
+ ...
Multiplying throughout by z
z[U(z) − u0] = u1 +
u2
z
+
u3
z2
+ ...
Applying the limit z → ∞ on both sides
lim
z→∞
z[U(z) − u0] = lim
z→∞
[u1 +
u2
z
+
u3
z2
+ ...]
lim
z→∞
z[U(z) − u0] = [u1 + (0) + (0) + ...]
u1 = lim
z→∞
z[U(z) − u0]
Dr. Vijaya kumar, MSRIT, Bengaluru - 560 054
Engineering Mathematics - IV - IM41 May 5, 2022 57 / 99
Continued Unit - II on Initial Value of the Z - Transforms ...
(ii) To prove this result
u2 = lim
z→
z2
[U(z) − u0 − u1
z
],
We know that, by definition of the Z - transforms
Z[un] = U(z) =
∞
X
n=0
unz−n
Expanding the summation in the RHS
U(z) = u0z0
+ u1z−1
+ u2z−2
+ u3z−3
+ u4z−4
+ ...
U(z) − u0 −
u1
z
=
u2
z2
+
u3
z3
+
u4
z4
+ ...
Multiplying throughout by z2
z2
[U(z) − u0 −
u1
z
] = u2 +
u3
z
+
u4
z2
+ ...
Applying the limit z → ∞ on both sides
lim
z→∞
z2
[U(z) − u0 −
u1
z
] = lim
z→∞
[u2 +
u3
z
+
u4
z2
+ ...]
lim
z→∞
z2
[U(z) − u0 −
u1
z
] = [u2 + (0) + (0) + ...]
Dr. Vijaya kumar, MSRIT, Bengaluru - 560 054
Engineering Mathematics - IV - IM41 May 5, 2022 58 / 99
Continued Unit - II on Initial Value of the Z - Transforms ...
u2 = lim
z→∞
z[U(z) − u0 −
u1
z
]
(iii) To prove this result
u3 = lim
z→
z3
[U(z) − u0 − u1
z
− u2
z2 ],
We know that, by definition of the Z - transforms
Z[un] = U(z) =
∞
X
n=0
unz−n
Expanding the summation in the RHS
U(z) = u0z0
+ u1z−1
+ u2z−2
+ u3z−3
+ u4z−4
+ ...
U(z) − u0 −
u1
z
−
u2
z2
=
u3
z3
+
u4
z4
+ ...
Multiplying throughout by z3
z3
[U(z) − u0 −
u1
z
−
u2
z2
] = u3 +
u4
z
+
u5
z2
+ ...
Applying the limit z → ∞ on both sides
lim
z→∞
z3
[U(z) − u0 −
u1
z
−
u2
z2
] = lim
z→∞
[u3 +
u4
z
+
u5
z2
+ ...]
Dr. Vijaya kumar, MSRIT, Bengaluru - 560 054
Engineering Mathematics - IV - IM41 May 5, 2022 59 / 99
Continued Unit - II on Initial Value and Final Value Theorem of the Z -
Transforms ...
lim
z→∞
z3
[U(z) − u0 −
u1
z
−
u2
z2
] = [u3 + (0) + (0) + ...]
u3 = lim
z→∞
z3
[U(z) − u0 −
u1
z
−
u2
z2
]
Final Value Theorem of the Z - Transforms:
Statement: If Z[un] = U(z), then prove that lim
n→∞
un = lim
z→1
(z − 1)U(z).
Dr. Vijaya kumar, MSRIT, Bengaluru - 560 054
Engineering Mathematics - IV - IM41 May 5, 2022 60 / 99
Continued Unit - II Problems on Initial Value of the Z - Transforms ...
Problem - 01: Find the values of the u0, u1, u2 and u3, given that
U(z) = 5z2
+3z+12
(z−1)4 .
Solution: By data U(z) = 5z2
+3z+12
(z−1)4
(i) To find the value of u0
We know that, by initial value of Z - transforms
u0 = lim
z→∞
U(z)
Substituting U(z) in the RHS
u0 = lim
z→∞
[
5z2
+ 3z + 12
(z − 1)4
] = lim
z→∞
z2
[
5 + 3
z
+ 12
z2
z4(1 − 1
z
)4
] = lim
z→∞
[
5 + 3
z
+ 12
z2
z2(1 − 1
z
)4
]
u0 = lim
z→∞
1
z2
[
5 + 3
z
+ 12
z2
(1 − 1
z
)4
] = (0)[
5 + (0) + (0)
(1 − 0)4
] = (0)(5)
u0 = 0
(ii) To find the value of u1
We know that, by initial value of Z - transforms
u1 = lim
z→∞
z[U(z) − u0]
Dr. Vijaya kumar, MSRIT, Bengaluru - 560 054
Engineering Mathematics - IV - IM41 May 5, 2022 61 / 99
Continued Unit - II Problems on Initial Value of the Z - Transforms ...
Substituting U(z) and u0 in the RHS
u1 = lim
z→∞
z[
5z2
+ 3z + 12
(z − 1)4
− 0] = lim
z→∞
z[
5z2
+ 3z + 12
(z − 1)4
] = lim
z→∞
z3
[
5 + 3
z
+ 12
z2
z4(1 − 1
z
)4
]
u1 = lim
z→∞
[
5 + 3
z
+ 12
z2
z(1 − 1
z
)4
] = lim
z→∞
1
z
[
5 + 3
z
+ 12
z2
(1 − 1
z
)4
] = (0)[
5 + (0) + (0)
(1 − 0)4
] = (0)(5)
u1 = 0
(iii) To find the value of u2
We know that, by initial value of Z - transforms
u2 = lim
z→∞
z2
[U(z) − u0 −
u1
z
]
Substituting U(z), u0 and u1 in the RHS
u2 = lim
z→∞
z2
[
5z2
+ 3z + 12
(z − 1)4
− 0 − 0] = lim
z→∞
z2
[
5z2
+ 3z + 12
(z − 1)4
] = lim
z→∞
z4
[
5 + 3
z
+ 12
z2
z4(1 − 1
z
)4
]
u2 = lim
z→∞
[
5 + 3
z
+ 12
z2
(1 − 1
z
)4
] = [
5 + (0) + (0)
(1 − 0)4
] =
5
1
u1 = 5
Dr. Vijaya kumar, MSRIT, Bengaluru - 560 054
Engineering Mathematics - IV - IM41 May 5, 2022 62 / 99
Continued Unit - II Problems on Initial Value of the Z - Transforms ...
(iv) To find the value of u3
We know that, by initial value of Z - transforms
u3 = lim
z→∞
z3
[U(z) − u0 −
u1
z
−
u2
z2
]
Substituting U(z), u0, u1 and u2 in the RHS
u3 = lim
z→∞
z3
[
5z2
+ 3z + 12
(z − 1)4
− 0 − 0 −
5
z2
] = lim
z→∞
z3
[
5z2
+ 3z + 12
(z − 1)4
−
5
z2
]
u3 = lim
z→∞
z3
[
5z4
+ 3z3
+ 12z2
− 5(z − 1)4
z2(z − 1)4
]
But (z − 1)4
= (z − 1)2
(z − 1)2
(z − 1)4
= (z2
− 2z + 1)(z2
− 2z + 1)
(z − 1)4
= (z4
− 2z3
+ z2
− 2z3
4z2
− 2z + z2
− 2z + 1)
(z − 1)4
= (z4
− 4z3
+ 6z2
− 4z + 1)
u3 = lim
z→∞
z3
[
5z4
+ 3z3
+ 12z2
− 5(z4
− 4z3
+ 6z2
− 4z + 1)
z2(z − 1)4
]
Dr. Vijaya kumar, MSRIT, Bengaluru - 560 054
Engineering Mathematics - IV - IM41 May 5, 2022 63 / 99
Continued Unit - II Problems on Initial Value of the Z - Transforms ...
u3 = lim
z→∞
z3
[
5z4
+ 3z3
+ 12z2
− 5z4
+ 20z3
− 30z2
+ 20z − 5
z2(z − 1)4
]
u3 = lim
z→∞
z3
[
23z3
− 18z2
+ 20z − 5
z2(z − 1)4
] = lim
z→∞
z6
[
23 − 18
z
+ 20
z2 − 5
z3
z6(1 − 1
z
)4
]
u3 = lim
z→∞
[
23 − 18
z
+ 20
z2 − 5
z3
(1 − 1
z
)4
] = [
23 − (0) + (0) − (0)
(1 − 0)4
] =
23
1
u3 = 23
Problem - 02: Find the values of the u0, u1, u2 and u3, when U(z) = 2z2
+4z+12
(z−1)4 .
Solution: By data U(z) = 2z2
+4z+12
(z−1)4
(i) To find the value of u0
We know that, by initial value of Z - transforms
u0 = lim
z→∞
U(z)
Substituting U(z) in the RHS
u0 = lim
z→∞
[
2z2
+ 4z + 12
(z − 1)4
] = lim
z→∞
z2
[
2 + 4
z
+ 12
z2
z4(1 − 1
z
)4
] = lim
z→∞
[
2 + 4
z
+ 12
z2
z2(1 − 1
z
)4
]
Dr. Vijaya kumar, MSRIT, Bengaluru - 560 054
Engineering Mathematics - IV - IM41 May 5, 2022 64 / 99
Continued Unit - II Problems on Initial Value of the Z - Transforms ...
u0 = lim
z→∞
1
z2
[
2 + 4
z
+ 12
z2
(1 − 1
z
)4
] = (0)[
2 + (0) + (0)
(1 − 0)4
] = (0)(2)
u0 = 0
(ii) To find the value of u1
We know that, by initial value of Z - transforms
u1 = lim
z→∞
z[U(z) − u0]
Substituting U(z) and u0 in the RHS
u1 = lim
z→∞
z[
2z2
+ 4z + 12
(z − 1)4
− 0] = lim
z→∞
z[
2z2
+ 4z + 12
(z − 1)4
] = lim
z→∞
z3
[
2 + 4
z
+ 12
z2
z4(1 − 1
z
)4
]
u1 = lim
z→∞
[
2 + 4
z
+ 12
z2
z(1 − 1
z
)4
] = lim
z→∞
1
z
[
2 + 4
z
+ 12
z2
(1 − 1
z
)4
] = (0)[
2 + (0) + (0)
(1 − 0)4
] = (0)(2)
u1 = 0
(iii) To find the value of u2
We know that, by initial value of Z - transforms
u2 = lim
z→∞
z2
[U(z) − u0 −
u1
z
]
Dr. Vijaya kumar, MSRIT, Bengaluru - 560 054
Engineering Mathematics - IV - IM41 May 5, 2022 65 / 99
Continued Unit - II Problems on Initial Value of the Z - Transforms ...
Substituting U(z), u0 and u1 in the RHS
u2 = lim
z→∞
z2
[
2z2
+ 4z + 12
(z − 1)4
− 0 − 0] = lim
z→∞
z2
[
2z2
+ 4z + 12
(z − 1)4
] = lim
z→∞
z4
[
2 + 4
z
+ 12
z2
z4(1 − 1
z
)4
]
u2 = lim
z→∞
[
2 + 4
z
+ 12
z2
(1 − 1
z
)4
] = [
2 + (0) + (0)
(1 − 0)4
] =
2
1
u1 = 2
(iv) To find the value of u3
We know that, by initial value of Z - transforms
u3 = lim
z→∞
z3
[U(z) − u0 −
u1
z
−
u2
z2
]
Substituting U(z), u0, u1 and u2 in the RHS
u3 = lim
z→∞
z3
[
2z2
+ 4z + 12
(z − 1)4
− 0 − 0 −
2
z2
] = lim
z→∞
z3
[
2z2
+ 4z + 12
(z − 1)4
−
2
z2
]
u3 = lim
z→∞
z3
[
2z4
+ 4z3
+ 12z2
− 2(z − 1)4
z2(z − 1)4
]
Dr. Vijaya kumar, MSRIT, Bengaluru - 560 054
Engineering Mathematics - IV - IM41 May 5, 2022 66 / 99
Continued Unit - II Problems on Initial Value of the Z - Transforms ...
But (z − 1)4
= (z − 1)2
(z − 1)2
(z − 1)4
= (z2
− 2z + 1)(z2
− 2z + 1)
(z − 1)4
= (z4
− 2z3
+ z2
− 2z3
4z2
− 2z + z2
− 2z + 1)
(z − 1)4
= (z4
− 4z3
+ 6z2
− 4z + 1)
u3 = lim
z→∞
z3
[
2z4
+ 4z3
+ 12z2
− 2(z4
− 4z3
+ 6z2
− 4z + 1)
z2(z − 1)4
]
u3 = lim
z→∞
z3
[
2z4
+ 4z3
+ 12z2
− 2z4
+ 8z3
− 12z2
+ 8z − 2
z2(z − 1)4
]
u3 = lim
z→∞
z3
[
12z3
+ 8z − 2
z2(z − 1)4
] = lim
z→∞
z6
[
12 + 8
z2 − 2
z3
z6(1 − 1
z
)4
]
u3 = lim
z→∞
[
12 + 8
z2 − 2
z3
(1 − 1
z
)4
] = [
12 − (0) + (0) − (0)
(1 − 0)4
] =
12
1
u3 = 12
Dr. Vijaya kumar, MSRIT, Bengaluru - 560 054
Engineering Mathematics - IV - IM41 May 5, 2022 67 / 99
Continued Unit - II Problems on Initial Value of the Z - Transforms ...
Problem - 03: If ZT (un) = z
z−1
− z
z2+1
, then find ZT (un+1).
Solution: By data ZT (un) = U(z) = z
z−1
− z
z2+1
We have, by shifting property
ZT (un+1) = z2
[U(z) − u0 −
u1
z
]
To find the value of u0, by initial value theorem
u0 = lim
z→∞
U(z)
u0 = lim
z→∞
[
z
z − 1
−
z
z2 + 1
] = lim
z→∞
[
z
z(1 − 1
z
)
−
z
z2(1 + 1
z2 )
]
u0 = lim
z→∞
[
1
(1 − 1
z
)
− (
1
z
)
1
(1 + 1
z2 )
] = [
1
(1 − 0)
− (0)
1
(1 + 0)
] = [
1
(1)
− (0)(
1
(1)
)]
u0 = 1
To find the value of u1, by initial value theorem
u1 = lim
z→∞
z[U(z) − u0]
u1 = lim
z→∞
z[
z
z − 1
−
z
z2 + 1
− 1] = lim
z→∞
z[
z(z2
+ 1) − z(z − 1) − (z − 1)(z2
+ 1)
(z − 1)(z2 + 1)
]
Dr. Vijaya kumar, MSRIT, Bengaluru - 560 054
Engineering Mathematics - IV - IM41 May 5, 2022 68 / 99
Continued Unit - II Problems on Initial Value of the Z - Transforms ...
u1 = lim
z→∞
z[
z3
+ z − z2
+ z − z3
− z + z2
+ 1
(z − 1)(z2 + 1)
] = lim
z→∞
z[
z + 1
(z − 1)(z2 + 1)
]
u1 = lim
z→∞
z2
[
(1 + 1
z
)
z3(1 − 1
z
)(1 + 1
z2 )
] = lim
z→∞
[
(1 + 1
z
)
z(1 − 1
z
)(1 + 1
z2 )
] = lim
z→∞
(
1
z
)[
(1 + 1
z
)
(1 − 1
z
)(1 + 1
z2 )
]
u0 = (0)[
(1 + 0)
(1 − 0)(1 + 0)
] = (0)(1) = 0
Substituting U(z), u0 and u1 in the above equation
ZT (un+1) = z2
[
z
z − 1
−
z
z2 + 1
− 1 −
(0)
z
]
ZT (un+1) = z2
[
z
z − 1
−
z
z2 + 1
− 1]
Dr. Vijaya kumar, MSRIT, Bengaluru - 560 054
Engineering Mathematics - IV - IM41 May 5, 2022 69 / 99
Continued Unit - II on Inverse Z - Transforms and Formulas ...
Definition of the Inverse Z - Transforms: The function U(z) is said to be
inverse Z - transforms of the function un. The inverse Z - transforms of the function
U(z) is denoted by Z−1
[U(z)] and is given by
un = Z−1
[U(z)]
where, Z−1
is the inverse Z - transforms operator.
Formulas of the Inverse Z - Transforms of standard functions:
1.
Z−1
[
z
z − 1
] = 1
2.
Z−1
[
z
z + 1
] = −1
3.
Z−1
[
z
z − a
] = an
4.
Z−1
[
z
z + a
] = (−a)n
5.
Z−1
[
1
z − a
] = an−1
Dr. Vijaya kumar, MSRIT, Bengaluru - 560 054
Engineering Mathematics - IV - IM41 May 5, 2022 70 / 99
Continued Unit - II on Formulas of the Inverse Z - Transforms ...
6.
Z−1
[
1
z + a
] = (−a)n−1
7.
Z−1
[
1
(z − a)2
] = (n − 1)an−2
8.
Z−1
[
1
(z + a)2
] = (n − 1)(−a)n−2
9.
Z−1
[
1
(z − a)3
] =
1
2
(n − 1)(n − 2)an−3
10.
Z−1
[
z2
(z − a)2
] = (n + 1)an
11.
Z−1
[
z3
(z − a)3
] =
1
2!
(n + 1)(n + 2)an
un
Dr. Vijaya kumar, MSRIT, Bengaluru - 560 054
Engineering Mathematics - IV - IM41 May 5, 2022 71 / 99
Continued Unit - II on Formulas of the Inverse Z - Transforms ...
12.
Z−1
[
z
(z − a)2
] = nan−1
13.
Z−1
[
az
(z − a)2
] = nan
14.
Z−1
[
az2
+ a2
z
(z − a)3
] = n2
an
15.
Z−1
[
z
z − 1
] = H(n)
16.
Z−1
[1] = δ(n)
Dr. Vijaya kumar, MSRIT, Bengaluru - 560 054
Engineering Mathematics - IV - IM41 May 5, 2022 72 / 99
Continued Unit - II Problems on Inverse Z - Transforms ...
Problem - 01 : Find the inverse Z - transforms of z
(z−1)(z+2)
.
Solution: by data z
(z−1)(z+2)
, Take
U(z) =
z
(z − 1)(z + 2)
U(z)
z
=
1
(z − 1)(z + 2)
(3)
Applying the partial fraction for the RHS expression
1
(z − 1)(z + 2)
=
A
(z − 1)
+
B
(z + 2)
1 = A(z + 2) + B(z − 1)
put, z = 1, we get 1 = A(3) + B(0), A = 1
3
put, z = −2, we get 1 = A(0) + B(−3), B = −1
3
. Substituting the value of A and B
in the above equation
1
(z − 1)(z + 2)
=
1
3(z − 1)
−
1
3(z + 2)
(4)
Dr. Vijaya kumar, MSRIT, Bengaluru - 560 054
Engineering Mathematics - IV - IM41 May 5, 2022 73 / 99
Continued Unit - II Problems on Inverse Z - Transforms ...
Using Eqn. (4) in Eqn. (3)
U(z)
z
=
1
3(z − 1)
−
1
3(z + 2)
Multiply throughout by z
U(z) =
1
3
(
z
z − 1
) −
1
3
(
z
z + 2
)
Applying inverse Z - transforms on both sides
Z−1
[U(z)] =
1
3
Z−1
[
z
z − 1
] −
1
3
Z−1
[
z
z + 2
]
Z−1
[
z
(z − 1)(z + 2)
] =
1
3
(1) −
1
3
(−2)n
Z−1
[
z
(z − 1)(z + 2)
] =
1
3
[1 − (−2)n
]
Dr. Vijaya kumar, MSRIT, Bengaluru - 560 054
Engineering Mathematics - IV - IM41 May 5, 2022 74 / 99
Continued Unit - II Problems on Inverse Z - Transforms ...
Problem - 02 : Find the inverse Z - transforms of 5z
(2−z)(3z−1)
.
Solution: by data 5z
(2−z)(3z−1)
, Take
U(z) =
5z
(2 − z)(3z − 1)
U(z)
z
=
5
(2 − z)(3z − 1)
(5)
Applying the partial fraction for the RHS expression
5
(2 − z)(3z − 1)
=
A
(2 − z)
+
B
(3z − 1)
5 = A(3z − 1) + B(2 − z)
put, z = 2, we get 5 = A(5) + B(0), A = 1
put, z = 1
3
, we get 5 = A(0) + B(5
3
), B = 3. Substituting the value of A and B in the
above equation
5
(2 − z)(3z − 1)
=
1
(2 − z)
+
3
(3z − 1)
(6)
Dr. Vijaya kumar, MSRIT, Bengaluru - 560 054
Engineering Mathematics - IV - IM41 May 5, 2022 75 / 99
Continued Unit - II Problems on Inverse Z - Transforms ...
Using Eqn. (6) in Eqn. (5)
U(z)
z
=
1
(2 − z)
+
3
(3z − 1)
Multiply throughout by z
U(z) =
z
(2 − z)
+ 3
z
(3z − 1)
Applying inverse Z - transforms on both sides
Z−1
[U(z)] = Z−1
[
z
−(z − 2)
] + 3Z−1
[
z
3(z − 1
3
)
]
Z−1
[
5z
(2 − z)(3z − 1)
] = (−1)(2)2
+
3
3
(
1
3
)n
Z−1
[
5z
(2 − z)(3z − 1)
] = −(2)2
+ (
1
3
)n
Dr. Vijaya kumar, MSRIT, Bengaluru - 560 054
Engineering Mathematics - IV - IM41 May 5, 2022 76 / 99
Continued Unit - II Problems on Inverse Z - Transforms ...
Problem - 03 : Find the inverse Z - transforms of 2z2
+3z
(z+2)(z−4)
.
Solution: by data 2z2
+3z
(z+2)(z−4)
, Take
U(z) =
2z2
+ 3z
(z + 2)(z − 4)
U(z)
z
=
2z + 3
(z + 2)(z − 4)
(7)
Applying the partial fraction for the RHS expression
2z + 3
(z + 2)(z − 4)
=
A
(z + 2)
+
B
(z − 4)
2z + 3 = A(z − 4) + B(z + 2)
put, z = −2, we get −1 = A(−6) + B(0), A = 1
6
put, z = 4, we get 11 = A(0) + B(6), B = 11
6
. Substituting the value of A and B in
the above equation
2z + 3
(z + 2)(z − 4)
=
1
6(z + 2)
+
11
6(z − 4)
(8)
Dr. Vijaya kumar, MSRIT, Bengaluru - 560 054
Engineering Mathematics - IV - IM41 May 5, 2022 77 / 99
Continued Unit - II Problems on Inverse Z - Transforms ...
Using Eqn. (8) in Eqn. (7)
U(z)
z
=
1
6(z + 2)
+
11
6(z − 4)
Multiply throughout by z
U(z) =
1
6
[
z
(z + 2)
] +
11
6
[
z
(z − 4)
]
Applying inverse Z - transforms on both sides
Z−1
[U(z)] =
1
6
Z−1
[
z
(z + 2)
] +
11
6
Z−1
[
z
(z − 4)
]
Z−1
[
2z2
+ 3z
(z + 2)(z − 4)
] =
1
6
(−2)n
+
11
6
(4)n
Z−1
[
2z2
+ 3z
(z + 2)(z − 4)
] =
1
6
[(−2)n
+ 11(4)n
]
Dr. Vijaya kumar, MSRIT, Bengaluru - 560 054
Engineering Mathematics - IV - IM41 May 5, 2022 78 / 99
Continued Unit - II Problems on Inverse Z - Transforms ...
Problem - 04 : Find the inverse Z - transforms of 4z2
−2z
z3−5z2+8z−4
.
Solution: by data 4z2
−2z
z3−5z2+8z−4
, Take
U(z) =
4z2
− 2z
z3 − 5z2 + 8z − 4
Factorize the denominator expression z3
− 5z2
+ 8z − 4 = (z − 1)(z − 2)2
U(z) =
4z2
− 2z
(z − 1)(z − 2)2
U(z)
z
=
4z − 2
(z − 1)(z − 2)2
(9)
Applying the partial fraction for the RHS expression
4z − 2
(z − 1)(z − 2)2
=
A
(z − 1)
+
B
(z − 2)
+
C
(z − 2)2
4z − 2 = A(z − 2)2
+ B(z − 1)(z − 2) + C(z − 1)
put, z = 1, we get 2 = A(1) + B(0) + C(0), A = 2,
put, z = 2, we get 6 = A(0) + B(0) + C(1), C = 6,
put, z = 0, we get −2 = A(4) + B(2) + C(−1), −2 = 8 + 2B − 6, B = 0. Substituting
the value of A, B and C in the above equation
Dr. Vijaya kumar, MSRIT, Bengaluru - 560 054
Engineering Mathematics - IV - IM41 May 5, 2022 79 / 99
Continued Unit - II Problems on Inverse Z - Transforms ...
4z − 2
(z − 1)(z − 2)2
=
2
(z − 1)
+
0
(z − 2)
+
6
(z − 2)2
4z − 2
(z − 1)(z − 2)2
=
2
(z − 1)
+
6
(z − 2)2
(10)
Using Eqn. (10) in Eqn. (9)
U(z)
z
=
2
(z − 1)
+
6
(z − 2)2
Multiply throughout by z
U(z) = 2
z
(z − 1)
+ 6
z
(z − 2)2
Applying inverse Z - transforms on both sides
Z−1
[U(z)] = 2Z−1
[
z
(z − 1)
] + 6Z−1
[
z
(z − 2)2
]
Z−1
[
4z2
− 2z
z3 − 5z2 + 8z − 4
] = 2(1) + 6n(2)n−1
Dr. Vijaya kumar, MSRIT, Bengaluru - 560 054
Engineering Mathematics - IV - IM41 May 5, 2022 80 / 99
Continued Unit - II Problems on Inverse Z - Transforms ...
Z−1
[
4z2
− 2z
z3 − 5z2 + 8z − 4
] = 2 + 6n(2)n−1
Problem - 05 : Find the inverse Z - transforms of 8z−z3
(4−z)3 .
Solution: by data 8z−z3
(4−z)3 , Take
U(z) =
8z − z3
(−1)3(z − 4)3
=
8z − z3
−(z − 4)3
U(z) =
z3
− 8z
(z − 4)3
We can observe in the denominator expression has repeated linear factors and we
have Z−1
[ z
(z−4)
] = 4n
, Z−1
[ 4z
(z−4)2 ] = n4n
and Z−1
[4z2
+16z
(z−4)3 ] = n2
4n
We have, apply
the partial fractions for the RHS expression for the above equation
z3
− 8z
(z − 4)3
= A
z
z − 4
+ B
4z
(z − 4)2
+ C
4z2
+ 16z
(z − 4)3
z3
− 8z = Az(z − 4)2
+ 4Bz(z − 4) + 4Cz(z + 4)
Dr. Vijaya kumar, MSRIT, Bengaluru - 560 054
Engineering Mathematics - IV - IM41 May 5, 2022 81 / 99
Continued Unit - II Problems on Inverse Z - Transforms ...
Canceling z on both sides
z2
− 8 = A(z − 4)2
+ 4B(z − 4) + 4C(z + 4)
put, z = 4, we get −8 = A(0) + 4B(0) + 4C(8), C = 1
4
.
Equating the coefficient of z2
on both sides, we get, A = 1.
Again by equating the coefficient of z on both sides, we have, 0 = −8A + 4B + 4C,
we get, B = 7
4
. Substituting the value of A, B and C in the above equation
z3
− 8z
(z − 4)3
= (1)
z
z − 4
+ (
7
4
)
4z
(z − 4)2
+ (
1
4
)
4z2
+ 16z
(z − 4)3
Applying the inverse Z - transforms on both sides
Z−1
[
z3
− 8z
(z − 4)3
] = Z−1
[
z
z − 4
] +
7
4
Z−1
[
4z
(z − 4)2
] +
1
4
Z−1
[
4z2
+ 16z
(z − 4)3
]
Using the above results, we get
Z−1
[
z3
− 8z
(z − 4)3
] = 4n
+
7
4
n4n
+
1
4
n2
4n
Dr. Vijaya kumar, MSRIT, Bengaluru - 560 054
Engineering Mathematics - IV - IM41 May 5, 2022 82 / 99
Continued Unit - II Problems on Inverse Z - Transforms ...
Problem - 06 : Find the inverse Z - transforms of z3
−20z
(z−2)3(z−4)
.
Solution: by data z3
−20z
(z−2)3(z−4)
, Take
U(z) =
z3
− 20z
(z − 2)3(z − 4)
We can observe in the denominator expression has repeated linear factors in first
factor and we have Z−1
[ z
(z−2)
] = 2n
, Z−1
[ 2z
(z−2)2 ] = n2n
, Z−1
[2z2
+4z
(z−2)3 ] = n2
4n
and
Z−1
[ z
(z−4)
] = 4n
We have, apply the partial fractions for the RHS expression for the
above equation
z3
− 20z
(z − 2)3(z − 4)
= A
z
z − 2
+ B
2z
(z − 2)2
+ C
2z2
+ 4z
(z − 2)3
+ D
z
z − 4
z3
− 20z = Az(z − 2)2
(z − 4) + 2Bz(z − 2)(z − 4) + C(2z2
+ 4z)(z − 4) + Dz(z − 2)3
Dr. Vijaya kumar, MSRIT, Bengaluru - 560 054
Engineering Mathematics - IV - IM41 May 5, 2022 83 / 99
Continued Unit - II Problems on Inverse Z - Transforms ...
Canceling z on both sides
z2
− 20 = A(z − 2)2
(z − 4) + 2B(z − 2)(z − 4) + C(2z + 4)(z − 4) + D(z − 2)3
put, z = 2, we get −16 = A(0) + 2B(0) + C(−16) + D(0), C = 1.
put, z = 4, we get −4 = A(0) + 2B(0) + C(0) + D(8), D = −1
2
.
Equating the coefficient of z3
on both sides, we get, A + D = 0, A = 1
2
.
put, z = 0, we get −20 = A(4)(−4) + 2B(8) + C(−16) + D(−8), B = 0. Substituting
the value of A, B, C and D in the above equation
z3
− 20z
(z − 2)3(z − 4)
= (
1
2
)
z
z − 2
+ (0)
2z
(z − 2)2
+ (1)
2z2
+ 4z
(z − 2)3
+ (
−1
2
)
z
z − 4
Applying the inverse Z - transforms on both sides
Z−1
[
z3
− 20z
(z − 2)3(z − 4)
] =
1
2
Z−1
[
z
z − 2
] + Z−1
[
2z2
+ 4z
(z − 2)3
] −
1
2
Z−1
[
z
z − 4
]
Using the above results, we get
Z−1
[
z3
− 20z
(z − 2)3(z − 4)
] =
1
2
2n
+ n2
2n
−
1
2
4n
= 2n−1
+ n2
2n
− 22n−1
Dr. Vijaya kumar, MSRIT, Bengaluru - 560 054
Engineering Mathematics - IV - IM41 May 5, 2022 84 / 99
Continued Unit - II on Application of the Z - Transforms to Solve the
Difference Equations ...
Introduction:
Definition of Difference equation: An equation which involving the set of values
of the dependent variable is known as a difference equation.
Let us consider the second order forward difference of y0 is equal to zero, that is
∆2
y0 = 0
∆(∆y0) = 0
we have, ∆y0 = y1 − y0
∆(y1 − y0) = 0
∆y1 − ∆y0 = 0
(y2 − y1) − (y1 − y0) = 0
y2 − 2y1 + y0 = 0
This kind of equation is called as a difference equation Examples:
1. yn+2 + 5yn+1 + 6yn = 3n
.
2. un+2 − 3yn+1 + 2yn = n2
.
Dr. Vijaya kumar, MSRIT, Bengaluru - 560 054
Engineering Mathematics - IV - IM41 May 5, 2022 85 / 99
Continued Unit - II on Application of the Z - Transforms to Solve the
Difference Equations ...
Working procedure to solve the difference equations:
Step - 01: Applying the Z - transforms on both sides.
Step - 02: Substituting the shifting property of Z - transforms, that is
Z[un+1] = z[U(z) − u0]
Z[un+2] = z2
[U(z) − u0 − u1
z
]
Z[un+3] = z3
[U(z) − u0 − u1
z
− u2
z2 ]
Step - 03: Substituting the given initial conditions and simplify U(z) in terms of z.
Step - 04: Applying the inverse Z - transforms on both sides, we get the solution of
the given difference equation.
Dr. Vijaya kumar, MSRIT, Bengaluru - 560 054
Engineering Mathematics - IV - IM41 May 5, 2022 86 / 99
Continued Unit - II Problems on Application of the Z - Transforms to
Solve the Difference Equations ...
Problem - 01: Solve the difference equation un+2 − 5un+1 + 6un = 2, given that
u0 = 3, u1 = 7, by using Z - transforms.
Solution: By data un+2 − 5un+1 + 6un = 2, and u0 = 3, u1 = 7
Applying the Z - transforms on both sides
Z[un+2] − 5Z[un+1] + 6Z[un] = Z[2]
Substituting the shifting property of the Z - transforms
z2
[U(z) − u0 −
u1
z
] − 5(z[U(z) − u0]) + 6(U(z)) = (
2z
z − 1
)
Substituting the given initial conditions u0 = 3 and u1 = 7
z2
[U(z) − (3) −
7
z
] − 5(z[U(z) − (3)]) + 6(U(z)) = (
2z
z − 1
)
Simplify
z2
U(z) − 3z2
− 7z − 5zU(z) + 15z + 6U(z) =
2z
z − 1
z2
U(z) − 5zU(z) + 6U(z) =
2z
z − 1
+ 3z2
+ 7z − 15z
Dr. Vijaya kumar, MSRIT, Bengaluru - 560 054
Engineering Mathematics - IV - IM41 May 5, 2022 87 / 99
Continued Unit - II Problems on Application of the Z - Transforms to
Solve the Difference Equations ...
(z2
− 5z + 6)U(z) =
2z
z − 1
+ 3z2
− 8z
(z − 2)(z − 3)U(z) =
2z + 3z2
(z − 1) − 8z(z − 1)
z − 1
U(z) =
3z3
− 11z2
+ 10z
(z − 1)(z − 2)(z − 3)
U(z)
z
=
3z2
− 11z + 10
(z − 1)(z − 2)(z − 3)
(11)
Now, applying the partial fractions for the RHS expression
3z2
− 11z + 10
(z − 1)(z − 2)(z − 3)
=
A
z − 1
+
B
z − 2
+
C
z − 3
3z2
− 11z + 10 = A(z − 2)(z − 3) + B(z − 1)(z − 3) + C(z − 1)(z − 2)
put, z = 1, we get, 2 = A(−1)(−2) + B(0) + C(0), A = 1.
put, z = 2, we get, 0 = A(0) + B(1)(−1) + C(0), B = 0.
put, z = 3, we get, 4 = A(0) + B(0) + C(2)(1), C = 2. Substituting the values of the
A, B and C in the above equation
Dr. Vijaya kumar, MSRIT, Bengaluru - 560 054
Engineering Mathematics - IV - IM41 May 5, 2022 88 / 99
Continued Unit - II Problems on Application of the Z - Transforms to
Solve the Difference Equations ...
3z2
− 11z + 10
(z − 1)(z − 2)(z − 3)
=
1
z − 1
+
0
z − 2
+
2
z − 3
3z2
− 11z + 10
(z − 1)(z − 2)(z − 3)
=
1
z − 1
+
2
z − 3
Using Eqn. (12) in Eqn. (11)
U(z)
z
=
1
z − 1
+
2
z − 3
(12)
Multiply throughout by z
U(z) =
z
z − 1
+ 2
z
z − 3
Applying inverse Z - transforms on both sides
Z−1
[U(z)] = Z−1
[
z
z − 1
] + 2Z−1
[
z
z − 3
]
but, Z−1
[U(z)] = un
un = (1) + 2(3n
) = 1 + 23n
This is required solution for the given difference equation.
Dr. Vijaya kumar, MSRIT, Bengaluru - 560 054
Engineering Mathematics - IV - IM41 May 5, 2022 89 / 99
Continued Unit - II Problems on Application of the Z - Transforms to
Solve the Difference Equations ...
Problem - 02: Solve yn+2 + 2yn+1 + yn = n, given that y0 = 0, y1 = 0, by using Z -
transforms.
Solution: Given yn+2 + 2yn+1 + yn = n and y0 = 0, y1 = 0
Applying the Z - transforms on both sides
Z[yn+2] + 2Z[yn+1] + Z[yn] = Z[n]
Substituting the shifting property of the Z - transforms
z2
[Y (z) − y0 −
y1
z
] + 2(z[Y (z) − y0]) + (Y (z)) = (
z
(z − 1)2
)
Substituting the given initial conditions y0 = 0 and y1 = 0
z2
[Y (z) − (0) −
0
z
] + 2(z[Y (z) − (0)]) + (Y (z)) =
z
(z − 1)2
Simplify
z2
Y (z) − (0) − (0) + 2zY (z) − (0) + Y (z) =
z
(z − 1)2
(z2
+ 2z + 1)Y (z) =
z
(z − 1)2
Dr. Vijaya kumar, MSRIT, Bengaluru - 560 054
Engineering Mathematics - IV - IM41 May 5, 2022 90 / 99
Continued Unit - II Problems on Application of the Z - Transforms to
Solve the Difference Equations ...
(z + 1)2
Y (z) =
z
(z − 1)2
Y (z) =
z
(z − 1)2(z + 1)2
Y (z)
z
=
1
(z − 1)2(z + 1)2
(13)
Applying the partial fractions for the RHS expression
1
(z − 1)2(z + 1)2
=
A
(z − 1)
+
B
(z − 1)2
+
C
(z + 1)
+
D
(z + 1)2
1 = A(z − 1)(z + 1)2
+ B(z + 1)2
+ C(z + 1)(z − 1)2
+ D(z − 1)2
put, z = 1, we get, 1 = A(0) + B(4) + C(0) + D(0), B = 1
4
.
put, z = −1, we get, 1 = A(0) + B(0) + C(0) + D(4), D = 1
4
.
Rewrite the above equation
1 = A(z3
+ z2
− z − 1) + B(z2
+ 2z + 1) + C(z3
− z2
− z + 1) + D(z2
− 2z + 1)
Equating the coefficient of z3
on both sides A + C = 0 Equating the coefficient of z2
on both sides A + B − C + D = 0, A − C = −1
2
,
Dr. Vijaya kumar, MSRIT, Bengaluru - 560 054
Engineering Mathematics - IV - IM41 May 5, 2022 91 / 99
Continued Unit - II Problems on Application of the Z - Transforms to
Solve the Difference Equations ...
solving these two equations, we get A = −1
4
and C = 1
4
. Substituting the values of A,
B, C and D in the above equation.
1
(z − 1)2(z + 1)2
=
−1
4(z − 1)
+
1
4(z − 1)2
+
1
4(z + 1)
+
1
4(z + 1)2
(14)
Using Eqn. (14) in (13)
Y (z)
z
=
−1
4(z − 1)
+
1
4(z − 1)2
+
1
4(z + 1)
+
1
4(z + 1)2
Multiplying throughout by z
Y (z) = −
1
4
z
z − 1
+
1
4
z
(z − 1)2
+
1
4
z
z + 1
+
1
4
z
(z + 1)2
Applying the inverse Z - transforms on both sides
Z−1
[Y (z)] = −
1
4
[Z−1
[
z
z − 1
] + Z−1
[
z
(z − 1)2
] + Z−1
[
z
z + 1
] + Z−1
[
z
(z + 1)2
]]
Dr. Vijaya kumar, MSRIT, Bengaluru - 560 054
Engineering Mathematics - IV - IM41 May 5, 2022 92 / 99
Continued Unit - II Problems on Application of the Z - Transforms to
Solve the Difference Equations ...
but, Z−1
[Y (z)] = yn
yn =
1
4
[−(1) + n(1)n−1
+ (−1)n
+ n(−1)n−1
] =
1
4
[−1 + n + (−1)n
+ n(−1)n−1
]
This is required solution for the given difference equation.
Problem - 03: Solve yn+2 − 4yn+1 + 3yn = H(n), given that y0 = 0, y1 = 0, by
using Z - transforms.
Solution: Given yn+2 − 4yn+1 + 3yn = H(n) and y0 = 0, y1 = 0
Applying the Z - transforms on both sides
Z[yn+2] − 4Z[yn+1] + 3Z[yn] = Z[H(n)]
Substituting the shifting property of the Z - transforms
z2
[Y (z) − y0 −
y1
z
] − 4(z[Y (z) − y0]) + 3(Y (z)) = (
z
(z − 1)
)
Substituting the given initial conditions y0 = 0 and y1 = 0
z2
[Y (z) − (0) −
0
z
] − 4(z[Y (z) − (0)]) + 3(Y (z)) =
z
(z − 1)
Dr. Vijaya kumar, MSRIT, Bengaluru - 560 054
Engineering Mathematics - IV - IM41 May 5, 2022 93 / 99
Continued Unit - II Problems on Application of the Z - Transforms to
Solve the Difference Equations ...
Simplify
z2
Y (z) − (0) − (0) − 4zY (z) − (0) + 3Y (z) =
z
(z − 1)
(z2
− 4z + 3)Y (z) =
z
(z − 1)
(z − 1)(z − 3)Y (z) =
z
(z − 1)
Y (z) =
z
(z − 1)2(z − 3)
Y (z)
z
=
1
(z − 1)2(z − 3)
(15)
Applying the partial fraction for RHS expression in the above equation
1
(z − 1)2(z − 3)
=
A
(z − 1)
+
B
(z − 1)2
+
C
(z − 3)
1 = A(z − 1)(z − 3) + B(z − 3) + C(z − 1)2
Dr. Vijaya kumar, MSRIT, Bengaluru - 560 054
Engineering Mathematics - IV - IM41 May 5, 2022 94 / 99
Continued Unit - II Problems on Application of the Z - Transforms to
Solve the Difference Equations ...
put, z = 1, we get, 1 = A(0) + B(−2) + C(0), B = −1
2
.
put, z = 3, we get, 1 = A(0) + B(0) + C(4), C = 1
4
.
put, z = 0, we get, 1 = A(3) + B(−3) + C(1), A = −1
4
. Substituting the values of the
A, B and C in the above equation
1
(z − 1)2(z − 3)
=
−1
4(z − 1)
+
−1
2(z − 1)2
+
1
4(z − 3)
(16)
Using Eqn. (16) IN Eqn.. (15)
Y (z)
z
=
−1
4(z − 1)
+
−1
2(z − 1)2
+
1
4(z − 3)
Multiplying throughout by z
Y (z) = −
1
4
z
(z − 1)
−
1
2
z
2(z − 1)2
+
1
4
z
(z − 3)
Applying the inverse Z - transforms on both sides
Z−1
[Y (z)] = −
1
4
Z−1
[
z
(z − 1)
] −
1
2
Z−1
[
z
(z − 1)2
] +
1
4
Z−1
[
z
(z − 3)
]
Dr. Vijaya kumar, MSRIT, Bengaluru - 560 054
Engineering Mathematics - IV - IM41 May 5, 2022 95 / 99
Continued Unit - II Problems on Application of the Z - Transforms to
Solve the Difference Equations ...
But, Z−1
[Y (z)] = yn
yn = −
1
4
(1) −
1
2
(n) +
1
4
(3)n
=
−1
4
−
n
2
+
(3)n
4
This is required solution for the given difference equation.
Problem - 04: Solve the difference equation un+2 − 5un+1 + 6un = Hn, given that
u0 = 0, u1 = 1, where Hn is a unit step sequence, by using Z - transforms.
Solution: Given un+2 − 5un+1 + 6un = Hn and u0 = 0, u1 = 1
Applying the Z - transforms on both sides
Z[un+2] − 5Z[un+1] + 6Z[un] = Z[Hn]
Substituting the shifting property of the Z - transforms
z2
[U(z) − u0 −
u1
z
] − 5(z[U(z) − u0]) + 6(U(z)) = (
z
(z − 1)
)
Substituting the given initial conditions u0 = 0 and u1 = 1
z2
[U(z) − (0) −
1
z
] − 5(z[U(z) − (0)]) + 6(U(z)) =
z
(z − 1)
Dr. Vijaya kumar, MSRIT, Bengaluru - 560 054
Engineering Mathematics - IV - IM41 May 5, 2022 96 / 99
Continued Unit - II Problems on Application of the Z - Transforms to
Solve the Difference Equations ...
z2
U(z) − 0 − z − 5zU(z) − 0 + 6U(z) =
z
(z − 1)
(z2
− 5z + 6)U(z) =
z
(z − 1)
+ z =
z + z2
− z
(z − 1)
(z − 2)(z − 3)U(z) =
z2
(z − 1)
U(z) =
z2
(z − 1)(z − 2)(z − 3)
U(z)
z
=
z
(z − 1)(z − 2)(z − 3)
(17)
Applying the partial fractions for the RHS expression
z
(z − 1)(z − 2)(z − 3)
=
A
(z − 1)
+
B
(z − 2)
+
C
(z − 3)
z = A(z − 2)(z − 3) + B(z − 1)(z − 3) + C(z − 1)(z − 2)
Dr. Vijaya kumar, MSRIT, Bengaluru - 560 054
Engineering Mathematics - IV - IM41 May 5, 2022 97 / 99
Continued Unit - II Problems on Application of the Z - Transforms to
Solve the Difference Equations ...
put, z = 1, we get, 1 = A(2) + B(0) + C(0), A = 1
2
.
put, z = 2, we get, 2 = A(0) + B(−2) + C(0), B = −1
2
.
put, z = 3, we get, 3 = A(0) + B(0) + C(2), C = 3
2
. Substituting the values of the A,
B and C in the above equation
z
(z − 1)(z − 2)(z − 3)
=
1
2(z − 1)
+
−1
2(z − 2)
+
3
2(z − 3)
Using Eqn. (18) in Eqn. (17)
U(z)
z
=
1
2(z − 1)
+
−1
2(z − 2)
+
3
2(z − 3)
Multiplying throughout by z
U(z) =
1
2
z
z − 1
−
1
2
z
z − 2
+
3
2
z
z − 3
Dr. Vijaya kumar, MSRIT, Bengaluru - 560 054
Engineering Mathematics - IV - IM41 May 5, 2022 98 / 99
Continued Unit - II Problems on Application of the Z - Transforms to
Solve the Difference Equations ...
Applying the inverse Z - transforms on both sides
Z−1
[U(z)] =
1
2
Z−1
[
z
z − 1
] −
1
2
Z−1
[
z
z − 2
] +
3
2
Z−1
[
z
z − 3
]
But, Z−1
[U(z)] = un
un =
1
2
(1) −
1
2
(2)n
+
3
2
(3)n
=
1
2
− (2)n−1
+
(3)n+1
2
This is required solution for the given difference equation.
Dr. Vijaya kumar, MSRIT, Bengaluru - 560 054
Engineering Mathematics - IV - IM41 May 5, 2022 99 / 99

More Related Content

Similar to maths ZT.pdf

Complex Numbers Mathmatics N4
Complex  Numbers  Mathmatics N4Complex  Numbers  Mathmatics N4
Complex Numbers Mathmatics N4Jude Jay
 
Modeling Transformations
Modeling TransformationsModeling Transformations
Modeling TransformationsTarun Gehlot
 
Higher order derivatives for N -body simulations
Higher order derivatives for N -body simulationsHigher order derivatives for N -body simulations
Higher order derivatives for N -body simulationsKeigo Nitadori
 
3130005 cvpde gtu_study_material_e-notes_all_18072019070728_am
3130005 cvpde gtu_study_material_e-notes_all_18072019070728_am3130005 cvpde gtu_study_material_e-notes_all_18072019070728_am
3130005 cvpde gtu_study_material_e-notes_all_18072019070728_amdataniyaarunkumar
 
Theme 4 Notes Complex Numbers (1).pdf
Theme 4 Notes Complex Numbers (1).pdfTheme 4 Notes Complex Numbers (1).pdf
Theme 4 Notes Complex Numbers (1).pdfThapeloTsepo1
 
Complex numbers 1
Complex  numbers 1Complex  numbers 1
Complex numbers 1Jude Jay
 
Important formulas for JEE Main 2020 - Subject-wise List
Important formulas for JEE Main 2020 - Subject-wise ListImportant formulas for JEE Main 2020 - Subject-wise List
Important formulas for JEE Main 2020 - Subject-wise ListKanhaMalik
 
Z TRANSFORM PROPERTIES AND INVERSE Z TRANSFORM
Z TRANSFORM PROPERTIES AND INVERSE Z TRANSFORMZ TRANSFORM PROPERTIES AND INVERSE Z TRANSFORM
Z TRANSFORM PROPERTIES AND INVERSE Z TRANSFORMTowfeeq Umar
 
3 d transformation
3 d transformation3 d transformation
3 d transformationMani Kanth
 
Fast and efficient exact synthesis of single qubit unitaries generated by cli...
Fast and efficient exact synthesis of single qubit unitaries generated by cli...Fast and efficient exact synthesis of single qubit unitaries generated by cli...
Fast and efficient exact synthesis of single qubit unitaries generated by cli...JamesMa54
 
Cubic Spline Interpolation
Cubic Spline InterpolationCubic Spline Interpolation
Cubic Spline InterpolationVARUN KUMAR
 

Similar to maths ZT.pdf (20)

Momentum sudut total
Momentum sudut totalMomentum sudut total
Momentum sudut total
 
Complex Numbers Mathmatics N4
Complex  Numbers  Mathmatics N4Complex  Numbers  Mathmatics N4
Complex Numbers Mathmatics N4
 
Chap 4 complex numbers focus exam ace
Chap 4 complex numbers focus exam aceChap 4 complex numbers focus exam ace
Chap 4 complex numbers focus exam ace
 
Modeling Transformations
Modeling TransformationsModeling Transformations
Modeling Transformations
 
Maths05
Maths05Maths05
Maths05
 
Higher order derivatives for N -body simulations
Higher order derivatives for N -body simulationsHigher order derivatives for N -body simulations
Higher order derivatives for N -body simulations
 
3130005 cvpde gtu_study_material_e-notes_all_18072019070728_am
3130005 cvpde gtu_study_material_e-notes_all_18072019070728_am3130005 cvpde gtu_study_material_e-notes_all_18072019070728_am
3130005 cvpde gtu_study_material_e-notes_all_18072019070728_am
 
Diff-Eqs
Diff-EqsDiff-Eqs
Diff-Eqs
 
Theme 4 Notes Complex Numbers (1).pdf
Theme 4 Notes Complex Numbers (1).pdfTheme 4 Notes Complex Numbers (1).pdf
Theme 4 Notes Complex Numbers (1).pdf
 
Complex Numbers 1.pdf
Complex Numbers 1.pdfComplex Numbers 1.pdf
Complex Numbers 1.pdf
 
Complex numbers 1
Complex  numbers 1Complex  numbers 1
Complex numbers 1
 
Chapter-3.pptx
Chapter-3.pptxChapter-3.pptx
Chapter-3.pptx
 
21 2 ztransform
21 2 ztransform21 2 ztransform
21 2 ztransform
 
Important formulas for JEE Main 2020 - Subject-wise List
Important formulas for JEE Main 2020 - Subject-wise ListImportant formulas for JEE Main 2020 - Subject-wise List
Important formulas for JEE Main 2020 - Subject-wise List
 
Z TRANSFORM PROPERTIES AND INVERSE Z TRANSFORM
Z TRANSFORM PROPERTIES AND INVERSE Z TRANSFORMZ TRANSFORM PROPERTIES AND INVERSE Z TRANSFORM
Z TRANSFORM PROPERTIES AND INVERSE Z TRANSFORM
 
3 d transformation
3 d transformation3 d transformation
3 d transformation
 
lec_3.pdf
lec_3.pdflec_3.pdf
lec_3.pdf
 
Z transforms
Z transformsZ transforms
Z transforms
 
Fast and efficient exact synthesis of single qubit unitaries generated by cli...
Fast and efficient exact synthesis of single qubit unitaries generated by cli...Fast and efficient exact synthesis of single qubit unitaries generated by cli...
Fast and efficient exact synthesis of single qubit unitaries generated by cli...
 
Cubic Spline Interpolation
Cubic Spline InterpolationCubic Spline Interpolation
Cubic Spline Interpolation
 

Recently uploaded

代办国外大学文凭《原版美国UCLA文凭证书》加州大学洛杉矶分校毕业证制作成绩单修改
代办国外大学文凭《原版美国UCLA文凭证书》加州大学洛杉矶分校毕业证制作成绩单修改代办国外大学文凭《原版美国UCLA文凭证书》加州大学洛杉矶分校毕业证制作成绩单修改
代办国外大学文凭《原版美国UCLA文凭证书》加州大学洛杉矶分校毕业证制作成绩单修改atducpo
 
Invezz.com - Grow your wealth with trading signals
Invezz.com - Grow your wealth with trading signalsInvezz.com - Grow your wealth with trading signals
Invezz.com - Grow your wealth with trading signalsInvezz1
 
100-Concepts-of-AI by Anupama Kate .pptx
100-Concepts-of-AI by Anupama Kate .pptx100-Concepts-of-AI by Anupama Kate .pptx
100-Concepts-of-AI by Anupama Kate .pptxAnupama Kate
 
Mature dropshipping via API with DroFx.pptx
Mature dropshipping via API with DroFx.pptxMature dropshipping via API with DroFx.pptx
Mature dropshipping via API with DroFx.pptxolyaivanovalion
 
CebaBaby dropshipping via API with DroFX.pptx
CebaBaby dropshipping via API with DroFX.pptxCebaBaby dropshipping via API with DroFX.pptx
CebaBaby dropshipping via API with DroFX.pptxolyaivanovalion
 
Generative AI on Enterprise Cloud with NiFi and Milvus
Generative AI on Enterprise Cloud with NiFi and MilvusGenerative AI on Enterprise Cloud with NiFi and Milvus
Generative AI on Enterprise Cloud with NiFi and MilvusTimothy Spann
 
Unveiling Insights: The Role of a Data Analyst
Unveiling Insights: The Role of a Data AnalystUnveiling Insights: The Role of a Data Analyst
Unveiling Insights: The Role of a Data AnalystSamantha Rae Coolbeth
 
VIP Call Girls Service Miyapur Hyderabad Call +91-8250192130
VIP Call Girls Service Miyapur Hyderabad Call +91-8250192130VIP Call Girls Service Miyapur Hyderabad Call +91-8250192130
VIP Call Girls Service Miyapur Hyderabad Call +91-8250192130Suhani Kapoor
 
Industrialised data - the key to AI success.pdf
Industrialised data - the key to AI success.pdfIndustrialised data - the key to AI success.pdf
Industrialised data - the key to AI success.pdfLars Albertsson
 
BabyOno dropshipping via API with DroFx.pptx
BabyOno dropshipping via API with DroFx.pptxBabyOno dropshipping via API with DroFx.pptx
BabyOno dropshipping via API with DroFx.pptxolyaivanovalion
 
Edukaciniai dropshipping via API with DroFx
Edukaciniai dropshipping via API with DroFxEdukaciniai dropshipping via API with DroFx
Edukaciniai dropshipping via API with DroFxolyaivanovalion
 
BPAC WITH UFSBI GENERAL PRESENTATION 18_05_2017-1.pptx
BPAC WITH UFSBI GENERAL PRESENTATION 18_05_2017-1.pptxBPAC WITH UFSBI GENERAL PRESENTATION 18_05_2017-1.pptx
BPAC WITH UFSBI GENERAL PRESENTATION 18_05_2017-1.pptxMohammedJunaid861692
 
FESE Capital Markets Fact Sheet 2024 Q1.pdf
FESE Capital Markets Fact Sheet 2024 Q1.pdfFESE Capital Markets Fact Sheet 2024 Q1.pdf
FESE Capital Markets Fact Sheet 2024 Q1.pdfMarinCaroMartnezBerg
 
定制英国白金汉大学毕业证(UCB毕业证书) 成绩单原版一比一
定制英国白金汉大学毕业证(UCB毕业证书)																			成绩单原版一比一定制英国白金汉大学毕业证(UCB毕业证书)																			成绩单原版一比一
定制英国白金汉大学毕业证(UCB毕业证书) 成绩单原版一比一ffjhghh
 
Brighton SEO | April 2024 | Data Storytelling
Brighton SEO | April 2024 | Data StorytellingBrighton SEO | April 2024 | Data Storytelling
Brighton SEO | April 2024 | Data StorytellingNeil Barnes
 
Carero dropshipping via API with DroFx.pptx
Carero dropshipping via API with DroFx.pptxCarero dropshipping via API with DroFx.pptx
Carero dropshipping via API with DroFx.pptxolyaivanovalion
 
Smarteg dropshipping via API with DroFx.pptx
Smarteg dropshipping via API with DroFx.pptxSmarteg dropshipping via API with DroFx.pptx
Smarteg dropshipping via API with DroFx.pptxolyaivanovalion
 
Week-01-2.ppt BBB human Computer interaction
Week-01-2.ppt BBB human Computer interactionWeek-01-2.ppt BBB human Computer interaction
Week-01-2.ppt BBB human Computer interactionfulawalesam
 
꧁❤ Greater Noida Call Girls Delhi ❤꧂ 9711199171 ☎️ Hard And Sexy Vip Call
꧁❤ Greater Noida Call Girls Delhi ❤꧂ 9711199171 ☎️ Hard And Sexy Vip Call꧁❤ Greater Noida Call Girls Delhi ❤꧂ 9711199171 ☎️ Hard And Sexy Vip Call
꧁❤ Greater Noida Call Girls Delhi ❤꧂ 9711199171 ☎️ Hard And Sexy Vip Callshivangimorya083
 

Recently uploaded (20)

꧁❤ Aerocity Call Girls Service Aerocity Delhi ❤꧂ 9999965857 ☎️ Hard And Sexy ...
꧁❤ Aerocity Call Girls Service Aerocity Delhi ❤꧂ 9999965857 ☎️ Hard And Sexy ...꧁❤ Aerocity Call Girls Service Aerocity Delhi ❤꧂ 9999965857 ☎️ Hard And Sexy ...
꧁❤ Aerocity Call Girls Service Aerocity Delhi ❤꧂ 9999965857 ☎️ Hard And Sexy ...
 
代办国外大学文凭《原版美国UCLA文凭证书》加州大学洛杉矶分校毕业证制作成绩单修改
代办国外大学文凭《原版美国UCLA文凭证书》加州大学洛杉矶分校毕业证制作成绩单修改代办国外大学文凭《原版美国UCLA文凭证书》加州大学洛杉矶分校毕业证制作成绩单修改
代办国外大学文凭《原版美国UCLA文凭证书》加州大学洛杉矶分校毕业证制作成绩单修改
 
Invezz.com - Grow your wealth with trading signals
Invezz.com - Grow your wealth with trading signalsInvezz.com - Grow your wealth with trading signals
Invezz.com - Grow your wealth with trading signals
 
100-Concepts-of-AI by Anupama Kate .pptx
100-Concepts-of-AI by Anupama Kate .pptx100-Concepts-of-AI by Anupama Kate .pptx
100-Concepts-of-AI by Anupama Kate .pptx
 
Mature dropshipping via API with DroFx.pptx
Mature dropshipping via API with DroFx.pptxMature dropshipping via API with DroFx.pptx
Mature dropshipping via API with DroFx.pptx
 
CebaBaby dropshipping via API with DroFX.pptx
CebaBaby dropshipping via API with DroFX.pptxCebaBaby dropshipping via API with DroFX.pptx
CebaBaby dropshipping via API with DroFX.pptx
 
Generative AI on Enterprise Cloud with NiFi and Milvus
Generative AI on Enterprise Cloud with NiFi and MilvusGenerative AI on Enterprise Cloud with NiFi and Milvus
Generative AI on Enterprise Cloud with NiFi and Milvus
 
Unveiling Insights: The Role of a Data Analyst
Unveiling Insights: The Role of a Data AnalystUnveiling Insights: The Role of a Data Analyst
Unveiling Insights: The Role of a Data Analyst
 
VIP Call Girls Service Miyapur Hyderabad Call +91-8250192130
VIP Call Girls Service Miyapur Hyderabad Call +91-8250192130VIP Call Girls Service Miyapur Hyderabad Call +91-8250192130
VIP Call Girls Service Miyapur Hyderabad Call +91-8250192130
 
Industrialised data - the key to AI success.pdf
Industrialised data - the key to AI success.pdfIndustrialised data - the key to AI success.pdf
Industrialised data - the key to AI success.pdf
 
BabyOno dropshipping via API with DroFx.pptx
BabyOno dropshipping via API with DroFx.pptxBabyOno dropshipping via API with DroFx.pptx
BabyOno dropshipping via API with DroFx.pptx
 
Edukaciniai dropshipping via API with DroFx
Edukaciniai dropshipping via API with DroFxEdukaciniai dropshipping via API with DroFx
Edukaciniai dropshipping via API with DroFx
 
BPAC WITH UFSBI GENERAL PRESENTATION 18_05_2017-1.pptx
BPAC WITH UFSBI GENERAL PRESENTATION 18_05_2017-1.pptxBPAC WITH UFSBI GENERAL PRESENTATION 18_05_2017-1.pptx
BPAC WITH UFSBI GENERAL PRESENTATION 18_05_2017-1.pptx
 
FESE Capital Markets Fact Sheet 2024 Q1.pdf
FESE Capital Markets Fact Sheet 2024 Q1.pdfFESE Capital Markets Fact Sheet 2024 Q1.pdf
FESE Capital Markets Fact Sheet 2024 Q1.pdf
 
定制英国白金汉大学毕业证(UCB毕业证书) 成绩单原版一比一
定制英国白金汉大学毕业证(UCB毕业证书)																			成绩单原版一比一定制英国白金汉大学毕业证(UCB毕业证书)																			成绩单原版一比一
定制英国白金汉大学毕业证(UCB毕业证书) 成绩单原版一比一
 
Brighton SEO | April 2024 | Data Storytelling
Brighton SEO | April 2024 | Data StorytellingBrighton SEO | April 2024 | Data Storytelling
Brighton SEO | April 2024 | Data Storytelling
 
Carero dropshipping via API with DroFx.pptx
Carero dropshipping via API with DroFx.pptxCarero dropshipping via API with DroFx.pptx
Carero dropshipping via API with DroFx.pptx
 
Smarteg dropshipping via API with DroFx.pptx
Smarteg dropshipping via API with DroFx.pptxSmarteg dropshipping via API with DroFx.pptx
Smarteg dropshipping via API with DroFx.pptx
 
Week-01-2.ppt BBB human Computer interaction
Week-01-2.ppt BBB human Computer interactionWeek-01-2.ppt BBB human Computer interaction
Week-01-2.ppt BBB human Computer interaction
 
꧁❤ Greater Noida Call Girls Delhi ❤꧂ 9711199171 ☎️ Hard And Sexy Vip Call
꧁❤ Greater Noida Call Girls Delhi ❤꧂ 9711199171 ☎️ Hard And Sexy Vip Call꧁❤ Greater Noida Call Girls Delhi ❤꧂ 9711199171 ☎️ Hard And Sexy Vip Call
꧁❤ Greater Noida Call Girls Delhi ❤꧂ 9711199171 ☎️ Hard And Sexy Vip Call
 

maths ZT.pdf

  • 1. Unit - II: Z - TRANSFORMS ... Transformation OR Transform: A transform is a mathematical devices which transforms one function to the another function. Most of the science and engineering problems are ending with the mathematical equations like equations, ordinary differential equations, partial differential equations and difference equations. In the present topic, we discuss the basic definition of Z - transform, properties of Z - transforms, Z - transforms of the standard functions, problems on Z - transforms, initial value and final value theorem of the Z - transforms and problems, inverse Z - transforms, definition of the inverse Z - transforms, formulas of the inverse Z -transforms of standard functions and problems. Application of Z -transforms to solve difference equations. Dr. Vijaya kumar, MSRIT, Bengaluru - 560 054 Engineering Mathematics - IV - IM41 May 5, 2022 1 / 99
  • 2. Continued Unit - II on Z - Transforms ... Definition of the Z - transforms: The Z - transforms of the discrete function un, defined for all n ≥ 0 and un = 0, for n < 0, is denoted by ZT [un] OR Z[un] OR U(z) OR Ū(z) and is defined as, ZT [un] = Z[un] = ∞ X n=0 unz−n = U(z) = Ū(z) Where Z OR ZT is the Z - transforms operator and z is the Z - transforms parameter. Definition of the Inverse Z - Transforms: The function U(z) is said to be inverse Z - transforms of the function un. The inverse Z - transforms of the function U(z) is denoted by Z−1 [U(z)] and is given by un = Z−1 [U(z)] where, Z−1 is the inverse Z - transforms operator. Properties of the Z - transforms: 1. Linearity Property: If c1, c2, ..., cp all are constants then Z[c1u1(n) + c2u2(n) + ... + cpup(n)] = c1Z[u1(n)] + c2Z[f2(x)] + ... + cpZ[up(x)] Dr. Vijaya kumar, MSRIT, Bengaluru - 560 054 Engineering Mathematics - IV - IM41 May 5, 2022 2 / 99
  • 3. Continued Unit - II on Properties of the Z - Transforms ... Poof: We have, by definition of Z - transforms Z[c1u1(n) + c2u2(n) + ... + cpup(n)] = ∞ X n=0 (c1u1(n) + c2u2(n) + ... + cpup(n))z−n Z[c1u1(n)+c2u2(n)+...+cpup(n)] = c1 ∞ X n=0 u1(n)z−n +c2 ∞ X n=0 u2(n)z−n +...+cp ∞ X n=0 up(n)z− Z[c1u1(n) + c2u2(n) + ... + cpup(n)] = c1Z[u1(n)] + c2Z[u2(n)] + ... + cpZ[up(n)] 2. Damping Property OR Damping rule: If Z[un = U(z)], then prove that (i) Z[an un] = U(z a ), (ii) Z[a−n un] = U(az). Poof: (i) We have, by definition of Z - transforms Z[un] = ∞ X n=0 unz−n = U(z) Therefore, Z[an un] = ∞ X n=0 (an un)z−n Dr. Vijaya kumar, MSRIT, Bengaluru - 560 054 Engineering Mathematics - IV - IM41 May 5, 2022 3 / 99
  • 4. Continued Unit - II on Properties of the Z - Transforms ... Z[an un] = ∞ X n=0 un z−n a−n Z[an un] = ∞ X n=0 un( z a )−n Comparing this with the definition here instead of z is z a Z[an un] = U( z a ) (ii) We have, by definition of Z - transforms Z[un] = ∞ X n=0 unz−n = U(z) Therefore, Z[a−n un] = ∞ X n=0 (a−n un)z−n Z[a−n un] = ∞ X n=0 un(az)−n Dr. Vijaya kumar, MSRIT, Bengaluru - 560 054 Engineering Mathematics - IV - IM41 May 5, 2022 4 / 99
  • 5. Continued Unit - II on Properties of the Z - Transforms ... Comparing this with the definition here instead of z is az Z[a−n un] = U(az) 3. Left Shifting Property: If Z[un = U(z)], then prove that (i) Z[un+1] = z[U(z) − u0], (ii) Z[un+2] = z2 [U(z) − u0 − u1 z ], (iii)Z[un+3] = z3 [U(z) − u0 − u1 z − u2 z2 ] and (iv) Z[un+k] = zk [U(z) − Pk−1 r=0 urzr ]. Poof: (i) We have, by definition of Z - transforms Z[un] = U(z) = ∞ X n=0 unz−n ∴ Z[un+1] = ∞ X n=0 un+1z−n Multiply and dividing by z in the RHS Z[un+1] = (z)( 1 z ) ∞ X n=0 un+1z−n Dr. Vijaya kumar, MSRIT, Bengaluru - 560 054 Engineering Mathematics - IV - IM41 May 5, 2022 5 / 99
  • 6. Continued Unit - II on Properties of the Z - Transforms ... Z[un+1] = z ∞ X n=0 un+1 1 z z−n Z[un+1] = z ∞ X n=0 un+1z−1 z−n Z[un+1] = z ∞ X n=0 un+1z−n−1 Z[un+1] = z ∞ X n=0 un+1z−(n+1) Expanding the summation in the RHS Z[un+1] = z[u1z−1 + u2z−2 + u3z−3 + ...] Now, adding and subtracting the missing term u0 in the RHS Z[un+1] = z[u0 + u1z−1 + u2z−2 + u3z−3 + ... − u0] Z[un+1] = z[(u0 + u1z−1 + u2z−2 + u3z−3 + ...) − u0] Dr. Vijaya kumar, MSRIT, Bengaluru - 560 054 Engineering Mathematics - IV - IM41 May 5, 2022 6 / 99
  • 7. Continued Unit - II on Properties of the Z - Transforms ... Z[un+1] = z[ ∞ X n=0 unz−n − u0] Z[un+1] = z[U(z) − u0] (ii) We have, by definition of Z - transforms Z[un] = U(z) = ∞ X n=0 unz−n ∴ Z[un+2] = ∞ X n=0 un+2z−n Multiply and dividing by z2 in the RHS Z[un+2] = (z2 )( 1 z2 ) ∞ X n=0 un+2z−n Z[un+2] = z2 ∞ X n=0 un+2 1 z2 z−n Dr. Vijaya kumar, MSRIT, Bengaluru - 560 054 Engineering Mathematics - IV - IM41 May 5, 2022 7 / 99
  • 8. Continued Unit - II on Properties of the Z - Transforms ... Z[un+2] = z2 ∞ X n=0 un+2z−2 z−n Z[un+2] = z2 ∞ X n=0 un+2z−n−2 Z[un+2] = z2 ∞ X n=0 un+2z−(n+2) Expanding the summation in the RHS Z[un+2] = z2 [u2z−2 + u3z−3 + u4z−4 + ...] Now, adding and subtracting the missing terms u0 and u1z−1 in the RHS Z[un+2] = z2 [u0 + u1z−1 + u2z−2 + u3z−3 + ... − u0 − u1z−1 ] Z[un+2] = z2 [(u0 + u1z−1 + u2z−2 + u3z−3 + ...) − u0 − u1z−1 ] Z[un+2] = z2 [ ∞ X n=0 unz−n − u0 − u1 z ] Z[un+2] = z2 [U(z) − u0 − u1 z ] Dr. Vijaya kumar, MSRIT, Bengaluru - 560 054 Engineering Mathematics - IV - IM41 May 5, 2022 8 / 99
  • 9. Continued Unit - II on Properties of the Z - Transforms ... (iii) We have, by definition of Z - transforms Z[un] = U(z) = ∞ X n=0 unz−n ∴ Z[un+3] = ∞ X n=0 un+3z−n Multiply and dividing by z3 in the RHS Z[un+3] = (z3 )( 1 z3 ) ∞ X n=0 un+3z−n Z[un+3] = z3 ∞ X n=0 un+3 1 z3 z−n Z[un+3] = z3 ∞ X n=0 un+3z−3 z−n Z[un+3] = z3 ∞ X n=0 un+3z−n−3 Dr. Vijaya kumar, MSRIT, Bengaluru - 560 054 Engineering Mathematics - IV - IM41 May 5, 2022 9 / 99
  • 10. Continued Unit - II on Properties of the Z - Transforms ... Z[un+3] = z3 ∞ X n=0 un+3z−(n+3) Expanding the summation in the RHS Z[un+3] = z3 [u3z−3 + u4z−4 + u5z−5 + ...] Now, adding and subtracting the missing terms u0, u1z−1 and u2z−2 in the RHS Z[un+3] = z3 [u0 + u1z−1 + u2z−2 + u3z−3 + ... − u0 − u1z−1 − u2z−2 ] Z[un+3] = z3 [(u0 + u1z−1 + u2z−2 + u3z−3 + ...) − u0 − u1z−1 − u2z−2 ] Z[un+3] = z3 [ ∞ X n=0 unz−n − u0 − u1 z − u2 z2 ] Z[un+3] = z3 [U(z) − u0 − u1 z − u2 z2 ] Similarly, we can prove (iv) Z[un+k] = zk [U(z) − Pk−1 r=0 urzr ]. Dr. Vijaya kumar, MSRIT, Bengaluru - 560 054 Engineering Mathematics - IV - IM41 May 5, 2022 10 / 99
  • 11. Continued Unit - II on Properties of the Z - Transforms ... 4. Right Shifting Property: If Z[un = U(z)], then prove that Z[un−k] = z−k U(z), k > 0. Property - 05: If Z[un = U(z)], then prove that Z[np ] = −z d dz [Z(np−1 )], where p is any positive integer. Poof: We have, by definition of Z - transforms Z[np ] = ∞ X n=0 np z−n Multiply and dividing by n and z in the RHS Z[np ] = (z) ∞ X n=0 (n)( 1 n )np ( 1 z )z−n Z[np ] = z ∞ X n=0 nn−1 np z−1 z−n Z[np ] = z ∞ X n=0 nnp−1 z−n−1 (1) Dr. Vijaya kumar, MSRIT, Bengaluru - 560 054 Engineering Mathematics - IV - IM41 May 5, 2022 11 / 99
  • 12. Continued Unit - II on Properties of the Z - Transforms ... Again, by definition of Z - transforms Z[np−1 ] = ∞ X n=0 np−1 z−n Differentiating w.r.t. z on both sides d dz Z[np−1 ] = d dz [ ∞ X n=0 np−1 z−n ] d dz [Z(np−1 )] = ∞ X n=0 d dz [np−1 z−n ] d dz [Z(np−1 )] = ∞ X n=0 np−1 (−n)z−n−1 d dz [Z(np−1 )] = − ∞ X n=0 nnp−1 z−n−1 ∞ X n=0 nnp−1 z−n−1 = − d dz [Z(np−1 )] (2) Dr. Vijaya kumar, MSRIT, Bengaluru - 560 054 Engineering Mathematics - IV - IM41 May 5, 2022 12 / 99
  • 13. Continued Unit - II on Properties of the Z - Transforms ... Substituting equation (2) in equation (1), we get Z[np ] = z[− d dz [Z(np−1 )]] Z[np ] = −z d dz [Z(np−1 )] Hence, the property is proved. Dr. Vijaya kumar, MSRIT, Bengaluru - 560 054 Engineering Mathematics - IV - IM41 May 5, 2022 13 / 99
  • 14. Continued Unit - II on Z - Transforms of the Standard Functions ... 1. Find the Z - transforms of 1. Solution: We know that, by definition of the Z - transforms Z[un] = ∞ X n=0 unz−n = U(z) but, here un = 1 Z[1] = ∞ X n=0 (1)z−n Z[1] = ∞ X n=0 1 zn Z[1] = ∞ X n=0 1 z !n We have, by Binomial expansion (1 − x)−1 = 1 + x + x2 + x3 + ... = ∞ X n=0 xn Dr. Vijaya kumar, MSRIT, Bengaluru - 560 054 Engineering Mathematics - IV - IM41 May 5, 2022 14 / 99
  • 15. Continued Unit - II on Z - Transforms of the Standard Functions ... here, x = 1 z in the above equation Z[1] = 1 − 1 z !−1 Z[1] = z − 1 z !−1 Z[1] = z z − 1 Replace 1 by −1 , we get Z[−1] = z z + 1 2. Find the Z - transforms of an . Solution: We know that, by definition of the Z - transforms Z[un] = ∞ X n=0 unz−n = U(z) but, here un = an Z[an ] = ∞ X n=0 (an )z−n n ∞ X an Dr. Vijaya kumar, MSRIT, Bengaluru - 560 054 Engineering Mathematics - IV - IM41 May 5, 2022 15 / 99
  • 16. Continued Unit - II on Z - Transforms of the Standard Functions ... Z[an ] = ∞ X n=0 ( a z )n We have, by Binomial expansion (1 − x)−1 = 1 + x + x2 + x3 + ... = ∞ X n=0 xn here, x = a z in the above equation Z[an ] = 1 − a z !−1 Z[an ] = z − a z !−1 Z[an ] = z z − a Replace a by −a , we get Z[(−a)n ] = z z + a Dr. Vijaya kumar, MSRIT, Bengaluru - 560 054 Engineering Mathematics - IV - IM41 May 5, 2022 16 / 99
  • 17. Continued Unit - II on Z - Transforms of the Standard Functions ... Similarly, the Z - transforms (a) Z[enθ ] = Z[(eθ )n ] = z z − eθ (b) Z[e−nθ ] = Z[(e−θ )n ] = z z − e−θ (c) Z[einθ ] = Z[(eiθ )n ] = z z − eiθ (d) Z[e−inθ ] = Z[(e−iθ )n ] = z z − e−iθ (e) Z - transforms of any constant k Z[k] = kz z − 1 Dr. Vijaya kumar, MSRIT, Bengaluru - 560 054 Engineering Mathematics - IV - IM41 May 5, 2022 17 / 99
  • 18. Continued Unit - II on Z - Transforms of the Standard Functions ... We know that, I. einθ = cos(nθ) + i sin(nθ) II. e−inθ = cos(nθ) − i sin(nθ) III. cos(nθ) = einθ + e−inθ 2 IV. einθ + e−inθ = 2 cos(nθ) V. sin(nθ) = einθ − e−inθ 2i VI. einθ − e−inθ = 2i sin(nθ) VII. sinh(nθ) = enθ − e−nθ 2 VIII. cosh(nθ) = enθ + e−nθ 2 Dr. Vijaya kumar, MSRIT, Bengaluru - 560 054 Engineering Mathematics - IV - IM41 May 5, 2022 18 / 99
  • 19. Continued Unit - II on Z - Transforms of the Standard Functions ... 3. Obtain the Z - transforms of sinh(nθ) and cosh(nθ). Solution: (i) To obtain the Z - transforms of sinh(nθ) We know that, Z[enθ ] = z z − eθ and Z[e−nθ ] = z z − e−θ Also, we have sinh(nθ) = enθ − e−nθ 2 Applying the Z - transforms on the both sides Z[sinh(nθ)] = Z[ enθ − e−nθ 2 ] Z[sinh(nθ)] = 1 2 " Z[enθ ] − Z[e−nθ ] # Substituting Z[enθ ] = z z−eθ and Z[e−nθ ] = z z−e−θ Dr. Vijaya kumar, MSRIT, Bengaluru - 560 054 Engineering Mathematics - IV - IM41 May 5, 2022 19 / 99
  • 20. Continued Unit - II on Z - Transforms of the Standard Functions ... Z[sinh(nθ)] = 1 2 " z z − eθ − z z − e−θ # Z[sinh(nθ)] = 1 2 " z(z − e−θ ) − z(z − eθ ) (z − eθ)(z − e−θ) # Z[sinh(nθ)] = 1 2 [ z2 − ze−θ − z2 + zeθ z2 − ze−θ − zeθ + eθe−θ ] Z[sinh(nθ)] = 1 2 " z(eθ − e−θ ) z2 − z(eθ + e−θ) + e0 # Z[sinh(nθ)] = 1 2 " z(2 sinh(θ)) z2 − z(2 cosh(θ)) + 1 # Z[sinh(nθ)] = 1 1 " z(sinh(θ)) z2 − 2z cosh(θ) + 1 # Z[sinh(nθ)] = z sinh(θ) z2 − 2z cosh(θ) + 1 Dr. Vijaya kumar, MSRIT, Bengaluru - 560 054 Engineering Mathematics - IV - IM41 May 5, 2022 20 / 99
  • 21. Continued Unit - II on Z - Transforms of the Standard Functions ... (ii) To obtain the Z - transforms of cosh(nθ) We know that, Z[enθ ] = z z − eθ and Z[e−nθ ] = z z − e−θ Also, we have cosh(nθ) = enθ + e−nθ 2 Applying the Z - transforms on the both sides Z[cosh(nθ)] = Z[ enθ + e−nθ 2 ] Z[cosh(nθ)] = 1 2 " Z[enθ ] + Z[e−nθ ] # Substituting Z[enθ ] = z z−eθ and Z[e−nθ ] = z z−e−θ Dr. Vijaya kumar, MSRIT, Bengaluru - 560 054 Engineering Mathematics - IV - IM41 May 5, 2022 21 / 99
  • 22. Continued Unit - II on Z - Transforms of the Standard Functions ... Z[cosh(nθ)] = 1 2 " z z − eθ + z z − e−θ # Z[cosh(nθ)] = 1 2 " z(z − e−θ ) + z(z − eθ ) (z − eθ)(z − e−θ) # Z[cosh(nθ)] = 1 2 " z2 − ze−θ + z2 − zeθ z2 − ze−θ − zeθ + eθe−θ # Z[cosh θ)] = 1 2 " 2z2 − z(eθ + e−θ ) z2 − z(eθ + e−θ) + e0 # Z[cosh(nθ)] = 1 2 " 2z2 − z(2 cosh(θ)) z2 − z(2 cosh(θ)) + 1 # Z[cosh(nθ)] = 1 1 " z2 − z(cosh(θ)) z2 − 2z cosh(θ) + 1 # Z[cosh(nθ)] = z2 − z cosh(θ) z2 − 2z cosh(θ) + 1 Dr. Vijaya kumar, MSRIT, Bengaluru - 560 054 Engineering Mathematics - IV - IM41 May 5, 2022 22 / 99
  • 23. Continued Unit - II on Z - Transforms of the Standard Functions ... 4. Obtain the Z - transforms of sin(nθ) and cos(nθ). Solution: We know that, einθ = cos(nθ) + i sin(nθ) Applying Z - transforms on both sides Z[einθ ] = Z[cos(nθ) + i sin(nθ)] Z[cos(nθ)] + iZ[sin(nθ)] = Z[einθ ] Substituting Z[einθ ] = z z−eiθ in the RHS Z[cos(nθ)] + iZ[sin(nθ)] = z z − eiθ Z[cos(nθ)] + iZ[sin(nθ)] = z z − (cos(θ) + i sin(θ)) Z[cos(nθ)] + iZ[sin(nθ)] = z (z − cos(θ)) − i sin(θ) Now, rationalize RHS Dr. Vijaya kumar, MSRIT, Bengaluru - 560 054 Engineering Mathematics - IV - IM41 May 5, 2022 23 / 99
  • 24. Continued Unit - II on Z - Transforms of the Standard Functions ... Z[cos(nθ)] + iZ[sin(nθ)] = z((z − cos(θ)) + i sin(θ)) ((z − cos(θ)) − i sin(θ))((z − cos(θ)) + i sin(θ)) Z[cos(nθ)] + iZ[sin(nθ)] = (z2 − z cos(θ)) + iz sin(θ) (z − cos(θ))2 − (i sin(θ))2 Z[cos(nθ)] + iZ[sin(nθ)] = (z2 − z cos(θ)) + iz sin(θ) z2 − 2z cos(θ) + cos2(θ) − i2 sin2 (θ) Z[cos(nθ)] + iZ[sin(nθ)] = (z2 − z cos(θ)) + iz sin(θ) z2 − 2z cos(θ) + cos2(θ) + sin2 (θ) Z[cos(nθ)] + iZ[sin(nθ)] = (z2 − z cos(θ)) + iz sin(θ) z2 − 2z cos(θ) + 1 Separate the real and imaginary parts in the RHS Z[cos(nθ)] + iZ[sin(nθ)] = z2 − z cos(θ) z2 − 2z cos(θ) + 1 + i z sin(θ) z2 − 2z cos(θ) + 1 Comparing the real and imaginary parts on both sides, we get Dr. Vijaya kumar, MSRIT, Bengaluru - 560 054 Engineering Mathematics - IV - IM41 May 5, 2022 24 / 99
  • 25. Continued Unit - II on Z - Transforms of the Standard Functions ... Z[sin(nθ)] = z sin(θ) z2 − 2z cos(θ) + 1 and Z[cos(nθ)] = z2 − z cos(θ) z2 − 2z cos(θ) + 1 5. Find the Z - transforms of n. Solution: We know that, by property Z[np ] = −z d dz [Z(np−1 )] Substitute p = 1 Z[n1 ] = −z d dz [Z(n1−1 )] Z[n] = −z d dz [Z(n0 )] Z[n] = −z d dz [Z(1)] substituting Z[1] = z z−1 in the RHS Dr. Vijaya kumar, MSRIT, Bengaluru - 560 054 Engineering Mathematics - IV - IM41 May 5, 2022 25 / 99
  • 26. Continued Unit - II on Z - Transforms of the Standard Functions ... Z[n] = −z d dz [ z z − 1 ] Differentiate using the quotient rule Z[n] = −z[ (z − 1)(1) − (z)(1) (z − 1)2 ] Z[n] = −z[ z − 1 − z (z − 1)2 ] Z[n] = −z[ −1 (z − 1)2 ] Z[n] = z (z − 1)2 6. Find the Z - transforms of n2 . Solution: We know that, by property Z[np ] = −z d dz [Z(np−1 )] Substitute p = 2 Z[n2 ] = −z d dz [Z(n2−1 )] Dr. Vijaya kumar, MSRIT, Bengaluru - 560 054 Engineering Mathematics - IV - IM41 May 5, 2022 26 / 99
  • 27. Continued Unit - II on Z - Transforms of the Standard Functions ... Z[n2 ] = −z d dz [Z(n1 )] Z[n2 ] = −z d dz [Z(n)] substituting Z[n] = z (z−1)2 in the RHS Z[n2 ] = −z d dz [ z (z − 1)2 ] Differentiate using the quotient rule Z[n2 ] = −z[ (z − 1)2 (1) − (z)2(z − 1)(1) (z − 1)4 ] Z[n2 ] = −z(z − 1)[ z − 1 − 2z (z − 1)4 ] Z[n2 ] = −z[ −z − 1 (z − 1)3 ] Z[n2 ] = z2 + z (z − 1)3 Dr. Vijaya kumar, MSRIT, Bengaluru - 560 054 Engineering Mathematics - IV - IM41 May 5, 2022 27 / 99
  • 28. Continued Unit - II on Z - Transforms of the Standard Functions ... 7. Find the Z - transforms of n3 . Solution: We know that, by property Z[np ] = −z d dz [Z(np−1 )] Substitute p = 3 Z[n3 ] = −z d dz [Z(n3−1 )] Z[n3 ] = −z d dz [Z(n2 )] substituting Z[n2 ] = z2 +z (z−1)3 in the RHS Z[n3 ] = −z d dz [fracz2 + z(z − 1)3 ] Differentiate using the quotient rule Z[n3 ] = −z[ (z − 1)3 (2z + 1) − (z2 + z)3(z − 1)2 (1) (z − 1)6 ] Z[n3 ] = −z(z − 1)2 [ (z − 1)(2z + 1) − 3(z2 + z) (z − 1)6 ] Dr. Vijaya kumar, MSRIT, Bengaluru - 560 054 Engineering Mathematics - IV - IM41 May 5, 2022 28 / 99
  • 29. Continued Unit - II on Z - Transforms of the Standard Functions ... Z[n3 ] = −z[ 2z2 + z − 2z − 1 − 3z2 − 3z (z − 1)4 ] Z[n3 ] = −z[ −z2 − 4z − 1 (z − 1)4 ] Z[n3 ] = z3 + 4z2 + z (z − 1)4 8. Find the Z - transforms of n4 . Solution: We know that, by property Z[np ] = −z d dz [Z(np−1 )] Substitute p = 4 Z[n4 ] = −z d dz [Z(n4−1 )] Z[n4 ] = −z d dz [Z(n3 )] Dr. Vijaya kumar, MSRIT, Bengaluru - 560 054 Engineering Mathematics - IV - IM41 May 5, 2022 29 / 99
  • 30. Continued Unit - II on Z - Transforms of the Standard Functions ... substituting Z[n3 ] = z3 +4z2 +z (z−1)4 in the RHS Z[n4 ] = −z d dz [ z3 + 4z2 + z (z − 1)4 ] Differentiate using the quotient rule Z[n4 ] = −z[ (z − 1)4 (3z2 + 8z + 1) − (z3 + 4z2 + z)4(z − 1)3 (1) (z − 1)8 ] Z[n4 ] = −z(z − 1)3 [ (z − 1)(3z2 + 8z + 1) − 4(z3 + 4z2 + z) (z − 1)8 ] Z[n4 ] = −z[ (z − 1)(3z2 + 8z + 1) − 4z3 − 16z2 − 4z (z − 1)5 ] Z[n4 ] = −z[ 3z3 + 8z2 + z − 3z2 − 8z − 1 − 4z3 − 16z2 − 4z (z − 1)5 ] Z[n4 ] = −z[ −z3 − 11z2 − 11z − 1 (z − 1)5 ] Z[n4 ] = z4 + 11z3 + 11z2 + z (z − 1)5 Dr. Vijaya kumar, MSRIT, Bengaluru - 560 054 Engineering Mathematics - IV - IM41 May 5, 2022 30 / 99
  • 31. Continued Unit - II on Z - Transforms of the Standard Functions ... 9. Find the Z - transforms of 1 n , for n ≥ 1. Solution: We know that, by definition of the Z - transforms Z " 1 n # = ∞ X n=1 1 n ! z−n Expanding the summation in the RHS Z " 1 n # = 1 1 ! z−1 + 1 2 ! z−2 + 1 3 ! z−3 + ... Z " 1 n # = 1 z + 1 2z2 + 1 3z3 + ... Z " 1 n # = (1 z ) 1 + (1 z )2 2 + (1 z )3 3 + ... Z[ 1 n ] = −[− 1 z ! 1 − 1 z !2 2 − 1 z !3 3 − ...] We know that, by logarithmic series log(1 − x) = −x − x2 − x3 − ... Dr. Vijaya kumar, MSRIT, Bengaluru - 560 054 Engineering Mathematics - IV - IM41 May 5, 2022 31 / 99
  • 32. Continued Unit - II on Z - Transforms of the Standard Functions ... Here, x = 1 z Z " 1 n # = − " log 1 − 1 z !# Z " 1 n # = − log z − 1 z ! Z " 1 n # = log z − 1 z !−1 Z " 1 n # = log z z − 1 ! 10. Find the Z - transforms of 1 n+1 , for n ≥ 0. Solution: We know that, by definition of the Z - transforms Z " 1 n + 1 # = ∞ X n=0 1 n + 1 ! z−n Expanding the summation in the RHS Z " 1 n + 1 # = 1 1 ! z0 + 1 2 ! z−1 + 1 3 ! z−2 + 1 4 ! z−3 + ... Dr. Vijaya kumar, MSRIT, Bengaluru - 560 054 Engineering Mathematics - IV - IM41 May 5, 2022 32 / 99
  • 33. Continued Unit - II on Z - Transforms of the Standard Functions ... Multiply and dividing by −z in the RHS Z " 1 n + 1 # = (−z)[− (1 z ) 1 − (1 z )2 2 − (1 z )3 3 − ...] We know that, by logarithmic series log(1 − x) = −x − x2 2 − x3 3 − ... Z " 1 n + 1 # = (−z)[log(1 − 1 z )] Z " 1 n + 1 # = −z log z − 1 z ! Z " 1 n + 1 # = z log z − 1 z !−1 Z " 1 n + 1 # = z log z z − 1 ! Dr. Vijaya kumar, MSRIT, Bengaluru - 560 054 Engineering Mathematics - IV - IM41 May 5, 2022 33 / 99
  • 34. Continued Unit - II on Z - Transforms of the Standard Functions ... 11. Define unit step sequence for discrete and hence find its Z - transforms. Solution: Definition of the unit step sequence for discrete: The unit step sequence for discrete is denoted by u(n) or H(n) and is defined as u(n) = H(n) = ( 1, for n ≥ 0 0, for n < 0 To find the Z - transforms of the unit step sequence for discrete: We have, by definition of Z - transforms Z[H(n)] = ∞ X n=0 (H(n))z−n Expanding the summation in the RHS Z[H(n)] = H(0)z0 + H(1)z−1 + H(2)z−2 + H(3)z−3 + ... Z[H(n)] = 1 + 1 z ! + 1 z !2 + 1 z !3 + ... Dr. Vijaya kumar, MSRIT, Bengaluru - 560 054 Engineering Mathematics - IV - IM41 May 5, 2022 34 / 99
  • 35. Continued Unit - II on Z - Transforms of the Standard Functions ... We have, by Binomial expansion (1 − x)−1 = 1 + x + x2 + x3 + ... Z[H(n)] = 1 − 1 z !−1 Z[H(n)] = z − 1 z !−1 Z[H(n)] = z z − 1 12. Define unit impulse sequence for discrete and hence find its Z - transforms. Solution: Definition of the unit impulse sequence for discrete: The unit impulse sequence for discrete is denoted by δ(n) and is defined as δ(n) = ( 1, for n = 0 0, for n 6= 0 Dr. Vijaya kumar, MSRIT, Bengaluru - 560 054 Engineering Mathematics - IV - IM41 May 5, 2022 35 / 99
  • 36. Continued Unit - II on Z - Transforms of the Standard Functions ... To find the Z - transforms of the unit impulse sequence for discrete: We have, by definition of Z - transforms Z[δ(n)] = ∞ X n=0 (δ(n))z−n Expanding the summation in the RHS Z[δ(n)] = δ(0)z0 + δ(1)z−1 + δ(2)z−2 + δ(3)z−3 + ... Z[δ(n)] = 1 + (0) + (0) + (0) + ... Z[δ(n)] = 1 Dr. Vijaya kumar, MSRIT, Bengaluru - 560 054 Engineering Mathematics - IV - IM41 May 5, 2022 36 / 99
  • 37. Continued Unit - II on Formulas of the Z - Transforms of the Standard Functions ... 1. Z[1] = z z − 1 2. Z[−1] = z z + 1 3. Z[an ] = z z − a 4. Z[(−a)n ] = z z + a 5. Z[sinh(nθ)] = z sinh(θ) z2 − 2z cosh(θ) + 1 6. Z[cosh(nθ)] = z2 − z cosh(θ) z2 − 2z cosh(θ) + 1 Dr. Vijaya kumar, MSRIT, Bengaluru - 560 054 Engineering Mathematics - IV - IM41 May 5, 2022 37 / 99
  • 38. Continued Unit - II on Formulas of the Z - Transforms of the Standard Functions ... 7. Z[sin(nθ)] = z sin(θ) z2 − 2z cos(θ) + 1 8. Z[cos(nθ)] = z2 − z cos(θ) z2 − 2z cos(θ) + 1 9. Z[n] = z (z − 1)2 10. Z[n2 ] = z2 + z (z − 1)3 11. Z[n3 ] = z3 + 4z2 + z (z − 1)4 Dr. Vijaya kumar, MSRIT, Bengaluru - 560 054 Engineering Mathematics - IV - IM41 May 5, 2022 38 / 99
  • 39. Continued Unit - II on Formulas of the Z - Transforms of the Standard Functions ... 12. Z[n4 ] = z4 + 11z3 + 11z2 + z (z − 1)5 13. Z[ 1 n ] = log( z z − 1 ) 14. Z[ 1 n + 1 ] = z log( z z − 1 ) 15. Z[H(n)] = z z − 1 16. Z[δ(n)] = 1 17. Z[k] = kz z − 1 Dr. Vijaya kumar, MSRIT, Bengaluru - 560 054 Engineering Mathematics - IV - IM41 May 5, 2022 39 / 99
  • 40. Continued Unit - II Problems on Z - Transforms ... Problem - 01 : Find the Z - transforms of the 3n + sin(5n) − cosh(4n) + 5. Solution: By data 3n + sin(5n) − cosh(4n) + 5 Applying the Z - transforms for the problem Z[3n + sin(5n) − cosh(4n) + 5] = 3Z[n] + Z[sin(5n)] − Z[cosh(4n)] + Z[5] Z[3n+sin(5n)−cosh(4n)+5] = 3( z (z − 1)2 )+( z sin(5) z2 − 2z cos(5) + 1 )−( z2 − z cosh(4) z2 − 2z cosh(4) + 1 )+ Z[3n+sin(5n)−cosh(4n)+5] = 3z (z − 1)2 + z sin(5) z2 − 2z cos(5) + 1 − z2 − z cosh(4) z2 − 2z cosh(4) + 1 + 5z z − 1 Problem - 02 : Find the Z - transforms of the cos(3n + 2). Solution: By data cos(3n + 2) cos(3n + 2) = cos(3n) cos(2) − sin(3n) sin(2) Applying the Z - transforms on both sides Z[cos(3n + 2)] = Z[cos(3n) cos(2) − sin(3n) sin(2)] Z[cos(3n + 2)] = Z[cos(3n) cos(2)] − Z[sin(3n) sin(2)] Z[cos(3n + 2)] = (cos(2))Z[cos(3n)] − (sin(2))Z[sin(3n)] Dr. Vijaya kumar, MSRIT, Bengaluru - 560 054 Engineering Mathematics - IV - IM41 May 5, 2022 40 / 99
  • 41. Continued Unit - II Problems on Z - Transforms ... Z[cos(3n + 2)] = (cos(2))( z2 − z cos(3) z2 − 2z cos(3) + 1 ) − (sin(2))( z sin(3) z2 − 2z cos(3) + 1 ) Z[cos(3n + 2)] = z2 cos(2) − z cos(3) cos(2) − z sin(3) sin(2) z2 − 2z cos(3) + 1 Z[cos(3n + 2)] = z2 cos(2) − z(cos(3) cos(2) + sin(3) sin(2)) z2 − 2z cos(3) + 1 Z[cos(3n + 2)] = z2 cos(2) − z(cos(3 − 2)) z2 − 2z cos(3) + 1 Z[cos(3n + 2)] = z2 cos(2) − z cos(1) z2 − 2z cos(3) + 1 Problem - 03 : Find the Z - transforms of the sin(π 4 − nπ 2 ). Solution: Given sin(π 4 − nπ 2 ) sin( π 4 − nπ 2 ) = sin( π 4 ) cos( nπ 2 ) − cos( π 4 ) sin( nπ 2 ) But, sin(π 4 ) = 1 √ 2 and cos(π 4 ) = 1 √ 2 Dr. Vijaya kumar, MSRIT, Bengaluru - 560 054 Engineering Mathematics - IV - IM41 May 5, 2022 41 / 99
  • 42. Continued Unit - II Problems on Z - Transforms ... sin( π 4 − nπ 2 ) = ( 1 √ 2 ) cos( nπ 2 ) − ( 1 √ 2 ) sin( nπ 2 ) Applying the Z - transforms on both sides Z[sin( π 4 − nπ 2 )] = ( 1 √ 2 )Z[cos( nπ 2 ) − sin( nπ 2 )] Z[sin( π 4 − nπ 2 )] = 1 √ 2 [( z2 − z cos(π 2 ) z2 − 2z cos(π 2 ) + 1 ) − ( z sin(π 2 ) z2 − 2z cos(π 2 ) + 1 )] sin(π 2 ) = 1 and cos(π 2 ) = 0 Z[sin( π 4 − nπ 2 )] = 1 √ 2 [( z2 − z(0) z2 − 2z(0) + 1 ) − ( z(1) z2 − 2z(0) + 1 )] Z[sin( π 4 − nπ 2 )] = z2 − z √ 2(z2 + 1) Problem - 04 : Find the Z - transforms of the 4n2 + cos(nπ 4 ) + 10a3 . Solution: Given 4n2 + cos(nπ 4 ) + 10a3 Applying the z - transforms for the given problem Z[4n2 + cos( nπ 4 ) + 10a3 ] = 4Z[n2 ] + Z[cos( nπ 4 )] + Z[10a3 ] Dr. Vijaya kumar, MSRIT, Bengaluru - 560 054 Engineering Mathematics - IV - IM41 May 5, 2022 42 / 99
  • 43. Continued Unit - II Problems on Z - Transforms ... Z[4n2 + cos( nπ 4 ) + 10a3 ] = 4( z2 + z (z − 1)3 ) + ( z2 − z cos(π 4 ) z2 − 2z cos(π 4 ) + 1 ) + ( 10a3 z z − 1 ) but, cos(π 4 = 1 √ 2 Z[4n2 + cos( nπ 4 ) + 10a3 ] = 4(z2 + z) (z − 1)3 + z2 − z( 1 √ 2 ) z2 − 2z( 1 √ 2 ) + 1 + 10a3 z z − 1 Z[4n2 + cos( nπ 4 ) + 10a3 ] = 4(z2 + z) (z − 1)3 + √ 2z2 − z √ 2z2 − 2z + √ 2 + 10a3 z z − 1 Problem - 05 : Find the Z - transforms of the (3n + 2)2 . Solution: By data (3n + 2)2 (3n + 2)2 = (3n)2 + (2)2 + 2(3n)(2) (3n + 2)2 = 9n2 + 4 + 12n Applying the z - transforms on both sides Z[(3n + 2)2 ] = Z[9n2 + 4 + 12n] Z[(3n + 2)2 ] = 9Z[n2 ] + 4Z[1] + 12Z[n] Dr. Vijaya kumar, MSRIT, Bengaluru - 560 054 Engineering Mathematics - IV - IM41 May 5, 2022 43 / 99
  • 44. Continued Unit - II Problems on Z - Transforms ... Z[(3n + 2)2 ] = 9( z2 + z (z − 1)3 ) + 4( z z − 1 ) + 12( z (z − 1)2 ) Z[(3n + 2)2 ] = 9(z2 + z) (z − 1)3 + 4z z − 1 + 12z (z − 1)2 Problem - 06 : Find the Z - transforms of the (2n − 1)3 . Solution: By data (2n − 1)3 (2n − 1)3 = (2n)3 − 3(2n)2 (1) + 3(2n)(1)2 − (1)3 (2n − 1)3 = 8n3 − 12n2 + 6n − 1 Applying the z - transforms on both sides Z[(2n − 1)3 ] = Z[8n3 − 12n2 + 6n − 1] Z[(2n − 1)3 ] = 8Z[n3 ] − 12Z[n2 ] + 6Z[n] − Z[1] Z[(2n − 1)3 ] = 8( z3 + 4z2 + z (z − 1)4 ) − 12( z2 + z (z − 1)3 ) + 6( z (z − 1)2 ) − ( z z − 1 ) Z[(2n − 1)3 ] = 8(z3 + 4z2 + z) (z − 1)4 − 12(z2 + z) (z − 1)3 + 6z (z − 1)2 − z z − 1 Dr. Vijaya kumar, MSRIT, Bengaluru - 560 054 Engineering Mathematics - IV - IM41 May 5, 2022 44 / 99
  • 45. Continued Unit - II Problems on Z - Transforms ... Problem - 07 : Find the Z - transforms of the an n. Solution: By data an n We know that, by damping property Z[an un] = U( z a ) Here, un = n and U(z) = Z[n] = z (z−1)2 , Replace z by z a Z[an n] = [ z (z − 1)2 ]z→ z a Z[an n] = [ (z a ) (z a − 1)2 ] Z[an n] = az (z − a)2 Problem - 08 : Find the Z - transforms of the a−n n2 . Solution: By data a−n n2 We know that, by damping property Z[a−n un] = U(az) Here, un = n2 and U(z) = Z[n2 ] = z2 +z (z−1)3 , Replace z by az Dr. Vijaya kumar, MSRIT, Bengaluru - 560 054 Engineering Mathematics - IV - IM41 May 5, 2022 45 / 99
  • 46. Continued Unit - II Problems on Z - Transforms ... Z[a−n n2 ] = [ z2 + z (z − 1)3 ]z→az Z[a−n n2 ] = [ (az)2 + az (az − 1)3 ] Z[a−n n2 ] = a2 z2 + az (az − 1)3 Problem - 09 : Find the Z - transforms of the an n2 . Solution: By data an n2 We know that, by damping property Z[an un] = U( z a ) Here, un = n2 and U(z) = Z[n2 ] = z2 +z (z−1)3 , Replace z by z a Z[an n2 ] = [ z2 + z (z − 1)3 ]z→ z a Dr. Vijaya kumar, MSRIT, Bengaluru - 560 054 Engineering Mathematics - IV - IM41 May 5, 2022 46 / 99
  • 47. Continued Unit - II Problems on Z - Transforms ... Z[an n2 ] = [ (z a )2 + (z a ) (z a − 1)3 ] Z[an n2 ] = az2 + a2 z (z − a)3 Problem - 10 : Find the Z - transforms of the a−n ebn . Solution: By data a−n ebn We know that, by damping property Z[a−n un] = U(az) Here, un = ebn and U(z) = Z[ebn ] = z z−eb , Replace z by az Z[a−n ebn ] = [ z z − eb ]z→az Z[a−n ebn ] = [ az az − eb ] Z[a−n ebn ] = az az − eb Dr. Vijaya kumar, MSRIT, Bengaluru - 560 054 Engineering Mathematics - IV - IM41 May 5, 2022 47 / 99
  • 48. Continued Unit - II Problems on Z - Transforms ... Problem - 11 : Find the Z - transforms of the an cos(nθ). Solution: By data an cos(nθ) We know that, by damping property Z[an un] = U( z a ) Here, un = cos(nθ) and U(z) = Z[cos(nθ)] = z2 −z cos(θ) z2−2z cos(θ)+1 , Replace z by z a Z[an cos(nθ)] = [ z2 − z cos(θ) z2 − 2z cos(θ) + 1 ]z→ z a Z[an cos(nθ)] = (z a )2 − (z a ) cos(θ) (z a )2 − 2(z a ) cos(θ) + 1 Z[an cos(nθ)] = z2 − az cos(θ) z2 − 2az cos(θ) + a2 Problem - 12 : Find the Z - transforms of the a−n sin(nθ). Solution: By data a−n sin(nθ) We know that, by damping property Z[a−n un] = U(az) Here, un = sin(nθ) and U(z) = Z[sin(nθ)] = z sin(θ) z2−2z cos(θ)+1 , Replace z by az Dr. Vijaya kumar, MSRIT, Bengaluru - 560 054 Engineering Mathematics - IV - IM41 May 5, 2022 48 / 99
  • 49. Continued Unit - II Problems on Z - Transforms ... Z[a−n sin(nθ)] = [ z sin(θ) z2 − 2z cos(θ) + 1 ]z→az Z[a−n sin(nθ)] = az sin(θ) (az)2 − 2az cos(θ) + 1 Z[a−n sin(nθ)] = az sin(θ) a2z2 − 2az cos(θ) + 1 Problem - 13 : Find the Z - transforms of the an sinh(nθ). Solution: By data an sinh(nθ) We know that, by damping property Z[an un] = U( z a ) Here, un = sinh(nθ) and U(z) = Z[sinh(nθ)] = z sinh(θ) z2−2z cosh(θ)+1 , Replace z by z a Z[an sinh(nθ)] = [ z sinh(θ) z2 − 2z cosh(θ) + 1 ]z→ z a Z[an sinh(nθ)] = (z a ) sinh(θ) (z a )2 − 2(z a ) cosh(θ) + 1 Dr. Vijaya kumar, MSRIT, Bengaluru - 560 054 Engineering Mathematics - IV - IM41 May 5, 2022 49 / 99
  • 50. Continued Unit - II Problems on Z - Transforms ... Z[an sinh(nθ)] = az sinh(θ) z2 − 2az cosh(θ) + a2 Problem - 14 : Find the Z - transforms of the a−n cosh(nθ). Solution: By data a−n cosh(nθ) We know that, by damping property Z[a−n un] = U(az) Here, un = cosh(nθ) and U(z) = Z[cosh(nθ)] = z2 −z cosh(θ) z2−2z cosh(θ)+1 , Replace z by az Z[a−n cosh(nθ)] = [ z2 − z cosh(θ) z2 − 2z cosh(θ) + 1 ]z→az Z[a−n cosh(nθ)] = (az)2 − (az) cosh(θ) (az)2 − 2(az) cosh(θ) + 1 Z[a−n cosh(nθ)] = a2 z2 − az cosh(θ) a2z2 − 2az cos(θ) + 1 Dr. Vijaya kumar, MSRIT, Bengaluru - 560 054 Engineering Mathematics - IV - IM41 May 5, 2022 50 / 99
  • 51. Continued Unit - II Problems on Z - Transforms ... Problem - 15 : Find the Z - transforms of the 1 n(n+1) . Solution: By data 1 n(n+1) , adding and subtracting n in the numerator 1 n(n + 1) = n + 1 − n n(n + 1) 1 n(n + 1) = (n + 1) − n n(n + 1) 1 n(n + 1) = (n + 1) n(n + 1) − n n(n + 1) 1 n(n + 1) = 1 n − 1 n + 1 Applying the Z - transforms on both sides Z[ 1 n(n + 1) ] = Z[ 1 n ] − Z[ 1 n + 1 ] We know that Z[ 1 n ] = log( z z − 1 ) and Z[ 1 n + 1 ] = z log( z z − 1 ) Dr. Vijaya kumar, MSRIT, Bengaluru - 560 054 Engineering Mathematics - IV - IM41 May 5, 2022 51 / 99
  • 52. Continued Unit - II Problems on Z - Transforms ... Using these results in the above equation Z[ 1 n(n + 1) ] = (log( z z − 1 )) − (z log( z z − 1 )) Z[ 1 n(n + 1) ] = (1 − z) log( z z − 1 ) Problem - 16 : Find the Z - transforms of the n n+1 . Solution: By data n n+1 , adding and subtracting 1 in the numerator n n + 1 = n + 1 − 1 n + 1 n n + 1 = (n + 1) − 1 n + 1 n n + 1 = (n + 1) n + 1 − 1 n + 1 n n + 1 = 1 − 1 n + 1 Applying the Z - transforms on both sides Z[ n n + 1 ] = Z[1] − Z[ 1 n + 1 ] Dr. Vijaya kumar, MSRIT, Bengaluru - 560 054 Engineering Mathematics - IV - IM41 May 5, 2022 52 / 99
  • 53. Continued Unit - II Problems on Z - Transforms ... We know that Z[1] = z z − 1 and Z[ 1 n + 1 ] = z log( z z − 1 ) Using these results in the above equation Z[ n n + 1 ] = z z − 1 − z log( z z − 1 ) Problem - 17 : Show that ZT [ 1 n! ] = e 1 z , and hence evaluate (i) ZT [ 1 (n+1)! ], (ii) ZT [ 1 (n+2)! ]. Solution: We have, by definition of Z - transform ZT [ 1 n! ] = ∞ X n=0 ( 1 n! )z−n Expanding the summation in the RHS ZT [ 1 n! ] = [( 1 0! )z0 + ( 1 1! )z−1 + ( 1 2! )z−2 + ( 1 3! )z−3 + ...] Dr. Vijaya kumar, MSRIT, Bengaluru - 560 054 Engineering Mathematics - IV - IM41 May 5, 2022 53 / 99
  • 54. Continued Unit - II Problems on Z - Transforms ... ZT [ 1 n! ] = 1 + (1 z ) 1! + (1 z )2 2! + (1 z )3 3! + ... We know that, by exponential series ex = 1 + x 1! + x2 2! + x3 3! + ... Here x = 1 z ZT [ 1 n! ] = e 1 z (i) To evaluate ZT [ 1 (n+1)! ] Taking un = 1 n! , un+1 = 1 (n+1)! , u0 = 1 0! = 1, and U(z) = ZT [un] = ZT [ 1 n! ] = e 1 z We have, by shifting property ZT [un+1] = z[U(z) − u0] Now, substituting all results ZT [ 1 (n + 1)! ] = z[e 1 z − 1] Dr. Vijaya kumar, MSRIT, Bengaluru - 560 054 Engineering Mathematics - IV - IM41 May 5, 2022 54 / 99
  • 55. Continued Unit - II Problems on Z - Transforms ... (ii) To evaluate ZT [ 1 (n+2)! ] Taking un = 1 n! , un+2 = 1 (n+2)! , u0 = 1 0! = 1, u1 = 1 1! = 1, and U(z) = ZT [un] = ZT [ 1 n! ] = e 1 z We have, by shifting property ZT [un+2] = z2 [U(z) − u0 − u1 z ] Now, substituting all results ZT [ 1 (n + 2)! ] = z2 [e 1 z − 1 − 1 z ] Dr. Vijaya kumar, MSRIT, Bengaluru - 560 054 Engineering Mathematics - IV - IM41 May 5, 2022 55 / 99
  • 56. Continued Unit - II on Initial Value of the Z - Transforms ... Initial Value Theorem of the Z - transforms: Statement: If Z[un = U(z)], then prove that u0 = lim z→ U(z). Hence prove that (i) u1 = lim z→ z[U(z) − u0], (ii) u2 = lim z→ z2 [U(z) − u0 − u1 z ], (iii) u3 = lim z→ z3 [U(z) − u0 − u1 z − u2 z2 ]. Proof: We know that, by definition of the Z - transforms Z[un] = U(z) = ∞ X n=0 unz−n Expanding the summation in the RHS U(z) = u0z0 + u1z−1 + u2z−2 + ... U(z) = u0(1) + u1 z + u2 z2 + ... Applying the limit z → ∞ on both sides lim z→∞ U(z) = lim z→∞ [u0 + u1 z + u2 z2 + ...] lim z→∞ U(z) = [u0 + (0) + (0) + ...] Dr. Vijaya kumar, MSRIT, Bengaluru - 560 054 Engineering Mathematics - IV - IM41 May 5, 2022 56 / 99
  • 57. Continued Unit - II on Initial Value of the Z - Transforms ... u0 = lim z→∞ U(z) Hence initial value theorem is proved (i) To prove this u1 = lim z→ z[U(z) − u0], We know that, by definition of the Z - transforms Z[un] = U(z) = ∞ X n=0 unz−n Expanding the summation in the RHS U(z) = u0z0 + u1z−1 + u2z−2 + u3z−3 + ... U(z) − u0 = u1 z + u2 z2 + u3 z3 + ... Multiplying throughout by z z[U(z) − u0] = u1 + u2 z + u3 z2 + ... Applying the limit z → ∞ on both sides lim z→∞ z[U(z) − u0] = lim z→∞ [u1 + u2 z + u3 z2 + ...] lim z→∞ z[U(z) − u0] = [u1 + (0) + (0) + ...] u1 = lim z→∞ z[U(z) − u0] Dr. Vijaya kumar, MSRIT, Bengaluru - 560 054 Engineering Mathematics - IV - IM41 May 5, 2022 57 / 99
  • 58. Continued Unit - II on Initial Value of the Z - Transforms ... (ii) To prove this result u2 = lim z→ z2 [U(z) − u0 − u1 z ], We know that, by definition of the Z - transforms Z[un] = U(z) = ∞ X n=0 unz−n Expanding the summation in the RHS U(z) = u0z0 + u1z−1 + u2z−2 + u3z−3 + u4z−4 + ... U(z) − u0 − u1 z = u2 z2 + u3 z3 + u4 z4 + ... Multiplying throughout by z2 z2 [U(z) − u0 − u1 z ] = u2 + u3 z + u4 z2 + ... Applying the limit z → ∞ on both sides lim z→∞ z2 [U(z) − u0 − u1 z ] = lim z→∞ [u2 + u3 z + u4 z2 + ...] lim z→∞ z2 [U(z) − u0 − u1 z ] = [u2 + (0) + (0) + ...] Dr. Vijaya kumar, MSRIT, Bengaluru - 560 054 Engineering Mathematics - IV - IM41 May 5, 2022 58 / 99
  • 59. Continued Unit - II on Initial Value of the Z - Transforms ... u2 = lim z→∞ z[U(z) − u0 − u1 z ] (iii) To prove this result u3 = lim z→ z3 [U(z) − u0 − u1 z − u2 z2 ], We know that, by definition of the Z - transforms Z[un] = U(z) = ∞ X n=0 unz−n Expanding the summation in the RHS U(z) = u0z0 + u1z−1 + u2z−2 + u3z−3 + u4z−4 + ... U(z) − u0 − u1 z − u2 z2 = u3 z3 + u4 z4 + ... Multiplying throughout by z3 z3 [U(z) − u0 − u1 z − u2 z2 ] = u3 + u4 z + u5 z2 + ... Applying the limit z → ∞ on both sides lim z→∞ z3 [U(z) − u0 − u1 z − u2 z2 ] = lim z→∞ [u3 + u4 z + u5 z2 + ...] Dr. Vijaya kumar, MSRIT, Bengaluru - 560 054 Engineering Mathematics - IV - IM41 May 5, 2022 59 / 99
  • 60. Continued Unit - II on Initial Value and Final Value Theorem of the Z - Transforms ... lim z→∞ z3 [U(z) − u0 − u1 z − u2 z2 ] = [u3 + (0) + (0) + ...] u3 = lim z→∞ z3 [U(z) − u0 − u1 z − u2 z2 ] Final Value Theorem of the Z - Transforms: Statement: If Z[un] = U(z), then prove that lim n→∞ un = lim z→1 (z − 1)U(z). Dr. Vijaya kumar, MSRIT, Bengaluru - 560 054 Engineering Mathematics - IV - IM41 May 5, 2022 60 / 99
  • 61. Continued Unit - II Problems on Initial Value of the Z - Transforms ... Problem - 01: Find the values of the u0, u1, u2 and u3, given that U(z) = 5z2 +3z+12 (z−1)4 . Solution: By data U(z) = 5z2 +3z+12 (z−1)4 (i) To find the value of u0 We know that, by initial value of Z - transforms u0 = lim z→∞ U(z) Substituting U(z) in the RHS u0 = lim z→∞ [ 5z2 + 3z + 12 (z − 1)4 ] = lim z→∞ z2 [ 5 + 3 z + 12 z2 z4(1 − 1 z )4 ] = lim z→∞ [ 5 + 3 z + 12 z2 z2(1 − 1 z )4 ] u0 = lim z→∞ 1 z2 [ 5 + 3 z + 12 z2 (1 − 1 z )4 ] = (0)[ 5 + (0) + (0) (1 − 0)4 ] = (0)(5) u0 = 0 (ii) To find the value of u1 We know that, by initial value of Z - transforms u1 = lim z→∞ z[U(z) − u0] Dr. Vijaya kumar, MSRIT, Bengaluru - 560 054 Engineering Mathematics - IV - IM41 May 5, 2022 61 / 99
  • 62. Continued Unit - II Problems on Initial Value of the Z - Transforms ... Substituting U(z) and u0 in the RHS u1 = lim z→∞ z[ 5z2 + 3z + 12 (z − 1)4 − 0] = lim z→∞ z[ 5z2 + 3z + 12 (z − 1)4 ] = lim z→∞ z3 [ 5 + 3 z + 12 z2 z4(1 − 1 z )4 ] u1 = lim z→∞ [ 5 + 3 z + 12 z2 z(1 − 1 z )4 ] = lim z→∞ 1 z [ 5 + 3 z + 12 z2 (1 − 1 z )4 ] = (0)[ 5 + (0) + (0) (1 − 0)4 ] = (0)(5) u1 = 0 (iii) To find the value of u2 We know that, by initial value of Z - transforms u2 = lim z→∞ z2 [U(z) − u0 − u1 z ] Substituting U(z), u0 and u1 in the RHS u2 = lim z→∞ z2 [ 5z2 + 3z + 12 (z − 1)4 − 0 − 0] = lim z→∞ z2 [ 5z2 + 3z + 12 (z − 1)4 ] = lim z→∞ z4 [ 5 + 3 z + 12 z2 z4(1 − 1 z )4 ] u2 = lim z→∞ [ 5 + 3 z + 12 z2 (1 − 1 z )4 ] = [ 5 + (0) + (0) (1 − 0)4 ] = 5 1 u1 = 5 Dr. Vijaya kumar, MSRIT, Bengaluru - 560 054 Engineering Mathematics - IV - IM41 May 5, 2022 62 / 99
  • 63. Continued Unit - II Problems on Initial Value of the Z - Transforms ... (iv) To find the value of u3 We know that, by initial value of Z - transforms u3 = lim z→∞ z3 [U(z) − u0 − u1 z − u2 z2 ] Substituting U(z), u0, u1 and u2 in the RHS u3 = lim z→∞ z3 [ 5z2 + 3z + 12 (z − 1)4 − 0 − 0 − 5 z2 ] = lim z→∞ z3 [ 5z2 + 3z + 12 (z − 1)4 − 5 z2 ] u3 = lim z→∞ z3 [ 5z4 + 3z3 + 12z2 − 5(z − 1)4 z2(z − 1)4 ] But (z − 1)4 = (z − 1)2 (z − 1)2 (z − 1)4 = (z2 − 2z + 1)(z2 − 2z + 1) (z − 1)4 = (z4 − 2z3 + z2 − 2z3 4z2 − 2z + z2 − 2z + 1) (z − 1)4 = (z4 − 4z3 + 6z2 − 4z + 1) u3 = lim z→∞ z3 [ 5z4 + 3z3 + 12z2 − 5(z4 − 4z3 + 6z2 − 4z + 1) z2(z − 1)4 ] Dr. Vijaya kumar, MSRIT, Bengaluru - 560 054 Engineering Mathematics - IV - IM41 May 5, 2022 63 / 99
  • 64. Continued Unit - II Problems on Initial Value of the Z - Transforms ... u3 = lim z→∞ z3 [ 5z4 + 3z3 + 12z2 − 5z4 + 20z3 − 30z2 + 20z − 5 z2(z − 1)4 ] u3 = lim z→∞ z3 [ 23z3 − 18z2 + 20z − 5 z2(z − 1)4 ] = lim z→∞ z6 [ 23 − 18 z + 20 z2 − 5 z3 z6(1 − 1 z )4 ] u3 = lim z→∞ [ 23 − 18 z + 20 z2 − 5 z3 (1 − 1 z )4 ] = [ 23 − (0) + (0) − (0) (1 − 0)4 ] = 23 1 u3 = 23 Problem - 02: Find the values of the u0, u1, u2 and u3, when U(z) = 2z2 +4z+12 (z−1)4 . Solution: By data U(z) = 2z2 +4z+12 (z−1)4 (i) To find the value of u0 We know that, by initial value of Z - transforms u0 = lim z→∞ U(z) Substituting U(z) in the RHS u0 = lim z→∞ [ 2z2 + 4z + 12 (z − 1)4 ] = lim z→∞ z2 [ 2 + 4 z + 12 z2 z4(1 − 1 z )4 ] = lim z→∞ [ 2 + 4 z + 12 z2 z2(1 − 1 z )4 ] Dr. Vijaya kumar, MSRIT, Bengaluru - 560 054 Engineering Mathematics - IV - IM41 May 5, 2022 64 / 99
  • 65. Continued Unit - II Problems on Initial Value of the Z - Transforms ... u0 = lim z→∞ 1 z2 [ 2 + 4 z + 12 z2 (1 − 1 z )4 ] = (0)[ 2 + (0) + (0) (1 − 0)4 ] = (0)(2) u0 = 0 (ii) To find the value of u1 We know that, by initial value of Z - transforms u1 = lim z→∞ z[U(z) − u0] Substituting U(z) and u0 in the RHS u1 = lim z→∞ z[ 2z2 + 4z + 12 (z − 1)4 − 0] = lim z→∞ z[ 2z2 + 4z + 12 (z − 1)4 ] = lim z→∞ z3 [ 2 + 4 z + 12 z2 z4(1 − 1 z )4 ] u1 = lim z→∞ [ 2 + 4 z + 12 z2 z(1 − 1 z )4 ] = lim z→∞ 1 z [ 2 + 4 z + 12 z2 (1 − 1 z )4 ] = (0)[ 2 + (0) + (0) (1 − 0)4 ] = (0)(2) u1 = 0 (iii) To find the value of u2 We know that, by initial value of Z - transforms u2 = lim z→∞ z2 [U(z) − u0 − u1 z ] Dr. Vijaya kumar, MSRIT, Bengaluru - 560 054 Engineering Mathematics - IV - IM41 May 5, 2022 65 / 99
  • 66. Continued Unit - II Problems on Initial Value of the Z - Transforms ... Substituting U(z), u0 and u1 in the RHS u2 = lim z→∞ z2 [ 2z2 + 4z + 12 (z − 1)4 − 0 − 0] = lim z→∞ z2 [ 2z2 + 4z + 12 (z − 1)4 ] = lim z→∞ z4 [ 2 + 4 z + 12 z2 z4(1 − 1 z )4 ] u2 = lim z→∞ [ 2 + 4 z + 12 z2 (1 − 1 z )4 ] = [ 2 + (0) + (0) (1 − 0)4 ] = 2 1 u1 = 2 (iv) To find the value of u3 We know that, by initial value of Z - transforms u3 = lim z→∞ z3 [U(z) − u0 − u1 z − u2 z2 ] Substituting U(z), u0, u1 and u2 in the RHS u3 = lim z→∞ z3 [ 2z2 + 4z + 12 (z − 1)4 − 0 − 0 − 2 z2 ] = lim z→∞ z3 [ 2z2 + 4z + 12 (z − 1)4 − 2 z2 ] u3 = lim z→∞ z3 [ 2z4 + 4z3 + 12z2 − 2(z − 1)4 z2(z − 1)4 ] Dr. Vijaya kumar, MSRIT, Bengaluru - 560 054 Engineering Mathematics - IV - IM41 May 5, 2022 66 / 99
  • 67. Continued Unit - II Problems on Initial Value of the Z - Transforms ... But (z − 1)4 = (z − 1)2 (z − 1)2 (z − 1)4 = (z2 − 2z + 1)(z2 − 2z + 1) (z − 1)4 = (z4 − 2z3 + z2 − 2z3 4z2 − 2z + z2 − 2z + 1) (z − 1)4 = (z4 − 4z3 + 6z2 − 4z + 1) u3 = lim z→∞ z3 [ 2z4 + 4z3 + 12z2 − 2(z4 − 4z3 + 6z2 − 4z + 1) z2(z − 1)4 ] u3 = lim z→∞ z3 [ 2z4 + 4z3 + 12z2 − 2z4 + 8z3 − 12z2 + 8z − 2 z2(z − 1)4 ] u3 = lim z→∞ z3 [ 12z3 + 8z − 2 z2(z − 1)4 ] = lim z→∞ z6 [ 12 + 8 z2 − 2 z3 z6(1 − 1 z )4 ] u3 = lim z→∞ [ 12 + 8 z2 − 2 z3 (1 − 1 z )4 ] = [ 12 − (0) + (0) − (0) (1 − 0)4 ] = 12 1 u3 = 12 Dr. Vijaya kumar, MSRIT, Bengaluru - 560 054 Engineering Mathematics - IV - IM41 May 5, 2022 67 / 99
  • 68. Continued Unit - II Problems on Initial Value of the Z - Transforms ... Problem - 03: If ZT (un) = z z−1 − z z2+1 , then find ZT (un+1). Solution: By data ZT (un) = U(z) = z z−1 − z z2+1 We have, by shifting property ZT (un+1) = z2 [U(z) − u0 − u1 z ] To find the value of u0, by initial value theorem u0 = lim z→∞ U(z) u0 = lim z→∞ [ z z − 1 − z z2 + 1 ] = lim z→∞ [ z z(1 − 1 z ) − z z2(1 + 1 z2 ) ] u0 = lim z→∞ [ 1 (1 − 1 z ) − ( 1 z ) 1 (1 + 1 z2 ) ] = [ 1 (1 − 0) − (0) 1 (1 + 0) ] = [ 1 (1) − (0)( 1 (1) )] u0 = 1 To find the value of u1, by initial value theorem u1 = lim z→∞ z[U(z) − u0] u1 = lim z→∞ z[ z z − 1 − z z2 + 1 − 1] = lim z→∞ z[ z(z2 + 1) − z(z − 1) − (z − 1)(z2 + 1) (z − 1)(z2 + 1) ] Dr. Vijaya kumar, MSRIT, Bengaluru - 560 054 Engineering Mathematics - IV - IM41 May 5, 2022 68 / 99
  • 69. Continued Unit - II Problems on Initial Value of the Z - Transforms ... u1 = lim z→∞ z[ z3 + z − z2 + z − z3 − z + z2 + 1 (z − 1)(z2 + 1) ] = lim z→∞ z[ z + 1 (z − 1)(z2 + 1) ] u1 = lim z→∞ z2 [ (1 + 1 z ) z3(1 − 1 z )(1 + 1 z2 ) ] = lim z→∞ [ (1 + 1 z ) z(1 − 1 z )(1 + 1 z2 ) ] = lim z→∞ ( 1 z )[ (1 + 1 z ) (1 − 1 z )(1 + 1 z2 ) ] u0 = (0)[ (1 + 0) (1 − 0)(1 + 0) ] = (0)(1) = 0 Substituting U(z), u0 and u1 in the above equation ZT (un+1) = z2 [ z z − 1 − z z2 + 1 − 1 − (0) z ] ZT (un+1) = z2 [ z z − 1 − z z2 + 1 − 1] Dr. Vijaya kumar, MSRIT, Bengaluru - 560 054 Engineering Mathematics - IV - IM41 May 5, 2022 69 / 99
  • 70. Continued Unit - II on Inverse Z - Transforms and Formulas ... Definition of the Inverse Z - Transforms: The function U(z) is said to be inverse Z - transforms of the function un. The inverse Z - transforms of the function U(z) is denoted by Z−1 [U(z)] and is given by un = Z−1 [U(z)] where, Z−1 is the inverse Z - transforms operator. Formulas of the Inverse Z - Transforms of standard functions: 1. Z−1 [ z z − 1 ] = 1 2. Z−1 [ z z + 1 ] = −1 3. Z−1 [ z z − a ] = an 4. Z−1 [ z z + a ] = (−a)n 5. Z−1 [ 1 z − a ] = an−1 Dr. Vijaya kumar, MSRIT, Bengaluru - 560 054 Engineering Mathematics - IV - IM41 May 5, 2022 70 / 99
  • 71. Continued Unit - II on Formulas of the Inverse Z - Transforms ... 6. Z−1 [ 1 z + a ] = (−a)n−1 7. Z−1 [ 1 (z − a)2 ] = (n − 1)an−2 8. Z−1 [ 1 (z + a)2 ] = (n − 1)(−a)n−2 9. Z−1 [ 1 (z − a)3 ] = 1 2 (n − 1)(n − 2)an−3 10. Z−1 [ z2 (z − a)2 ] = (n + 1)an 11. Z−1 [ z3 (z − a)3 ] = 1 2! (n + 1)(n + 2)an un Dr. Vijaya kumar, MSRIT, Bengaluru - 560 054 Engineering Mathematics - IV - IM41 May 5, 2022 71 / 99
  • 72. Continued Unit - II on Formulas of the Inverse Z - Transforms ... 12. Z−1 [ z (z − a)2 ] = nan−1 13. Z−1 [ az (z − a)2 ] = nan 14. Z−1 [ az2 + a2 z (z − a)3 ] = n2 an 15. Z−1 [ z z − 1 ] = H(n) 16. Z−1 [1] = δ(n) Dr. Vijaya kumar, MSRIT, Bengaluru - 560 054 Engineering Mathematics - IV - IM41 May 5, 2022 72 / 99
  • 73. Continued Unit - II Problems on Inverse Z - Transforms ... Problem - 01 : Find the inverse Z - transforms of z (z−1)(z+2) . Solution: by data z (z−1)(z+2) , Take U(z) = z (z − 1)(z + 2) U(z) z = 1 (z − 1)(z + 2) (3) Applying the partial fraction for the RHS expression 1 (z − 1)(z + 2) = A (z − 1) + B (z + 2) 1 = A(z + 2) + B(z − 1) put, z = 1, we get 1 = A(3) + B(0), A = 1 3 put, z = −2, we get 1 = A(0) + B(−3), B = −1 3 . Substituting the value of A and B in the above equation 1 (z − 1)(z + 2) = 1 3(z − 1) − 1 3(z + 2) (4) Dr. Vijaya kumar, MSRIT, Bengaluru - 560 054 Engineering Mathematics - IV - IM41 May 5, 2022 73 / 99
  • 74. Continued Unit - II Problems on Inverse Z - Transforms ... Using Eqn. (4) in Eqn. (3) U(z) z = 1 3(z − 1) − 1 3(z + 2) Multiply throughout by z U(z) = 1 3 ( z z − 1 ) − 1 3 ( z z + 2 ) Applying inverse Z - transforms on both sides Z−1 [U(z)] = 1 3 Z−1 [ z z − 1 ] − 1 3 Z−1 [ z z + 2 ] Z−1 [ z (z − 1)(z + 2) ] = 1 3 (1) − 1 3 (−2)n Z−1 [ z (z − 1)(z + 2) ] = 1 3 [1 − (−2)n ] Dr. Vijaya kumar, MSRIT, Bengaluru - 560 054 Engineering Mathematics - IV - IM41 May 5, 2022 74 / 99
  • 75. Continued Unit - II Problems on Inverse Z - Transforms ... Problem - 02 : Find the inverse Z - transforms of 5z (2−z)(3z−1) . Solution: by data 5z (2−z)(3z−1) , Take U(z) = 5z (2 − z)(3z − 1) U(z) z = 5 (2 − z)(3z − 1) (5) Applying the partial fraction for the RHS expression 5 (2 − z)(3z − 1) = A (2 − z) + B (3z − 1) 5 = A(3z − 1) + B(2 − z) put, z = 2, we get 5 = A(5) + B(0), A = 1 put, z = 1 3 , we get 5 = A(0) + B(5 3 ), B = 3. Substituting the value of A and B in the above equation 5 (2 − z)(3z − 1) = 1 (2 − z) + 3 (3z − 1) (6) Dr. Vijaya kumar, MSRIT, Bengaluru - 560 054 Engineering Mathematics - IV - IM41 May 5, 2022 75 / 99
  • 76. Continued Unit - II Problems on Inverse Z - Transforms ... Using Eqn. (6) in Eqn. (5) U(z) z = 1 (2 − z) + 3 (3z − 1) Multiply throughout by z U(z) = z (2 − z) + 3 z (3z − 1) Applying inverse Z - transforms on both sides Z−1 [U(z)] = Z−1 [ z −(z − 2) ] + 3Z−1 [ z 3(z − 1 3 ) ] Z−1 [ 5z (2 − z)(3z − 1) ] = (−1)(2)2 + 3 3 ( 1 3 )n Z−1 [ 5z (2 − z)(3z − 1) ] = −(2)2 + ( 1 3 )n Dr. Vijaya kumar, MSRIT, Bengaluru - 560 054 Engineering Mathematics - IV - IM41 May 5, 2022 76 / 99
  • 77. Continued Unit - II Problems on Inverse Z - Transforms ... Problem - 03 : Find the inverse Z - transforms of 2z2 +3z (z+2)(z−4) . Solution: by data 2z2 +3z (z+2)(z−4) , Take U(z) = 2z2 + 3z (z + 2)(z − 4) U(z) z = 2z + 3 (z + 2)(z − 4) (7) Applying the partial fraction for the RHS expression 2z + 3 (z + 2)(z − 4) = A (z + 2) + B (z − 4) 2z + 3 = A(z − 4) + B(z + 2) put, z = −2, we get −1 = A(−6) + B(0), A = 1 6 put, z = 4, we get 11 = A(0) + B(6), B = 11 6 . Substituting the value of A and B in the above equation 2z + 3 (z + 2)(z − 4) = 1 6(z + 2) + 11 6(z − 4) (8) Dr. Vijaya kumar, MSRIT, Bengaluru - 560 054 Engineering Mathematics - IV - IM41 May 5, 2022 77 / 99
  • 78. Continued Unit - II Problems on Inverse Z - Transforms ... Using Eqn. (8) in Eqn. (7) U(z) z = 1 6(z + 2) + 11 6(z − 4) Multiply throughout by z U(z) = 1 6 [ z (z + 2) ] + 11 6 [ z (z − 4) ] Applying inverse Z - transforms on both sides Z−1 [U(z)] = 1 6 Z−1 [ z (z + 2) ] + 11 6 Z−1 [ z (z − 4) ] Z−1 [ 2z2 + 3z (z + 2)(z − 4) ] = 1 6 (−2)n + 11 6 (4)n Z−1 [ 2z2 + 3z (z + 2)(z − 4) ] = 1 6 [(−2)n + 11(4)n ] Dr. Vijaya kumar, MSRIT, Bengaluru - 560 054 Engineering Mathematics - IV - IM41 May 5, 2022 78 / 99
  • 79. Continued Unit - II Problems on Inverse Z - Transforms ... Problem - 04 : Find the inverse Z - transforms of 4z2 −2z z3−5z2+8z−4 . Solution: by data 4z2 −2z z3−5z2+8z−4 , Take U(z) = 4z2 − 2z z3 − 5z2 + 8z − 4 Factorize the denominator expression z3 − 5z2 + 8z − 4 = (z − 1)(z − 2)2 U(z) = 4z2 − 2z (z − 1)(z − 2)2 U(z) z = 4z − 2 (z − 1)(z − 2)2 (9) Applying the partial fraction for the RHS expression 4z − 2 (z − 1)(z − 2)2 = A (z − 1) + B (z − 2) + C (z − 2)2 4z − 2 = A(z − 2)2 + B(z − 1)(z − 2) + C(z − 1) put, z = 1, we get 2 = A(1) + B(0) + C(0), A = 2, put, z = 2, we get 6 = A(0) + B(0) + C(1), C = 6, put, z = 0, we get −2 = A(4) + B(2) + C(−1), −2 = 8 + 2B − 6, B = 0. Substituting the value of A, B and C in the above equation Dr. Vijaya kumar, MSRIT, Bengaluru - 560 054 Engineering Mathematics - IV - IM41 May 5, 2022 79 / 99
  • 80. Continued Unit - II Problems on Inverse Z - Transforms ... 4z − 2 (z − 1)(z − 2)2 = 2 (z − 1) + 0 (z − 2) + 6 (z − 2)2 4z − 2 (z − 1)(z − 2)2 = 2 (z − 1) + 6 (z − 2)2 (10) Using Eqn. (10) in Eqn. (9) U(z) z = 2 (z − 1) + 6 (z − 2)2 Multiply throughout by z U(z) = 2 z (z − 1) + 6 z (z − 2)2 Applying inverse Z - transforms on both sides Z−1 [U(z)] = 2Z−1 [ z (z − 1) ] + 6Z−1 [ z (z − 2)2 ] Z−1 [ 4z2 − 2z z3 − 5z2 + 8z − 4 ] = 2(1) + 6n(2)n−1 Dr. Vijaya kumar, MSRIT, Bengaluru - 560 054 Engineering Mathematics - IV - IM41 May 5, 2022 80 / 99
  • 81. Continued Unit - II Problems on Inverse Z - Transforms ... Z−1 [ 4z2 − 2z z3 − 5z2 + 8z − 4 ] = 2 + 6n(2)n−1 Problem - 05 : Find the inverse Z - transforms of 8z−z3 (4−z)3 . Solution: by data 8z−z3 (4−z)3 , Take U(z) = 8z − z3 (−1)3(z − 4)3 = 8z − z3 −(z − 4)3 U(z) = z3 − 8z (z − 4)3 We can observe in the denominator expression has repeated linear factors and we have Z−1 [ z (z−4) ] = 4n , Z−1 [ 4z (z−4)2 ] = n4n and Z−1 [4z2 +16z (z−4)3 ] = n2 4n We have, apply the partial fractions for the RHS expression for the above equation z3 − 8z (z − 4)3 = A z z − 4 + B 4z (z − 4)2 + C 4z2 + 16z (z − 4)3 z3 − 8z = Az(z − 4)2 + 4Bz(z − 4) + 4Cz(z + 4) Dr. Vijaya kumar, MSRIT, Bengaluru - 560 054 Engineering Mathematics - IV - IM41 May 5, 2022 81 / 99
  • 82. Continued Unit - II Problems on Inverse Z - Transforms ... Canceling z on both sides z2 − 8 = A(z − 4)2 + 4B(z − 4) + 4C(z + 4) put, z = 4, we get −8 = A(0) + 4B(0) + 4C(8), C = 1 4 . Equating the coefficient of z2 on both sides, we get, A = 1. Again by equating the coefficient of z on both sides, we have, 0 = −8A + 4B + 4C, we get, B = 7 4 . Substituting the value of A, B and C in the above equation z3 − 8z (z − 4)3 = (1) z z − 4 + ( 7 4 ) 4z (z − 4)2 + ( 1 4 ) 4z2 + 16z (z − 4)3 Applying the inverse Z - transforms on both sides Z−1 [ z3 − 8z (z − 4)3 ] = Z−1 [ z z − 4 ] + 7 4 Z−1 [ 4z (z − 4)2 ] + 1 4 Z−1 [ 4z2 + 16z (z − 4)3 ] Using the above results, we get Z−1 [ z3 − 8z (z − 4)3 ] = 4n + 7 4 n4n + 1 4 n2 4n Dr. Vijaya kumar, MSRIT, Bengaluru - 560 054 Engineering Mathematics - IV - IM41 May 5, 2022 82 / 99
  • 83. Continued Unit - II Problems on Inverse Z - Transforms ... Problem - 06 : Find the inverse Z - transforms of z3 −20z (z−2)3(z−4) . Solution: by data z3 −20z (z−2)3(z−4) , Take U(z) = z3 − 20z (z − 2)3(z − 4) We can observe in the denominator expression has repeated linear factors in first factor and we have Z−1 [ z (z−2) ] = 2n , Z−1 [ 2z (z−2)2 ] = n2n , Z−1 [2z2 +4z (z−2)3 ] = n2 4n and Z−1 [ z (z−4) ] = 4n We have, apply the partial fractions for the RHS expression for the above equation z3 − 20z (z − 2)3(z − 4) = A z z − 2 + B 2z (z − 2)2 + C 2z2 + 4z (z − 2)3 + D z z − 4 z3 − 20z = Az(z − 2)2 (z − 4) + 2Bz(z − 2)(z − 4) + C(2z2 + 4z)(z − 4) + Dz(z − 2)3 Dr. Vijaya kumar, MSRIT, Bengaluru - 560 054 Engineering Mathematics - IV - IM41 May 5, 2022 83 / 99
  • 84. Continued Unit - II Problems on Inverse Z - Transforms ... Canceling z on both sides z2 − 20 = A(z − 2)2 (z − 4) + 2B(z − 2)(z − 4) + C(2z + 4)(z − 4) + D(z − 2)3 put, z = 2, we get −16 = A(0) + 2B(0) + C(−16) + D(0), C = 1. put, z = 4, we get −4 = A(0) + 2B(0) + C(0) + D(8), D = −1 2 . Equating the coefficient of z3 on both sides, we get, A + D = 0, A = 1 2 . put, z = 0, we get −20 = A(4)(−4) + 2B(8) + C(−16) + D(−8), B = 0. Substituting the value of A, B, C and D in the above equation z3 − 20z (z − 2)3(z − 4) = ( 1 2 ) z z − 2 + (0) 2z (z − 2)2 + (1) 2z2 + 4z (z − 2)3 + ( −1 2 ) z z − 4 Applying the inverse Z - transforms on both sides Z−1 [ z3 − 20z (z − 2)3(z − 4) ] = 1 2 Z−1 [ z z − 2 ] + Z−1 [ 2z2 + 4z (z − 2)3 ] − 1 2 Z−1 [ z z − 4 ] Using the above results, we get Z−1 [ z3 − 20z (z − 2)3(z − 4) ] = 1 2 2n + n2 2n − 1 2 4n = 2n−1 + n2 2n − 22n−1 Dr. Vijaya kumar, MSRIT, Bengaluru - 560 054 Engineering Mathematics - IV - IM41 May 5, 2022 84 / 99
  • 85. Continued Unit - II on Application of the Z - Transforms to Solve the Difference Equations ... Introduction: Definition of Difference equation: An equation which involving the set of values of the dependent variable is known as a difference equation. Let us consider the second order forward difference of y0 is equal to zero, that is ∆2 y0 = 0 ∆(∆y0) = 0 we have, ∆y0 = y1 − y0 ∆(y1 − y0) = 0 ∆y1 − ∆y0 = 0 (y2 − y1) − (y1 − y0) = 0 y2 − 2y1 + y0 = 0 This kind of equation is called as a difference equation Examples: 1. yn+2 + 5yn+1 + 6yn = 3n . 2. un+2 − 3yn+1 + 2yn = n2 . Dr. Vijaya kumar, MSRIT, Bengaluru - 560 054 Engineering Mathematics - IV - IM41 May 5, 2022 85 / 99
  • 86. Continued Unit - II on Application of the Z - Transforms to Solve the Difference Equations ... Working procedure to solve the difference equations: Step - 01: Applying the Z - transforms on both sides. Step - 02: Substituting the shifting property of Z - transforms, that is Z[un+1] = z[U(z) − u0] Z[un+2] = z2 [U(z) − u0 − u1 z ] Z[un+3] = z3 [U(z) − u0 − u1 z − u2 z2 ] Step - 03: Substituting the given initial conditions and simplify U(z) in terms of z. Step - 04: Applying the inverse Z - transforms on both sides, we get the solution of the given difference equation. Dr. Vijaya kumar, MSRIT, Bengaluru - 560 054 Engineering Mathematics - IV - IM41 May 5, 2022 86 / 99
  • 87. Continued Unit - II Problems on Application of the Z - Transforms to Solve the Difference Equations ... Problem - 01: Solve the difference equation un+2 − 5un+1 + 6un = 2, given that u0 = 3, u1 = 7, by using Z - transforms. Solution: By data un+2 − 5un+1 + 6un = 2, and u0 = 3, u1 = 7 Applying the Z - transforms on both sides Z[un+2] − 5Z[un+1] + 6Z[un] = Z[2] Substituting the shifting property of the Z - transforms z2 [U(z) − u0 − u1 z ] − 5(z[U(z) − u0]) + 6(U(z)) = ( 2z z − 1 ) Substituting the given initial conditions u0 = 3 and u1 = 7 z2 [U(z) − (3) − 7 z ] − 5(z[U(z) − (3)]) + 6(U(z)) = ( 2z z − 1 ) Simplify z2 U(z) − 3z2 − 7z − 5zU(z) + 15z + 6U(z) = 2z z − 1 z2 U(z) − 5zU(z) + 6U(z) = 2z z − 1 + 3z2 + 7z − 15z Dr. Vijaya kumar, MSRIT, Bengaluru - 560 054 Engineering Mathematics - IV - IM41 May 5, 2022 87 / 99
  • 88. Continued Unit - II Problems on Application of the Z - Transforms to Solve the Difference Equations ... (z2 − 5z + 6)U(z) = 2z z − 1 + 3z2 − 8z (z − 2)(z − 3)U(z) = 2z + 3z2 (z − 1) − 8z(z − 1) z − 1 U(z) = 3z3 − 11z2 + 10z (z − 1)(z − 2)(z − 3) U(z) z = 3z2 − 11z + 10 (z − 1)(z − 2)(z − 3) (11) Now, applying the partial fractions for the RHS expression 3z2 − 11z + 10 (z − 1)(z − 2)(z − 3) = A z − 1 + B z − 2 + C z − 3 3z2 − 11z + 10 = A(z − 2)(z − 3) + B(z − 1)(z − 3) + C(z − 1)(z − 2) put, z = 1, we get, 2 = A(−1)(−2) + B(0) + C(0), A = 1. put, z = 2, we get, 0 = A(0) + B(1)(−1) + C(0), B = 0. put, z = 3, we get, 4 = A(0) + B(0) + C(2)(1), C = 2. Substituting the values of the A, B and C in the above equation Dr. Vijaya kumar, MSRIT, Bengaluru - 560 054 Engineering Mathematics - IV - IM41 May 5, 2022 88 / 99
  • 89. Continued Unit - II Problems on Application of the Z - Transforms to Solve the Difference Equations ... 3z2 − 11z + 10 (z − 1)(z − 2)(z − 3) = 1 z − 1 + 0 z − 2 + 2 z − 3 3z2 − 11z + 10 (z − 1)(z − 2)(z − 3) = 1 z − 1 + 2 z − 3 Using Eqn. (12) in Eqn. (11) U(z) z = 1 z − 1 + 2 z − 3 (12) Multiply throughout by z U(z) = z z − 1 + 2 z z − 3 Applying inverse Z - transforms on both sides Z−1 [U(z)] = Z−1 [ z z − 1 ] + 2Z−1 [ z z − 3 ] but, Z−1 [U(z)] = un un = (1) + 2(3n ) = 1 + 23n This is required solution for the given difference equation. Dr. Vijaya kumar, MSRIT, Bengaluru - 560 054 Engineering Mathematics - IV - IM41 May 5, 2022 89 / 99
  • 90. Continued Unit - II Problems on Application of the Z - Transforms to Solve the Difference Equations ... Problem - 02: Solve yn+2 + 2yn+1 + yn = n, given that y0 = 0, y1 = 0, by using Z - transforms. Solution: Given yn+2 + 2yn+1 + yn = n and y0 = 0, y1 = 0 Applying the Z - transforms on both sides Z[yn+2] + 2Z[yn+1] + Z[yn] = Z[n] Substituting the shifting property of the Z - transforms z2 [Y (z) − y0 − y1 z ] + 2(z[Y (z) − y0]) + (Y (z)) = ( z (z − 1)2 ) Substituting the given initial conditions y0 = 0 and y1 = 0 z2 [Y (z) − (0) − 0 z ] + 2(z[Y (z) − (0)]) + (Y (z)) = z (z − 1)2 Simplify z2 Y (z) − (0) − (0) + 2zY (z) − (0) + Y (z) = z (z − 1)2 (z2 + 2z + 1)Y (z) = z (z − 1)2 Dr. Vijaya kumar, MSRIT, Bengaluru - 560 054 Engineering Mathematics - IV - IM41 May 5, 2022 90 / 99
  • 91. Continued Unit - II Problems on Application of the Z - Transforms to Solve the Difference Equations ... (z + 1)2 Y (z) = z (z − 1)2 Y (z) = z (z − 1)2(z + 1)2 Y (z) z = 1 (z − 1)2(z + 1)2 (13) Applying the partial fractions for the RHS expression 1 (z − 1)2(z + 1)2 = A (z − 1) + B (z − 1)2 + C (z + 1) + D (z + 1)2 1 = A(z − 1)(z + 1)2 + B(z + 1)2 + C(z + 1)(z − 1)2 + D(z − 1)2 put, z = 1, we get, 1 = A(0) + B(4) + C(0) + D(0), B = 1 4 . put, z = −1, we get, 1 = A(0) + B(0) + C(0) + D(4), D = 1 4 . Rewrite the above equation 1 = A(z3 + z2 − z − 1) + B(z2 + 2z + 1) + C(z3 − z2 − z + 1) + D(z2 − 2z + 1) Equating the coefficient of z3 on both sides A + C = 0 Equating the coefficient of z2 on both sides A + B − C + D = 0, A − C = −1 2 , Dr. Vijaya kumar, MSRIT, Bengaluru - 560 054 Engineering Mathematics - IV - IM41 May 5, 2022 91 / 99
  • 92. Continued Unit - II Problems on Application of the Z - Transforms to Solve the Difference Equations ... solving these two equations, we get A = −1 4 and C = 1 4 . Substituting the values of A, B, C and D in the above equation. 1 (z − 1)2(z + 1)2 = −1 4(z − 1) + 1 4(z − 1)2 + 1 4(z + 1) + 1 4(z + 1)2 (14) Using Eqn. (14) in (13) Y (z) z = −1 4(z − 1) + 1 4(z − 1)2 + 1 4(z + 1) + 1 4(z + 1)2 Multiplying throughout by z Y (z) = − 1 4 z z − 1 + 1 4 z (z − 1)2 + 1 4 z z + 1 + 1 4 z (z + 1)2 Applying the inverse Z - transforms on both sides Z−1 [Y (z)] = − 1 4 [Z−1 [ z z − 1 ] + Z−1 [ z (z − 1)2 ] + Z−1 [ z z + 1 ] + Z−1 [ z (z + 1)2 ]] Dr. Vijaya kumar, MSRIT, Bengaluru - 560 054 Engineering Mathematics - IV - IM41 May 5, 2022 92 / 99
  • 93. Continued Unit - II Problems on Application of the Z - Transforms to Solve the Difference Equations ... but, Z−1 [Y (z)] = yn yn = 1 4 [−(1) + n(1)n−1 + (−1)n + n(−1)n−1 ] = 1 4 [−1 + n + (−1)n + n(−1)n−1 ] This is required solution for the given difference equation. Problem - 03: Solve yn+2 − 4yn+1 + 3yn = H(n), given that y0 = 0, y1 = 0, by using Z - transforms. Solution: Given yn+2 − 4yn+1 + 3yn = H(n) and y0 = 0, y1 = 0 Applying the Z - transforms on both sides Z[yn+2] − 4Z[yn+1] + 3Z[yn] = Z[H(n)] Substituting the shifting property of the Z - transforms z2 [Y (z) − y0 − y1 z ] − 4(z[Y (z) − y0]) + 3(Y (z)) = ( z (z − 1) ) Substituting the given initial conditions y0 = 0 and y1 = 0 z2 [Y (z) − (0) − 0 z ] − 4(z[Y (z) − (0)]) + 3(Y (z)) = z (z − 1) Dr. Vijaya kumar, MSRIT, Bengaluru - 560 054 Engineering Mathematics - IV - IM41 May 5, 2022 93 / 99
  • 94. Continued Unit - II Problems on Application of the Z - Transforms to Solve the Difference Equations ... Simplify z2 Y (z) − (0) − (0) − 4zY (z) − (0) + 3Y (z) = z (z − 1) (z2 − 4z + 3)Y (z) = z (z − 1) (z − 1)(z − 3)Y (z) = z (z − 1) Y (z) = z (z − 1)2(z − 3) Y (z) z = 1 (z − 1)2(z − 3) (15) Applying the partial fraction for RHS expression in the above equation 1 (z − 1)2(z − 3) = A (z − 1) + B (z − 1)2 + C (z − 3) 1 = A(z − 1)(z − 3) + B(z − 3) + C(z − 1)2 Dr. Vijaya kumar, MSRIT, Bengaluru - 560 054 Engineering Mathematics - IV - IM41 May 5, 2022 94 / 99
  • 95. Continued Unit - II Problems on Application of the Z - Transforms to Solve the Difference Equations ... put, z = 1, we get, 1 = A(0) + B(−2) + C(0), B = −1 2 . put, z = 3, we get, 1 = A(0) + B(0) + C(4), C = 1 4 . put, z = 0, we get, 1 = A(3) + B(−3) + C(1), A = −1 4 . Substituting the values of the A, B and C in the above equation 1 (z − 1)2(z − 3) = −1 4(z − 1) + −1 2(z − 1)2 + 1 4(z − 3) (16) Using Eqn. (16) IN Eqn.. (15) Y (z) z = −1 4(z − 1) + −1 2(z − 1)2 + 1 4(z − 3) Multiplying throughout by z Y (z) = − 1 4 z (z − 1) − 1 2 z 2(z − 1)2 + 1 4 z (z − 3) Applying the inverse Z - transforms on both sides Z−1 [Y (z)] = − 1 4 Z−1 [ z (z − 1) ] − 1 2 Z−1 [ z (z − 1)2 ] + 1 4 Z−1 [ z (z − 3) ] Dr. Vijaya kumar, MSRIT, Bengaluru - 560 054 Engineering Mathematics - IV - IM41 May 5, 2022 95 / 99
  • 96. Continued Unit - II Problems on Application of the Z - Transforms to Solve the Difference Equations ... But, Z−1 [Y (z)] = yn yn = − 1 4 (1) − 1 2 (n) + 1 4 (3)n = −1 4 − n 2 + (3)n 4 This is required solution for the given difference equation. Problem - 04: Solve the difference equation un+2 − 5un+1 + 6un = Hn, given that u0 = 0, u1 = 1, where Hn is a unit step sequence, by using Z - transforms. Solution: Given un+2 − 5un+1 + 6un = Hn and u0 = 0, u1 = 1 Applying the Z - transforms on both sides Z[un+2] − 5Z[un+1] + 6Z[un] = Z[Hn] Substituting the shifting property of the Z - transforms z2 [U(z) − u0 − u1 z ] − 5(z[U(z) − u0]) + 6(U(z)) = ( z (z − 1) ) Substituting the given initial conditions u0 = 0 and u1 = 1 z2 [U(z) − (0) − 1 z ] − 5(z[U(z) − (0)]) + 6(U(z)) = z (z − 1) Dr. Vijaya kumar, MSRIT, Bengaluru - 560 054 Engineering Mathematics - IV - IM41 May 5, 2022 96 / 99
  • 97. Continued Unit - II Problems on Application of the Z - Transforms to Solve the Difference Equations ... z2 U(z) − 0 − z − 5zU(z) − 0 + 6U(z) = z (z − 1) (z2 − 5z + 6)U(z) = z (z − 1) + z = z + z2 − z (z − 1) (z − 2)(z − 3)U(z) = z2 (z − 1) U(z) = z2 (z − 1)(z − 2)(z − 3) U(z) z = z (z − 1)(z − 2)(z − 3) (17) Applying the partial fractions for the RHS expression z (z − 1)(z − 2)(z − 3) = A (z − 1) + B (z − 2) + C (z − 3) z = A(z − 2)(z − 3) + B(z − 1)(z − 3) + C(z − 1)(z − 2) Dr. Vijaya kumar, MSRIT, Bengaluru - 560 054 Engineering Mathematics - IV - IM41 May 5, 2022 97 / 99
  • 98. Continued Unit - II Problems on Application of the Z - Transforms to Solve the Difference Equations ... put, z = 1, we get, 1 = A(2) + B(0) + C(0), A = 1 2 . put, z = 2, we get, 2 = A(0) + B(−2) + C(0), B = −1 2 . put, z = 3, we get, 3 = A(0) + B(0) + C(2), C = 3 2 . Substituting the values of the A, B and C in the above equation z (z − 1)(z − 2)(z − 3) = 1 2(z − 1) + −1 2(z − 2) + 3 2(z − 3) Using Eqn. (18) in Eqn. (17) U(z) z = 1 2(z − 1) + −1 2(z − 2) + 3 2(z − 3) Multiplying throughout by z U(z) = 1 2 z z − 1 − 1 2 z z − 2 + 3 2 z z − 3 Dr. Vijaya kumar, MSRIT, Bengaluru - 560 054 Engineering Mathematics - IV - IM41 May 5, 2022 98 / 99
  • 99. Continued Unit - II Problems on Application of the Z - Transforms to Solve the Difference Equations ... Applying the inverse Z - transforms on both sides Z−1 [U(z)] = 1 2 Z−1 [ z z − 1 ] − 1 2 Z−1 [ z z − 2 ] + 3 2 Z−1 [ z z − 3 ] But, Z−1 [U(z)] = un un = 1 2 (1) − 1 2 (2)n + 3 2 (3)n = 1 2 − (2)n−1 + (3)n+1 2 This is required solution for the given difference equation. Dr. Vijaya kumar, MSRIT, Bengaluru - 560 054 Engineering Mathematics - IV - IM41 May 5, 2022 99 / 99